You are on page 1of 157

CYTOLOGY. CELL MEMABRANE.

Tests of the “KROK-1” database

2015

A large number of cisterns flattened in the center and extended on the periphery and small
vesicles with secretory granules were found in the biopsy hepatocytes during the electron
microscopic study of the biliary pole. Name this organelle:
A. *Golgi apparatus
B. Lysosomes
C. Endoplasmic reticulum
D. Pinocytotic vesicles
Е. Microtubules

The organism to be identified has a nucleus surrounded by a nuclear membrane. Genetic material is
concentrated predominantly in the chromosomes that consist of DNA strands and protein
molecules. These cells divide mitotically. Identify these organisms:
A. *Eukaryotes
B. Bacteriophages
C. Prokaryotes
D. Viruses
E. Bacteria

2014

An electron micrograph shows a cell-to-cell adhesion consisting, in each cell, of an attachment


plaque. The intercellular space is filled with electron-dense substance including transmembrane
fibrous structures. Specify this adhesion:
A *Desmosomes
B Synapse
C Tight junction
D Nexus
E Adherens junction

2013

(questions are absent)

2012

(questions are absent)

2005-2011

The action of microbial toxins on the cells caused significant damage of the glycocalyx. What
function of the cell membrane will be significantly disordered?
A *Receptor
B Transport
C Respiratory
1
D Formation of contact
E Protective

The harmful factors caused the acute disorder of the endocytosis and exocytosis in the liver and
blood cells. What layer of the cell membrane was first injured?
A. *Cortical layer
B. The layer of the lipids end proteins
C. Upper part of membrane
D. Integral
E. Glycocalyx

CYTOLOGY. CYTOPLASM. ORGANELLS. INCLUSIONS.

2015
(questions are absent)

2014

The secretory granules appear and disappear in the cytoplasm of the pancreatic cells during the
secretory cycle. Which structural elements can these granules be related to?
A *To inclusions
B To microfilaments
C To lysosomes
D To exocytosis vacuoles
E To granular endoplasmic reticulum

Significant reduction of the protein synthesis in hepatocytes resulted by the long-term effects of the
toxic substances on the body. What organelles are most affected by intoxication?
A *Granular endoplasmic reticulum
B Mitochondria
C Microtubules
D Lysosomes
E Golgi apparatus

2013

In the tissue culture the nucleoluses were damaged by the radioactive irradiation. Recovery of what
organelles in the cytoplasm becomes problematic?
A *Ribosomes
B Lysosomes
C Endoplasmic reticulum
D Microtubule
E Golgi apparatus

The structure of the ribosome was disordered in the cell. What process is primarily affected?
A *Protein synthesis (translation)
B Protein synthesis (transcription)
C Synthesis of carbohydrates
D Synthesis of lipids
E Synthesis of minerals

The researchers destroyed the structure of one of the cell parts during a scientific experiment. As a
2
result the cell lost the ability to division. What structure was broken?
A *Centrosome
B Glycocalyx
C Plastic complex
D Microfibrille
E Mitochondria

2012

The patient was hospitalized in the hospital with poisoning. It was established that the
detoxification processes were disordered in the liver. Which organelles of hepatocytes were
injured?
A *Agranular endoplasmic reticulum
B Mitochondria
C Granular endoplasmic reticulum
D Golgi apparatus
E Ribosomes

The organelles, which consist of cisterns that flattened in the center and extended on the periphery
and small vesicles, were found at the electron microphotography of the nervous cells. What are
these organelles?
A *Golgi apparatus
B Centrioles
C Lysosomes
D Peroxisomes
E Mitochondria

The abnormal biopolymers were found in the body cells of the child (7 years) with congenital
"storage diseases". Name this organelle.
A *Lysosomes
B Ribosomes
C Granular endoplasmic reticulum
D Mitochondria
E Peroxisomes

A high content of hydrolytic enzymes in the cytoplasm was founded during the examination. Which
organelle activity shows this fact?
A *Lysosomes
B Mitochondria
C Polysomes
D Endoplasmic reticulum
E Centrioles

The organelle that is the big polyprotease complex and consists of tubular and two regulatory parts
that are located at both ends of organelle was representing on electronic photo. The function of this
organelle is proteolysis. Name this organelle.
A *Proteasome
B Centrioles
C Inclusion
D Ribosome
E Golgi complex

3
The vesicles with peroxide oxidation enzymes - catalase, peroxidase (0,05-1,5 microns in diameter)
were revealed in the cytoplasm of hepatocytes during the histochemical investigation. What are
these organelles?
A * Peroxisomes
B Lysosomes
C Melanosomes
D Liposomes
E Phagosomes

Low level of albumins and fibrinogen was detected in the patient's blood. What organelle decreased
activity of the liver hepatocytes can cause it?
A *Rough endoplasmic reticulum
B Smooth endoplasmic reticulum
C Mitochondria
D Golgi complex
E Lysosomes

2005-2011

At what component of the cell the lysosomes formation takes place?


A. *At the Golgi apparatus
B. At the nucleus
C. At the ribosomes
D. At the mitochondria
E. At the organizing cell center

The microtubules contain the protein:


A. *Tubulin
B. Desmin
C. Dynein
D. Calmodulin
E. Vimentin

An organelle that participates in glucosylation of proteins and lipids with the formation of
glycosaminoglycans was detected in the histological preparation of the spinal ganglion in the
pseudounipolar neurons after impregnation with silver salts. Name this organelle.
A *Golgi apparatus
B Mitochondria
C Rough endoplasmic reticulum
D Smooth endoplasmic reticulum
E Centrioles

Labeled aminoacids, alanine and tryptophane were introduced to a mouse in order to study
localization of protein biosynthesis in its cells. Around what organelles will the accumulation of
labeled amino acids be observed?
A *Ribosomes
B Agranular endoplasmic reticulum
C Cell center
D Lysosomes
E Golgi apparatus

4
Ultramicroscopic examination of «dark» hepatocytes population in the cell cytoplasm detected a
developed granular endoplasmic reticulum. What function has this organelle in these cells?
A *Synthesis of blood plasma proteins
B Carbohydrate synthesis
C Detoxification
D Bile production
E Calcium ion depositing

The hysterectomy was made to a 67-year-old woman because of a tumor. At the histological
investigation of this tumor the multipolar mitoses were found at the tumor cells (chromosome
migration to more than two cellular poles). Which organelles disorder can this multipolar mitosis
cause?
A *Centrioles
B Secondary lysosomes
C Smooth endoplasmic reticulum
D Rough endoplasmic reticulum
E Peroxisomes

The cell was treated by a substance, which blocks the nucleotide phosphorylation process in the
mitochondria. What process will be broken?
A. *ATP resynthesize
B. Synthesis of mitochondrial proteins
C. Oxidative phosphorylation
D. Integration of functional protein molecules
E. Fragmentation of large mitochondria smaller

The cell of the laboratory animal was overdosed with Roentgen rays. As a result albuminous
fragments formed in the cytoplasm. What cell organelle will take part at their utilization?
A *Lysosomes
B Endoplasmic reticulum
C Cells center
D Golgi complex
E Ribosome

CYTOLOGY. NUCLEUS. CELL CICLE.

2015
(questions are absent)

2014

The histone proteins synthesis was artificially blocked in the cell. What cell structure will be
damaged?
A *Nuclear chromatin
B Nucleolus
C Golgi apparatus
D Cell membrane
E Nuclear envelope

2013

The cell without the nucleus and nucleolus is presented at the electron microphotography. The
5
chromosomes are free; the centrioles migrate to the poles. In which phase of the cell cycle is the
cell?
A *In prophase
B In anaphase
C In metaphase
D In telophase
E In interphase

2012

The culture of the tumor cells was affected by colchicine, which inhibits formation of the proteins-
tubulins that are necessary for the spindle apparatus formation. What stage of the cell cycle will be
affected?
A *Mitosis
B Presynthetic period
C Synthetic period
D Postsynthetic period
E G0 - phase

Human somatic cells in the metaphase were found in the histological preparation. How many
chromosomes the metaphase plate consists of in case if each chromosome has two sister
chromatids?
A *46 chromosomes
B 92 chromosomes
C 23 chromosomes
D 48 chromosomes
E 24 chromosomes

The cells whose nuclei contain sex chromatin (Barr body) were found in the amniotic fluid during
the investigation that obtained by amniocentesis (amniotic membrane puncture). What could it
mean?
A *Development of the female sex fetus
B Development of the male sex fetus
C Genetic abnormalities in fetal development
D Trisomy
E Рolyploidy

2005-2011

It is known that the syntheses of the proteins-tubulins are blockaded under the colchicine action.
What stage of the cell cycle will be disordered in this case?
A. *Postsynthetic (premitotic) period of the interphase
B. Prophase of mitosis
C. Synthetic period of the interphase
D. Formation of the metaphase plate
E. Presynthetic (postmitotic) period of the interphase

A very low content of heterochromatin was found at the hepatocyte nuclei during microscopic
study. What functional state of the cell does it mean?
A *Increase of protein synthesis
B Apoptosis
C Necrosis
6
D Reduction of protein synthesis
E Entry into mitosis

Compaction of the nucleus was observed on the electron microphotography of the cells taken from
animals after a chemical exposure. What is the state of the nucleus in the cell?
A *Karyopyknosis
B Mitosis
C Amitosis
D Polyploidy
E Meiosis

At the electronic microphotography the nucleus is surrounded by the thin lamina of biological
membrane. How does it called?
A * Nuclear envelope
B Plasmolemma
C Cytolemma
D Nuclear pore
E Pore complex

The navy-blue chromatin granules were found at the nucleus on the slide that is stained by
hematoxylin and eosin. What is the stage of the cell cycle?
A * Interphase
B Prophase
C Metaphase
D Anaphase
E Telophase

A few cells of the oral cavity epithelium were taken for the examination. Sex chromatin (Barr body)
was found in the nuclei of epithelial cells after special processing of the histological specimen.
Which chromosome forms a sex chromatin?
A *Х – female condensed chromosome
B Х – female dispersed chromosome
C Х – male condensed chromosome
D Х – male dispersed chromosome
E Y – condensed chromosome

While studying maximally spiraled chromosomes of human karyotype the process of cell division
was stopped in the following phase:
A *Metaphase
B Prophase
C Interphase
D Anaphase
E Telophase

Moving of the daughter chromatids to the poles of the cell is observed in the mitotically dividing
cell. On what stage of the mitotic cycle is this cell?
A *Anaphase
B Metaphase
C Telophase
D Prophase
E Interphase
7
The study of mitotic cycle phases of the onion root revealed the cell, in which the chromosomes are
situated in the equatorial plane, forming a star. What stage of the cell mitosis is it?
A *Metaphase
B Anaphase
C Telophase
D Interphase
E Prophase

The syntheses of proteins - tubulines, which take part in the mitosis formation, were destroyed
during the postsynthetic period of mitotic cycle. It can cause the impairment of:
A *Chromosome separation
B Cytokinesis
C Duration of mitosis
D Chromosome despiralization
E Chromosome spiralization

There are three periods of the cell cycle interphase. What process takes the place during the S-
period?
A *Replication of the DNA
B Meiosis
C Cytokinesis
D Mitosis
E Аmitosis

Cells in the tissue culture were affected by colchicine, a substance that inhibits the contraction of
microtubules of the spindle apparatus, with the aim of obtaining a human karyotype. At which stage
will mitosis be stopped?
A *Metaphase
B Telophase
C Interphase
D Anaphase
E Prophase

It is known that apoptosis is programmed cell death that occurs in human during the whole
ontogenesis. Which of the following features are typical for apoptosis?
A * Activation of genes whose products kill the cells are taking place during apoptosis.
B Condensation of the nucleus and the cytoplasm
C The cell which dies often breaks up into fragments.
D Genes which primarily breaking down DNA into large and then into smaller fragments are
activated during apoptosis
E Apoptosis often occurs in the embryonic period.

A common physiological phenomenon which is programmed and genetically controlled process that
is induced by killer genes is observed during embryogenesis. In this case there are condensation of
the nucleus, condensation of chromatin and disintegration of the nucleus into the “micronuclei”,
condensation of the cytoplasm and its fragmentation. The cell splits into bodies containing the
“micronuclei” that are phagocytized by macrophages. What phenomenon is this?
A *Apoptosis
B Pyknosis
C Necrosis
D Endomitosis
8
E Endoreproduction
A tumor of the pyloric part of the stomach was diagnosed in the patient. What proteins are able to
inhibit the mitotic division of newly formed cells?
A. *Keylons
B. Ribonuclease
C. Lysozyme
D. Hemoglobin
E. Myoglobin

The scraping from the mucous membrane of the cheeks was taken. A smear was made and stained
with methylene blue. Hemispherical clumps of heterochromatin near the inner membrane of the
epithelial cells nuclei are visible under a microscope. What structure is this?
A *Barr body
B Fordyce spots
C Bisha adipose body
D У-chromosome
E Ribosomes

EMBRYOLOGY. COMPARATIVE EMBRYOLOGY OF BIRDS AND MAMMALS

Tests of the “KROK-1” database

2015
(questions are absent)

2014
(questions from this topic are absent)

2013

The chicken embryo at the stage of the mesoderm differentiation to somites and splanchnotom was
revealed in the histological preparation. What material is the axial skeleton developing from?
A *Sclerotome
B Dermatome
C Nephrotome
D Splanchnotom
E Myotome

The gametes precursors (gonoblasts) were revealed in the embryo at 2 nd - 3rd weeks of
embryogenesis. Where differentiate these cells?
A *In the yolk sac
B In the mesenchyme
C In the embryonic ectoderm
D In dermatomes
E In the embryonic endoderm

Study of the biopsy material of an embryo revealed a zone of developmental abnormality in a


somite. The zone was located close to the endoderm and the notochord. What formations may
abnormal development in case of pregnancy continuation have?
A *Skeletal tissues
B Genito-urinary system
9
C Skeletal striated muscle tissue
D Cardiac striated muscle tissue
E Fibrous connective tissue of the skin
2012

The sclerotome was destroyed in the bird embryo during the experiment. What structure disorder by
this manipulation can be caused?
A * Axial skeleton
B Skin connective tissue
C Internal organs stroma
D Gonadal stroma
E Chord

The myotome was destroyed in the rabbit embryo during the experiment. What structure disorder
by this manipulation can be caused?
A * Skeletal muscle
B Axial skeleton
C Skin connective tissue
D Smooth muscle
E Serous membranes

The outer germ layer (ectoderm) was destroyed in the frog embryo during the experiment. What
morphological structure of this embryo will be not developed?
A * Epidermis
B Somites
C Nephrotome
D Splanchnotom
E Bone tissue

2011-2005
(questions from this topic are absent)

EMBRYOLOGY. EMBRYOLOGY OF HUMAN 1

2015
An embryo at an early gastrula stage was found during forensic medical examination of a woman
who died in a car accident. Name the place of its localization under normal development.
A. The wall of the uterus
B. Ampulla of the uterine tube
C. Uterine part of oviduct
D. Ovary
Е. Abdominal cavity

2014

The embryo on the stage of early gastrulation was found during the forensic medical expertise of a
woman who died in the road accident. Name the place of the embryo localization in case of its
normal development.
A *Uterus wall
B Ampullar part of the oviduct
C Uterus part of the oviduct
D Ovary
10
E Abdominal cavity

The embryo composed of to blastomeres was found during the microscopic investigation of female
reproductive organs removed during the operation. Name the place of the embryo localization in
case of its normal development.
A * Oviduct, close to its ampullar part
B Oviduct, close to its uterus part
C Uterus cavity
D Abdominal cavity
E Ovary

2013

The process of a zygote cleavage ends with the blastula formation. What type of blastula is typical
for a human?
A * Blastocysts
B Coeloblastula
C Discoblastula
D Amphiblastula
E Morula

2012

The human embryo that doesn’t attach to the endometrium was found in the uterus cavity. What
stage of embryogenesis is this?
A * Blastocyst
B Zygote
C Morula
D Gastrula
E Neurula

2011-2005

The human ovum represented on histological preparation. There are small amount of yolk
inclusions in the cytoplasm of it. Identify the type of ovum.
A *Secondary isolecithal
B Isolecithal
C Telolecithal
D Alecithal
E Centrolecital

Chronic poisoning of 38-year-old miner by cadmium compounds resulted in male sterility. Semen
analysis revealed the inability of sperm to move. Damage of what cytoskeleton components for this
pathology may be responsible?
A *Axonemal microtubules
B Actin microfilaments
C Intermediate filaments
D Microtubule spindle
E Actomyosine complex

EMBRYOLOGY. EMBRYOLOGY OF HUMAN 2


11
2014

The primary Hensen's node wasn’t formed in the embryo during the gastrulation. The development
of what axial organ will be inhibited?
A * Chorda
B Neural crest
C Neural groove
D Neural tube
E Mantle layer of the neural tube

2013

The human blastocyst implantation begins. What period of embryogenesis does at the same time
with implantation start?
A * Gastrulation
B Invagination
C Differentiation
D Histogenesis
E Cleavage

The chorion is determined at the microscopic examination of the embryo membranes. What is the
main function of this organ?
A * Exchange of substances between the mother and fetus
B Hematopoietic
C Production of the amniotic fluid.
D Formation of the primordial germ cells.
E Formation of the lymphocytes

The early gastrulation of the human embryo occurs by delamination of the embryoblast. At what
structure does the nervous system rudiment situate?
A * At the epiblast
B At the trophoblast
C At the hypoblast
D At the marginal zone of the hypoblast
E At the central zone of the hypoblast

2012

The hydramnios was diagnosed at the pregnant women during the ultrasound examination. Disorder
of what extraembryonic organ function can in such pathological condition result?
A *Amniotic membrane
B Chorion
C Placenta
D Yolk sac
E Allantois

12
Ectodermal cells differentiated into neuroblasts and spongioblasts in the neural tube of the human
embryo. Layers formed in consequence of the movement of these cells in the neural tube. Which
layer mainly contained bodies of the neuroblasts?
A *Mantle layer
B Ependymal layer
C Edge veil
D White matter
E Sheath of the spinal cord

Embryonic shield with two layers of cells (ectoderm and endoderm) was revealed in the human
embryo taken from spontaneous abortion. At what stage of an embryonic development was an
embryo?
A * Gastrulation
B Cleavage
C Progenesis
D Organogenesis
E-

It is known that the megalocytes may appear in the human peripheral blood. When the appearance
of these cells in the blood is normal?
A * In the embryonic period
B At the age of 1 year
C At the age from 1 to 30 years
D In old age
E During the pregnancy

The human embryo implantation in the uterine wall (during the 7 th day) is the one of the critical
periods of embryogenesis. What gastrulation process does in embryoblast during this period occur?
A * Delamination
B Migration
C Epiboly
D Invagination
E Neurulation

"A person was born in a shirt." What kind of "shirt" is referred in this proverb about?
A * Amniotic
B Yolk
C Serous
D Chorionic
E Trophoblastic

2011-2005

It is known that cells of neural crest migrate for long distances and are the progenitors of many cell
types in the human body. What cells do not develop from neural crest?
A. *Myocytes
B. The nerve cells of the autonomic ganglia
C. Medulla of adrenal glands
D. Melanocytes
E. Sensitive nuclei of cranial nerves

13
Disorder of the cerebrum development was found during the ultrasonic scanning of the pregnant
woman. The chronic alcoholism is in anamnesis. What critical period of the embryogenesis can this
pathology related to?
A. *15-20 weeks of embryogenesis
B. 20-24 weeks of embryogenesis
C. 7-8 days of embryogenesis
D. 3-8 weeks of embryogenesis
E. Neonatal period

The mesenchyme at the yolk-sac wall was destroyed on the early stage of embryogenesis. What
consequences can be happened after this manipulation?
A. *Disorder of the blood vessels formation
B. Disorder of trophic of the embryo
C. Disorder of metabolic products excretion
D. Disorder of the allantois formation
E. Disorder of the amnion formation

During the examination of a pregnant woman, the doctor drew attention to the data of anamnesis,
which indicated that in the 3-4 weeks of pregnancy woman had an acute infectious disease. Which
stage of embryogenesis may be disordered?
A. *Formation of the intestinal tube.
B. Implantation
C. Placentation
D. Development of the brain
E. Formation of the reproductive system

The works of Speman prove that the development of certain structures of the embryo is a result of
the action of inductors. What develops under the influence of inducers of the dorsal lip of
blastopore?
A. *Neural tube
B. Chorda
C. Mesoderm
D. Primary gut
E. Somites

Sex hormone supports the function of the corpus luteum. The level of this hormone in a woman’s
blood used as a pregnancy test. Which structure produces this hormone at the end of the second
week of embryo development?
A *Syncytiotrophoblast
B Cytotrophoblast
C Amniotic epithelium
D Epithelium of the yolk sac
E Allantois

Neural tube, neural crests, placodes, skin ectoderm and prechordal plate develop from the
embryonic ectoderm during the process of differentiation. How is called the process of the neural
tube formation?
A *Neurulation
14
B Gastrulation
C Somitogenesis
D Histogenesis
E Organogenesis

The germ of a provisional organ shaped as elastic cord, covered by amniotic epithelium and
surrounded by mucous connective tissue is determined during the study of the early stages of
human embryogenesis. Name this organ.
A *Umbilical cord
B Yolk sac
C Allantois
D Placenta
E Amnion

EPITHELIAL TISSUE.

Tests of the “KROK-1” database

2015
(questions are absent)

2014
(questions from the named topic are absent)

2013

The villus of the small intestine is covered by the tissue that consists only of the cells, which form a
layer on the basement membrane. The tissue does not contain blood vessels. What tissue does the
surface of the villus cover?
A * Epithelial tissue
B Cemented irregular fibrous connective tissue
C Dense irregular connective tissue
D Smooth muscle tissue
E Reticular tissue

2012

The structures of tight junction between the epithelial cells were affected during the experiment.
What function of the epithelium will be disordered?
A * Mechanical
B Absorption
C Vitamin D-producing
D Secretory
E Excretory

2001-2011
(questions from the named topic are absent)

15
EPITHELIAL TISSUE. GLANDULAR EPITHELIUM.

Tests of the “KROK-1” database

2015
(questions are absent)

2014
A slide contains the preparation of a gland composed of several secretory saccule-shaped parts that
open in the common excretory duct. What gland is it?
A *Simple branched alveolar gland
B Compound branched alveolar gland
C Simple unbranched alveolar gland
D Compound unbranched alveolar gland
E Simple branched tubular gland

The secretory parts of the apocrine glands contain myoepithelial cells. What is the function of these
cells?
A *Contractive
B Secretory
C Protective
D Regenerative
E Supporting

2013
(questions from the named topic are absent)

2012

A diagram shows an exocrine gland that has unbranched excretory duct with a terminal part in form
of a saccule opening into the duct. How is this gland called according to the morphological
classification of exocrine glands?
A *Simple unbranched alveolar
B Compound branched alveolar
C Simple branched tubular
D Compound unbranched alveolar
E Compound unbranched alveolar tubular

2005-2011

The gland that consists of two acinar secretory parts which open into common excretory duct was
found in the skin. What kind of a gland is this?
A *Simple branched alveolar gland
B Compound branched alveolar gland
C Simple unbranched alveolar gland
D Compound unbranched alveolar gland
E Simple branched tubular gland

BLOOD
BLOOD. RED BLOOD CELLS.

16
Tests of the “KROK-1” database
2015
18% of erythrocytes of the spherical, flattened, globular and spinous forms were revealed in the
blood of a 26 years old man. Other erythrocytes were in the form of the biconcave disks. How is
this phenomenon called?
A. Physiological poikylocytosis
B. Pathological poikilocytosis
C. Physiological anisocytosis
D. Pathological anisocytosis
E. Erythrocytosis

2014

The patient blood was taken for analysis. The 30% of red blood cells with an irregular shape were
found. Clinically, this phenomenon is described as:
A * Pathological poikylocytosis
B Anisocytosis
C Physiological poikylocytosis
D Macrocytosis
E Microcytosis

2013
(questions from the named topic are absent)

2012

The acute decline of the hemoglobin was revealed in the patient's blood during the examination at
the clinic. What function of the blood will be disordered?
A * Respiratory
B Humoral
C Homeostatic
D Protective
E Trophic

The anemia developed in a 50-year-old patient with chronic nephritis. What was the most likely
cause of the anemia at this patient?
A * Decreasing of erythropoietin production
B Absence of gland
C Lack of vitamin B12
D Disorders of porphyrin synthesis
E Immunological damage cells - precursors of erythropoiesis

The reduced hemoglobin amount was revealed in the blood test. What function of the blood will be
disordered?
A * Transport of gases
B Transport of hormones
C Providing immunity
D Clotting
E Transport of nutrients

It was revealed 18% of erythrocytes of the spherical, ball-shaped, flat and thorn-like shape in the
17
blood of a 26-year-old man. Other erythrocytes were in the form of the biconcave disks. How is
such phenomenon called?
A *Physiological poikylocytosis
B Pathological poikylocytosis
C Physiological anisocytosis
D Pathological anisocytosis
E Erythrocytosis

The 12,5% erythrocytes with diameter greater than 8 microns and 12.5% erythrocytes with diameter
less than 6 microns were revealed in the patient's blood. The rest of the red blood cells have a
diameter of 7,1–7,9 microns. What is the name of this phenomenon?
A * Physiological anisocytosis
B Pathological anisocytosis
C Physiological poikylocytosis
D Pathological poikylocytosis
E Erythrocytosis

2001-2011

Transfusion of Rh-positive blood to Rh-negative patient results in the formation of Rh antibodies


and hemolysis. What blood cells are the carriers of Rh factor?
A *Erythrocytes
B Platelets
C Lymphocytes
D Monocytes
E Neutrophils

LYMPH. WHITE BLOOD CELLS. PLATELETS.

Tests of the “KROK-1” database

2015
(questions are absent)

2014

The signs of inflammation: pain, redness, edema as signs of immediate hypersensitivity were
revealed at the child around the skin wound. Which blood cells do these changes cause?
A * Basophils
B Eosinophils
C Neutrophils
D Lymphocytes
E Monocytes

The forensic pathologist determined that it was female blood according to results of the blood stains
studying at the crime scene. What signs were present on the ground of this conclusion?
A * Presence of the satellite in the neutrophils nuclei
B Presence of the microcytes and macrocytes
C Poikylocytosis
D Presence of the eosinophils specific granules
E According to the number of red blood cells

18
The cells with the histamine and heparin granules in the cytoplasm were defined in the blood smear.
What kind of cell is this?
A * Basophils
B Neutrophils
C Eosinophils
D Monocytes
E Erythrocytes

The cells, which accounts for 0,5% of the total leukocytes number with the S-shaped curved
nucleus and metachromatic colored granules in the cytoplasm, were founded in the patient's blood
smear. What are these cells?
A * Basophils
B Neutrophils
C Eosinophils
D Monocytes
E Lymphocytes

The numerous plasma cells plasmocytes were revealed in the blood of a 16 years old girl with
autoimmune inflammation of the thyroid gland. With the proliferation and differentiation of what
blood cells the increase of the plasmocytes number is associated?
A * B-lymphocytes
B T-helper
C Tissue basophils
D T-killer
E T-suppressor

The B-lymphocytes were marked with the tracer in the experiment. The foreign protein was injected
under the skin of the animal. Which cells will this tracer contain in the connective tissue?
A * Plasmocytes
B T-lymphocytes
C Macrophages
D Tissue basophils
E Fibroblasts

One of the blood cells populations was selectively stimulated in the experiment. Permeability of
blood vessels was significantly increased as result of it. That leads to edema of perivascular tissue
end deceleration of the blood clotting. Which blood cells were stimulated?
A * Basophils
B Erythrocytes
C Platelets
D Eosinophils
E Lymphocytes

The rounded cells with the segmented nuclei are predominating of the leucocytes in the smear of
peripheral blood. The granules of their cytoplasm stained with both acidic and basic dyes. What are
these cells?
A * Segmented neutrophils
B Basophils
C Eosinophils
D Young neutrophils
E Monocytes

19
The large cells with low-basophilic cytoplasm and bean-shaped nucleus were found in a blood
smear. The cell is the largest from visible in the visual field. What are the cells?
A * Monocytes
B Macrophages
C Plasmocytes
D Middle lymphocytes
E Small lymphocytes

2013
A specimen of the human red bone marrow smear revealed accumulation of gigantic cells located
near sinusoidal capillaries. Name formed elements of blood which formed from these cells.
A. *Platelets
B. Red blood cells
C. White blood cells
D. Lymphocytes
E. Monocytes

To determine the functional activity of blood cells the suspension of the microorganisms was
introduced into a test tube containing leukocyte mass. Inside of what cells will the phagocytized
bacteria be detected?
A * Neutrophils and monocytes
B Lymphocytes and basophils
C Lymphocytes and eosinophils
D Monocytes and lymphocytes
E Lymphocytes and neutrophils

The neutrophils were defined during the study of the connective tissue slide. What is the function of
these cells in the tissues?
A * Phagocytosis of microorganisms
B Trophic
C Support function
D Regulation of the smooth muscle cells contraction
E Dilatation of the blood vessels.

It is known that plasmocytes produce specific antibodies against the antigen. The number of
plasmocytes increases after the antigen introduction. At the expense of which blood cells will the
plasmocytes number be increased?
A * B-lymphocytes
B T-lymphocytes
C Monocytes
D Basophils
E Eosinophil

2012

The increasing of the total leukocytes number was revealed in the general blood analysis of a
patient with pneumonia. What is the name of this phenomenon?
A * Leukocytosis
B Anemia
C Leukopenia
D Anisocytosis
E Poikylocytosis
20
The chromatin of one of the neutrophils nucleus segments is forming the drumstick. What is the
name of this structural formation?
A * Barr's body
B Lyon’s body
C Decondensed chromatin
D Euchromatin
E Pacinian corpuscles

The helminthiasis was diagnosed at the 6-year-old child. What changes of the leukogram (WBC)
can be expected?
A * Increasing of the eosinophils number
B Increasing of the neutrophils number
C Reducing the eosinophils number
D Increasing of the monocytes number
E Increasing of the lymphocytes number

The nurse complains about hands injury that resembles the eczema. She said that after streptomycin
injection (she makes it to the patient) the skin itching is increased and the vesicles with the watery
fluid appear on the skin. The symptoms disappear during the holidays. The blood test was made on
suspicion of allergic reaction. Increased number of what blood cells can be detected?
A * Eosinophilic leukocytes
B Basophilic leukocytes
C Monocytes
D Neutrophilic leukocytes
E Lymphocytes

The 20% large (diameter 20 mm), rounded cells with low-basophilic cytoplasm and bean-shaped
nucleus were found in a blood smear. Clinically, this phenomenon is described as:
A * Monocytosis
B Lymphocytosis
C Leukopenia
D Neutrophilia
E Reticulocytosis

The cells, which make up 0.5% of the total white blood cells number, were revealed at the patient's
blood smear. They have S-shaped nucleus and metachromatic colored granules in the cytoplasm.
What are these cells?
A *Basophils
B Neutrophils
C Eosinophils
D Monocytes
E Lymphocytes

The helminthiasis was detected at the child (10 years). What changes can be expected in the
leukogram (WBC)?
A * Number of eosinophils will increase
B Number of platelets will increase
C Number of red blood cells will increase
D Number of segmented neutrophils will increase
E Number of basophils will increase
21
In the analysis the laboratory assistant made an additional conclusion that the blood belongs to the
female. Specific features of what cells structure enable us to conclusion?
A * Neutrophilic leukocytes
B Erythrocytes
C Lymphocytes
D Monocytes
E Basophilic leukocytes

Blood analysis is recommended to be performed on an empty stomach and in the morning. What
changes in the blood composition can occur if to perform blood test after food intake?
A *Increased contents of leukocytes
B Increased contents of erythrocytes
C Increased plasma proteins
D Reduced contents of thrombocytes
E Reduced contents of erythrocytes

2005-2011
Plasma cell produces specific antibody for specific antigen. During injection of antigen quantity of
plasma cell increased. At the expense of what blood cells occurred increasing of plasma cells
quantity?
A. *B lymphocytes
B. Eosinophils
C. Basophils
D. T lymphocytes
E. Monocytes

A large number of rounded cells with segmented nucleus (three or more segments) and small pink-
purple granules in the cytoplasm were revealed in a blood smear of patient with pneumonia. What is
this cell?
A *Neutrophils
B Monocytes
C Eosinophilic granulocytes
D Basophilic granulocytes
E Lymphocytes

General blood test of a child found the percentage of lymphocytes and neutrophils in the ratio of
45:45. Which age may correspond to a physiological crossroads?
A * 5 days and 5 years
B 14 years
C 1 day and 3 years
D 1 year
E 1 day

Presence of inflammation in the kidney observed at a patient. Blood test was made. There was
observed a large number of cells with following cell morphology: nucleus with 2 - 5 segments, pink
– purple granules in the cytoplasm. What is this cell?
A *Neutrophilic granulocytes segmented
B Monocytes
C Lymphocytes
D Acidophilic granulocytes
22
E Basophilic granulocytes

A blood test was made to a patient with allergic rhinitis. In the blood was observed a large number
of cells with following cell morphology: nucleus with 2 - 3 segments, oxyphilic cytoplasm filled
with bright pink large granules. What cell is this?
A *Acidophilic granulocytes
B Lymphocytes
C Monocytes
D Basophilic granulocytes
E Neutrophils

Among of blood cells there are cells whose number is 3-11% of the total number of leukocytes. The
main characteristic of these cells is to converse into a macrophagic system cells and phagocytosis.
What are these cells?
A *Monocytes
B Basophils
C Eosinophils
D Lymphocytes
E Neutrophils

A blood test made to a patient with pneumonia. The blood test reveals increased number of the cells
that have bean-shaped and rod-shaped nucleus, small granules, some of which are dyed basophilic
other oxyphilic. Name such changes of leukogram (WBC).
A. *Leukocyte formula shift to the left
B. Leukocyte formula shift to the right
C. Neuthrocytopenia
D. Neuthrocytosis
E. Lymphocytosis

Philopodium of megakaryocytes are going through the pores of red bone marrow sinusoidal
capillaries into the lumen of blood vessels, where they are fragmenting into individual plates. What
blood cells are formed in this way?
A * Platelets
B Erythrocytes
C Lymphocytes
D Reticulocytes
E Monocytes

In blood was observed a large number of cells with following cell morphology: nucleus with 2 - 3
segments, a cytoplasm filled with large granules that at staining showing metachromasia. What are
these cells?
A. *Basophils
B. Neutrophils
C Eosinophils
D. Monocytes
E. Red blood cells

One of the blood cells populations was selectively stimulated. As a result of it the process of blood
clotting was significantly slowed. Which blood cells subjected to stimulation?
A. *Basophils
B. Platelets
C. Monocytes
23
D. Eosinophils
E. Neutrophils

16% rounded cells with sizes 4.5-7 mm, having a large spherical nucleus, basophilic cytoplasm
forming a narrow border around the nucleus were detected in a blood smear of a patient after
suffering from flu. What state of blood do they describe?
A. * Lymphocytopenia
B. Monocytosis
C. Neuthrocytosis
D. Lymphocytosis
E. Monocytopenia

HEMATOPOIESIS.

Tests of the “KROK-1” database

2015

During postembryonic hematopoiesis in the red bone marrow the cells of one of the cellular
differons demonstrate gradual decrease in cytoplasmic basophilia as well as increase in oxyphilia,
the nucleus is being forced out. Such morphological changes are typical for the following
hematopoietic type:
A. Erythropoiesis
B. Lymphopoiesis
C. Neutrophil cytopoiesis
D. Eosinophil cytopoiesis
E. Basophil cytopoiesis

2014

The significant reduction of megakaryocytes was found during the histological investigation of the
red bone marrow punctate of a 35-year-old patient. How will the correlation of blood cells change
in this case?
A * Number of platelets will decrease
B Number of red blood cells will decrease
C Number of eosinophils will decrease
D Number of neutrophils will decrease
E Number of B-lymphocytes will decrease

It is known that megalocytes can be present in the peripheral blood of human. When can these cells
be present in the peripheral blood in normal condition?
A *In the embryogenesis
B Till the age 1 year
C At the age from 1 to 30
D At the old age
E During the pregnancy

2013
(questions from the named topic are absent)

2012
24
The megakaryocyte with demarcation channels in the peripheral part of the cytoplasm was
determined at the electron microphotograph of the red bone marrow. What is the function of these
structures?
A * Formation of platelets
B Increasing of the cell surface
C Increase of the ion channels number
D Cell division
E Cell destruction

The cells of granulocytic series were revealed at the biopsy material of the red bone marrow.
Specify what changes do in the nucleus occur during the differentiation of these cells?
A * Segmentation
B Polyploidisation
C Pyknosis
D Enucleation
E Increasing the size

The cells, differentiation of which is characterized by pycnosis and nucleus removing, were
revealed in the myeloid tissue punctate of a 6-year-old child. What kind of hematopoiesis is
characterized by such morphological changes?
A * Erythropoiesis
B Thrombocytopoiesis
C Lymphocytopoiesis
D Granulocytopoiesis
E Monocytopoiesis

A newborn baby has disorder of the thymus development. What type of the hematopoiesis will be
disordered?
A * Lymphocytopoiesis
B Monocytopoiesis
C Erythropoiesis
D Granulocytopoiesis
E Thrombocytopoiesis

2005-2011

In course of an experiment, a big number of stem cells of the red bone marrow were in some way
destruct. What cell population regeneration in the loose connective tissue will be inhibited?
A *Of macrophages
B Of fibroblasts
C Of pigment cells
D Of lipocytes
E Of pericytes

Clusters of developing erythrocytes surround and receive iron from some cell in groupings called
erythroblastic islands. Identify this cell.
A *Macrophage
B Fibroblasts
C Endothelial
25
D Lipocyte
E Pericyte

The least differentiated cells that are forming populations and characterized by self-inducing
differentiation in several ways through the progenitor cells into functionally mature cells, are
present at every tissue during the human life (or were present in embryogenesis). What are these
cells?
A * Stem cells
B Specialized cells
C Multifunction cells
D Cells of histogenesis
E Blastomere

The cells that characterized by polychromatophilia, appears of meshwork (remnants of granular


endoplasmic reticulum and free ribosomes) in the cytoplasm present in the blood smear. Their
number is normally 1-5% of the total number of red blood cells. Increased number of them is
diagnostic feature of enhanced hematopoiesis. What are these cells?
A * Reticulocytes
B Erythroblasts
C Erythroblasts polychromatophilic
D Basophilic erythroblast
E Pre-erythroblast

A significant number of hematopoietic cells were died after the massive exposure of the mice in the
laboratory condition. There were a rapid renewal of the stem cells and all committed progenitor
cells after a while. At the level of which class of the blood cell lineages the hormonal regulation of
the process of hematopoiesis by hemopoietin’s is happened?
A. *Ш class – unipotential progenitor cells
B. ІІ class – polypotential partially determinate progenitor cells
C. І class – polypotential blood stem cells
D. V class – maturing cells
E. VI class – mature cells

CONNECTIVE TISSUE. LOOSE CONNECTIVE TISSUE. CELLS.

Tests of the “KROK-1” database

2015
The inflammatory process in tissues and organs is accompanied by their redness and swelling. What
leukocytes in the connective tissue provide the expansion of blood vessels and increase their
permeability?
A. Basophils
B. Neutrophils
C. Eosinophils
D. Т-lymphocytes
E. В-lymphocytes

2014
The cells of next morphology: intensely basophilic cytoplasm, eccentrically placed nucleus with
chromatin, which is located in a "spoke wheel" and highlights the cytoplasm near it – were found in
the lymph node histological sections of the experimental animals after the antigen stimulation. What

26
is the type of cell?
A * Plasmocytes
B Macrophages
C Fibroblasts
D Adipocytes
E Tissue basophils (mast cells)

The histamine plays a central role in the development of allergic clinical manifestations. What cells
produce it?
A * Mast cells
B T lymphocytes
C Macrophages
D B-lymphocytes
E Plasmocytes

The local esophagus stenosis as a result of scar formation developed at the patient after the
esophagus chemical burn. Which cells of the loose connective tissue involved in the formation of
scars?
A * Mature specialized fibroblasts
B Young unspecialized fibroblasts
C Fibrocytes
D Myofibroblasts
E Fibroclasts

2013

A foreign body entered into the skin and leads to inflammation development. What connective
tissue cells take place in the skin reaction against of the foreign body?
A * Neutrophils, macrophages, fibroblasts
B Macrophages
C Melanocytes
D Adipocytes
E Adventitial cells

2012
The cesarean section was performed to a patient. The uterus wall was cut and fetus released. By
what mechanism the myometrium regeneration will be in the area of the wound?
A *Formation of connective tissue scar
B Formation of new smooth muscle
C Formation of striated muscle fibers
D Proliferation of myosatellitocytes
E Hypertrophy of smooth muscle cells

A live vaccine was injected into a human organism. Increased activity of what connective tissue
cells can be expected?
A *Plasma cells and lymphocytes
B Macrophages and fibroblasts
C Pigmented cells and pericytes
D Adipocytes and adventitial cells
E Fibroblasts and labrocytes

27
The connective tissue scar is developing in the wound place during the healing. Which cells provide
the process?
A * Fibroblasts
B Macrophages
C Fibrocytes
D Mast cells
E Melanocytes

The allergic dermatitis of both hands developed at the women as a result of the contact with the
chromium compounds at the production. Which skin cells mainly participated in the realization of
this disease?
A * Tissue basophils
B Plasmocytes
C Macrophages
D Neutrophils
E Lymphocytes

A big number of the elongated cells with dense nucleus and many lysosomes, phagosomes and
pinocytotic vacuoles in the basophilic cytoplasm were found in the lavage of the patient with acute
tibia wound. What are these cells?
A * Macrophages of connective tissue
B Fibroblasts
C Fibrocytes
D Plasmocytes
E Tissue basophils

In course of an experiment a big number of stem cells of red bone marrow was destructed in some
way. Regeneration of what cell populations will be inhibited in the loose connective tissue?
A *Of macrophages
B Of fibroblasts
C Of pigment cells
D Of lipocytes
E Of pericytes

A significant number of red bone marrow stem cells was destroyed at the experiment. What cells of
connective tissue renewal will be inhibited?
A * Macrophages
B Fibroblasts
C Pigment cells
D Adipocytes
E Pericytes

An inflammation is characterized by the dilation of the blood capillaries at the site of injury,
decreased blood circulation, increased vascular permeability. Which of the following cells play the
primary role in these processes?
A * Tissue basophils
B Fibroblasts
C Plasmocytes
D Eosinophils
E Macrophages

2005-2011
28
After the radioactive exposure a patient has stem cells disorder. The regeneration of what cells of
friable connective tissue will be damaged?
A. *Macrophages
B. Pericytes
C. Fibroblasts
D. Pigment cells
E. Adipocytes

Low concentration of the specific antibodies was found in the blood of a patient with influenza. The
function of what cell of connective tissue is inhibited?
A. *Plasmocyte
B. Lymphocyte
C. Macrophage
D. Macrophage
E. Labrocyte

The microscopic examination of wound lavage of a patient with acute wound process of his shin
revealed big content of irregular extended-formed cells, with dense nucleus, the basophilic
cytoplasm which includes many lysosomes, phagosomes and pinocytotic vesicles. What cells were
found in the wound?
A. Tissue basophils
B. *Connective tissue macrophages
C. Fibrocytes
D. Fibroblasts
E. Plasmocytes

A bleeding accompanied by slow blood clotting (at normal number of platelets in blood tests)
occurred in women after the limb injury. What substance of loose connective tissue prevents the
blood clotting process? By what cell does it produced?
A. * Heparin, tissue basophils
B. Heparin, macrophages
C. Histamine, tissue basophils
D. Heparin, plasma cells
E. Heparin, fibroblast

The allergic reaction in the form of urticaria developed at a patient (formations of the vesicles under
the epidermis as result of the plasma release into loose connective tissue). What substance of mast
cells causes an increase of blood vessel permeability?
A. * Histamine
B. Heparin
C. Hyaluronic acid
D. Chondroitin sulfate type A
E. Chondroitin sulfate type C

Leading role in the vascular phase of inflammation plays the histamine. Which cell of loose
connective tissue produces histamine?
A. * Tissue basophils
B. Fibroblasts
C. Plasma cell
D. Fibrocyte
E. Macrophage

29
The drug ketotifen, which is able to inhibit the histamine release from a particular cell type, is used
for long-term therapy in patients with asthma. Name these cells.
A *Mast cells
B Lymphocytes
C Eosinophils
D Macrophages
E Plasma cells

Increased number of basophils in the connective tissue of the mucous is accompanied by edema of
the tissues during allergic rhinitis (inflammation of the nasal mucosa). With what function of the
tissue basophils does it connected?
A * Biogenic amines synthesis
B Intercellular substance production
C Phagocytosis
D Antibody production
E Heat production

CONNECTIVE TISSUE. DENCE CONNECTIVE TISSUE. INTRACELLULAR SPACE.

Tests of the “KROK-1” database

2015

A slide represents a tissue with spherical cells, each of them containing a large fat drop covered
with thin cytoplasm layer in its center. Nucleus is compressed and situated at the cell periphery.
What tissue is it?
A. White adipose tissue
B. Brown adipose tissue
C. Mucous tissue
D. Pigmented tissue
E. Reticular tissue

2014

The lower limb was injured during the athlete training. Traumatologist diagnosed the tendon
rupture. What type of connective tissue the tendon belongs to?
A * Dense regular fibrous tissue
B Dense irregular fibrous tissue
C Loose connective tissue
D Reticular tissue
E Cartilage tissue

2013

The damaged Achilles tendon function was renewed after the treating. What is the mechanism of
tendon regeneration?
A * Synthesis of the collagen fibers
B Synthesis of the hyaline cartilage
C Formation of the adipose tissue
D Synthesis of the fibrous cartilage
E Replacements of the place of injury by the muscle tissue
30
Decreased blood supply of the organs causes hypoxia that activates fibroblasts function. What
elements volume is increased in this case?
A *Intercellular substance
B Vessels of microvascular bed
C Nerve elements
D Parenchymatous elements of the organ
E Lymphatic vessels

2012-

The substance that disorders the collagen fibers formation was introduced to the animals. How will
change the tendon properties in this case?
A * The tendon hardness to a rupture will decrease
B No change
C The tendon elasticity will decrease
D The tendon hardness to a rupture and elasticity will decrease
E The tendon hardness to a rupture will increase but elasticity will decrease

2005-2011

Collagen, elastin and reticulin belong to the fibrillar elements of the connective tissue. Indicate the
aminoacid which constitutes only collagen, and identification of which in biological fluids is used
for the diagnosing of the connective tissue diseases.
A. *Hydroxyproline
B. Proline
C. Lysine
D. Phenylalanine
E. Glycine

The hyaluronidase increase occurs at a patient under the action of bacteria. How the intracellular
substance permeability will change in this case?
A. *Increase of permeability
B. No effect
C. Reduce of permeability
D. Slowing of metabolism
E. Reduce the content of glycosaminoglycans

BONE TISSUE

Tests of the “KROK-1” database

2015
(questions are absent)

2014

The local resorption of some bones was noticed by the physician on the roentgenogram of a 57-
year-old patient. With what cells the increased activity can these changes be associated?
A * Osteoclasts
B Chondroblasts
31
C Osteocytes
D Osteoblasts
E Chondrocytes

2013

The signs of regenerative process (callus) are present in the histological preparation of the bones.
What tissue forms this structure?
A * Woven (nonlamellar) bone
B Loose connective tissue
C Reticular tissue
D. Epithelial tissue
E Lamellar bone

2012

A tissue was represented in the histological slide. The cells of this tissue don’t have the processes.
Each one of them contains tens of nuclei and one border is ruffled. Through this border the
secretion of hydrolytic enzymes is going. What tissue is presented on histological slide?
A * Bone tissue
B Cartilage tissue
C Epithelial tissue
D Nervous tissue
E Muscular tissue

The ossification resulted by the increasing of the calcium level in the bones was revealed at the
worker of the enterprise, which produces vanadium compounds. With activity of what cells can it
be associated?
A * Osteoblast
B Osteocytes
C Osteoclasts
D Chondrocytes
E Fibroblasts

The resorption of bone was revealed at the patient. With the increased activity of what cells can it
be associated?
A * Osteoclast
B Osteoblasts and osteoclasts
C Osteocytes and osteoblasts
D Osteoblasts
E Osteocytes

The patient with a diagnosis of the clavicle fracture was admitted into a hospital. What cellular
elements will take part in the regeneration of the bone tissue?
A * Osteoblasts
B Osteoclasts
C Osteocytes
D Chondrocytes
E Fibroblasts

The excessive loss of bone tissue mass (it reflects the development of osteoporosis) is observed at
the elderly people. Activation of what bone cells can cause the development of this disease?
32
A * Osteoclast
B Osteoblasts
C Macrophages
D Tissue basophils
E Osteocytes

2005-2011

The bones knitting at the fracture place is resulted by the osteoblasts activation. What is the
function of these cells?
A. *Formation of the osteoid substance and transformation into osteocytes
B. Formation of the connective tissue primary, from which will be bone formation
C. Formation of the cartilage isogenic group
D. Proliferation and calcification
E. Formation of the bone and transformation into osteoclasts

The hydroxyapatite crystals deposition along the collagen fibers takes place during the intercellular
substance of bone calcification. Presence of alkaline phosphatase is necessary for this process in the
intercellular substance. What cell produces this enzyme?
A *Osteoblasts
B Osteocytes
C Osteoclasts
D Chondroblast
E Chondrocytes

CARTILLAGE TISSUE

Tests of the “KROK-1” database

2015
(questions are absent)

2014

The disorder of the motile function resulted by the age changes of the hyaline cartilage was revealed
at a 70-year-old patient. What age changes caused limitation of the joints movement?
A * Deposition of calcium in the intercellular substance
B Increasing the number of isogenic groups
C Increasing the number of cartilage cells
D Thickening of the perichondrium
E Increasing the hydrophilicity of the ground substance

The isogenic cells groups were revealed at the histological slide of the cartilage. What cells are the
primary in the formation of these groups?
A * Chondrocytes I type.
B Chondroblasts
C Prechondroblasts
D Chondrocytes II type
E Chondrocytes III type

2013
33
The tissue was represented in a histological slide. The cells of this tissue are situated singly or by
isogroups and the fibrous structures of the intercellular substance are not visible. What tissue is
represented in the slide?
A * Hyaline cartilage
B Smooth muscle tissue
C Epithelial tissue
D Fibrous cartilage
E Bone tissue

2012

The tissue is visible at the larynx tumor biopsy material. The cells of this tissue are situated singly
or forming the isogenic cells group located in the same plane. The presence of elastic and collagen
fibers is detected histologically. Which structures could develop this tumor?
A * Elastic cartilage
B Hyaline cartilage
C Fibrous cartilage
D Smooth muscle tissue
E Bone tissue

The two slides were proposed to the student. On the 1st one there is the elastic cartilage (stained by
the orcein), and on the 2nd one – the hyaline cartilage (stained by the hematoxylin-eosin). According
to what features can we tell one from the other?
A * Presence of the elastic fibers
B Presence of the cells isogenic groups
C Presence of young cartilage zone
D Presence of the perichondrium
E Presence of amorphous substance

2005-2011

The elongation of the bones is stopped after the puberty. Decrease of what cell proliferation will
stop elongation of the bones?
A. *Chondroblasts
B. Osteocytes
C. Osteoclasts
D. Fibroblasts
E. Neutrophils

During the study of histological preparations of the airways it was found that the supporting
structures of the large and medium bronchi wall are different according to their tissue composition.
What tissue replaced the hyaline cartilage in the fibro-cartilaginous layer of secondary bronchi
(compared to the large bronchi)?
A * Elastic cartilage
B Loose connective
C Hyaline cartilage
D Smooth muscle
E Fibrous cartilage

Synovial fluid normally doesn’t contain collagen. Analysis of synovial fluid of patient suffering
from rheumatism, showed the presence of various types of collagen. Specify the type of erosive
34
tissue if the synovial fluid is determined by collagen type II.
A *Articular surface of the cartilage
B Synovial membrane
C Vascular endothelium
D Myocytes vessels
E Bone epiphysis

CARTILLAGE TISSUE. BONE TISSUE. OSTEOCHONDROGENESIS.

2015
(questions are absent)

2014

The basophilic cells with highly developed organelles with synthetic activity were found at the
histological examination of a tubular bone diaphysis. These cells are situated under the fiber layer.
They take place at the regeneration process of the bone. At what layer of diaphysis are they
situated?
A * Periosteum
B Proper bone
C Osteoid layer
D Layer of external general plate
E Layer of external general plate

2013

The articular cartilage, as it is known, doesn’t have the perichondrium. What kind of growth is
possible at this cartilage during the regeneration process?
A * Interstitial
B Appositional
C By application
D Appositional and interstitial
E It is not growing

2012

In course of indirect histogenesis of tubular bone tissue a plate is formed between epiphyseal and
diaphyseal ossification centers that provide further longitudinal growth of bones. What structure is
it?
A * Epiphysial plate
B Osseous cuff
C Osseous plate
D Osteon
E Layer of interior general plates

The disorder of cartilage tissue regeneration resulted by the injury of the undifferentiated cartilage
cells was observed at the patient with severe upper extremity injury. What cells were injured?
A * Cells of the perichondrium inner layer
B Cells of the perichondrium outer layer
C Isogenic groups cells
D Cells of the young cartilage zone
35
E Cells originating from blood vessels

2005-2011

The cartilage part of the rib was damaged as a result of trauma. At the expense of what layer of
perichondrium the regeneration of the cartilage will be possible?
A * Chondrogenic
B Fibrous
C Elastic
D Collagen
E Connective tissue

MUSCLE TISSUE

Tests of the “KROK-1” database

2015
(questions are absent)

2014

The destruction of the thin myofilaments is observed after the action of the hydrolytic enzymes.
What structures were damaged?
A * Actin myofilaments
B Myosin myofilaments
C Tonofibrils
D Tropocollagen complexes
E Nucleoprotein complexes

The tissue was represented at the histological slide. The structural unit of this tissue is the muscle
fiber, which consists of the myosymplast and satellitocytes and is covered by a basement
membrane. What tissue is characterized by this structure?
A * Skeletal striated muscle tissue
B Smooth muscle tissue
C Cardiac muscle tissue
D Loose connective tissue
E Reticular tissue

2013

Negative environmental factors have caused the dysfunction of myosatellite cells. What function of
the whole muscle fiber is likely to be changed in this case?
A *Regeneration
B Contraction
C Trophism
D Contractile thermogenesis
E Relaxation

2012

36
A microspecimen of the submandibular salivary gland shows some basket-shaped cells
concentrated around the acinus and excretory ducts. These cells surround bases of the serous cells
and are called myoepitheliocytes. These cells relate to the following tissue:
A *Muscular tissue
B Epithelial tissue
C Neural tissue
D Special connective tissue
E Loose fibrous connective tissue

In course of a conditional experiment the development of mesenchyme cells was completely


inhibited. Development of the following muscle tissue will be disturbed:
A *Smooth muscle tissue
B Neural muscle tissue
C Epidermal muscle tissue
D Cardiac muscle tissue
E Skeletal muscle tissue

A big number of the intermediate microfilaments that contain desmin were revealed at the
cytoplasm of the cell which is spindle-shaped and has a rod-like nucleus. What tissue do these cells
belong to?
A * Muscle
B Nervous
C Epithelial
D Connective
E–

The destruction of the thick myofilaments is observed after mechanical injury of striated muscle
fibers. Where is the localization of the pathological changes?
A * In the disk A
B In the disk I
C In the disk A half
D In the A and I disks
E In the disk I half

Patient with injured muscles of the lower extremities was admitted to the traumatology department.
Due to what cells is reparative regeneration of the muscle fibers and restoration of the muscle
function possible?
A *Satellite-cells
B Myoblasts
C Myofibroblasts
D Fibroblasts
E Myoepithelial cells

2005-2011 рр.

It is known, that calcium ions, along with other factors, provide contraction of the muscle tissue.
With what structures does calcium interact during contraction?
A. *Protein troponin of thin fibrils
B. Protein myosin of thick fibrils
C. Protein actin of thin fibrils
D. Actomyosine complex of sarcolemma
E. Protein sequestrin
37
The skeletal muscle fibers are damaged due to injury. What structures can be the source of
reparative regeneration of the skeletal muscle fibers?
A. *Myosatellitocytes
B. Myofibrils
C. Myofilaments
D. Sarcolemma
E. Endomysium

Name the protein that is the main component of the thin myofilaments of the muscle cells.
A. *Actin
B. Tubulin
C. Dynein
D. Desmin
E. Keratin

General increase of the skeletal muscle mass takes place at the sportsman. Hypertrophy of what
kind of the skeletal muscle fibers will be prevailed in this case?
A. *White muscle fiber
A. Red muscle fiber
B. Intermediate muscle fiber
C. Cardiac muscle fiber
D. Smooth myocytes

The non-cellular structure with elongated oval shape and many nuclei under the plasmolemma
appears in histological preparation stained by hematoxylin-eosin. The cytoplasm is transverse
striated. What structure is this?
A *Skeletal muscle
B Collagen fibers
C Myelinated nerve fiber
D Contractive cardiac muscle fiber
E Smooth muscle cells

The labels of the tongue and stomach slides were mixed in the laboratory. Is it possible to define the
slides according to the muscle structure? Name a specific feature of the tongue muscle.
A *Yes. Presence of the symplast.
B No
C Yes, the intercalated discs presence
D Yes, the lack of transverse-striation
E Yes, the central position of the cell nuclei

NERVOUS TISSUE. NEURONS. NEUROGLIA.

Tests of the “KROK-1” database

2015
(Questions are absent)

2014

A sensitive neural ganglion consists of roundish neurons with one extension that divides into axon
38
and dendrite at some distance from the perikaryon. What are these cells called?
A *Pseudounipolar
B Unipolar
C Bipolar
D Multipolar
E Apolar

2013
(Questions are absent)

2012-

The degeneration of the nerve fibers is accompanied by breakage of axial cylinders. Myelin
destruction may develop in case of traumatic injury of the upper limb. What neural structures will
renew the myelin during the regeneration?
A * Neurolemmocytes (Schwann cells)
B Mesaxon
C Perineurium
D Endoneurium
E Astrocytes

2005-2011 рр.

The Nissl bodies lysis is observed in a case of functional exhaustion of the neuron. It is resulted in
decay of basophilic substance and brightening of the neuroplasma. What organelles changes do this
phenomenon is associated with?
A. * Granular endoplasmic reticulum
B. Neurofibrils
C. Lysosomes
D. Golgi apparatus
E. Mitochondria

The patient who had an accident was admitted to the hospital. A tourniquet had been imposed on a
forearm to stop the bleeding for a long time. This resulted in a loss of sensation and movement in
the arm. Examination of the patient revealed a violation of nerve conduction in a mixed nerve
caused by its destruction without displacement of nerve fibers fragments. Proliferation of what cells
will provide the axial cylinders regeneration in mixed nerves?
A. * Schwann's cell
B. Fibrous astrocytes
C. Neurons
D. Ependymal cell
E. Protoplasmic astrocytes

Elderly patient consulted a neurologist with complaints about the difficulty of the neck movements,
numbness and loss of the right hand sensation. Examination of the patient revealed a deformation of
the spine in the lower cervical and upper thoracic part of the spinal cord caused by excessive
deposition of calcium salts. What cells dysfunction influences the loss of sensitivity in the hand?
A. * Pseudounipolar neurons
B. The neurons of the spinal cord anterior horns
C The neurons of the spinal cord posterior horns
D. The neurons of the spinal cord lateral horns
E. Pyramidal neurons
39
The multipolar neurones are observed in the grey matter of the histological specimen of the
cerebellum cross-section. According to what morphological feature does these cells attribute to
multipolar?
A * Amount of the processes
B Length of the processes
C Shape of the terminal axon extension
D Shape of the perikaryon
E Size of the cells

The cells of neuroglia are forming from two sources: ectoderm and mesenchyme. What cells have
the mesenchymal origin?
A * Microgliocytes
B Ependymal cell
C Fibrillar astrocyte
D Protoplasmic astrocyte
E Oligodendrocyte

The spongioblasts were destroyed during the experiment. What nerve tissue violations will occur
during it differentiation?
A *Disorder of neuroglia development
B Disorder of white matter of the spinal cord development
C Disorder of gray matter of the spinal cord development
D Disorder of the spinal ganglion development
E Disorder of the cerebrum development

The channel that is surrounded by a dense layer of cells we can see on the cross-section of the
spinal cord. What are these cells and where they are located in the body?
A * Ependymal cells, all cerebrum ventricles
B Astrocytes, all cerebrum ventricles
C Oligodendrocytes, gray matter of the spinal cord
D Astrocytes, white matter of the spinal cord
E Neuron, myelin sheath

The area of peripheral nerve longer than 10 cm was damaged after a mechanical injury. The motor
activity of the upper extremity is impaired because of it. An allotransplantation of the nerve was
proposed to the patient. What glial cells will take a part in the regeneration of the damaged area?
A. * Neurolemmocytes
B. Fibrous astrocytes
C. Protoplasmic astrocytes
D. Ependymal cells
E. Microgliocytes

The convulsions that associated with incomplete nerve fibers myelination can easily arise at the
children of the first year. What glial cells associated with this situation?
A. * Oligodendrocyte
B. Ependymal cells
C. Fibrous astrocytes
D. Microglia cells
E. Protoplasmic astrocytes

40
NERVOUS TISSUE. NERVE FIBERS. NERVE ENDINGS. SYNAPSES.

Tests of the “KROK-1” database

2015

As a result of a mechanical injury over 10 cm portion of a peripheral nerve was damaged. This
caused an impairment of the upper limb activity. The patient was offered nerve transplantation.
What glial cells will participate in regeneration and provide the trophism of the injured limb?
A. Schwann cells
B. Fibrous cells
C. Protoplasmic cells
D. Microglia
E. Ependymal cells

Several axial cylinders with mesaxons are clearly visible in the electron microscope picture of the
fiber cross-section. What fiber is it?
A. *Unmyelinated nerve fiber
A. Reticular fiber
B. Collagen fiber
C. Elastic fiber
Е. Myelinated nerve fiber

2014
(Tests on the subject available)

2013

The toxic substances action violates the mechanism of nerve impulses transmission in the
experiment. What structure does the implementation of this function provide?
A * Synapse
B Neurolemma
C Neurofibrills
D Mitochondria
E Chromophilic substance

2012
(Tests on the subject available)

2005-2011

The concentrically oriented dark and bright rings, which are located around the axial cylinder, were
revealed in the electron microscope pictures of the myelin fiber cross-section. What substances are
forming the dark rings?
A *Lipids
B Protein
C Vitamins
D Carbohydrates
E Water

The several axial cylinders with mesaxon were revealed in the electron microscope pictures of the
fiber cross-section. What kind of fiber is this?
41
A * Unmyelinated nerve fiber
B Reticular fiber
C Collagen fiber
D Elastic fiber
E Myelinated nerve fiber

The fiber with one of its shell that is black and has a light notches located at an angle to the axial
cylinder was founded in the nervous tissue slide colored by osmic acid. What is a shell?
A *Myelin
B Neurolemma
C Axolemma
D Basement membrane
E Endoneurium

NERVOUS SYSTEM. КОРА ВЕЛИКИХ ПІВКУЛЬ. СТОВБУР МОЗКУ.

Tests of the “KROK-1” database

2015
(questions are absent)
2014

A part of the central nervous system has layerwise allocation of neurocytes, among which there are
cells of such forms: stellate, fusiform, horizontal, pyramidal. What part of the CNS has this
structure?
A. *Cortex of large hemispheres
B. Cerebellum
C. Hypothalamus
D. Medulla oblongata
E. Spinal cord

2013

An organ of nervous system which consists of grey and white substances is presented in a
histological specimen. Grey substance is located on the periphery and consists of 6 layers:
molecular, external granular, external pyramidal, internal granular, internal pyramidal and the layer
of polymorphic cells.
A. *Cerebral cortex
B. Cerebellum
C. Pons cerebelli
D. Spinal ganglion
E. Spinal cord

Precentral gyrus section of the cerebral cortex is presented in the histological specimen. Indicate
which layers mostly developed in this case.
A. *Pyramidal external and internal and layer of polymorphic cells
B. Molecular
C. External and internal granular
D. Molecular and layer of polymorphic cells
E. Molecular, pyramidal external and internal

42
2012

One of sections of central nervous system has layerwise arrangement of neurocytes. There are cells
of the following forms: stellate, fusiform, horizontal, pyramidal among them. What section of
central nervous system is this structure typical for?
A Cortex of cerebrum
B Spinal cord
C Cerebellum
D Medulla oblongata
E Hypothalamus

A specimen, dyed by the method of silver impregnation, is being investigated. Pyramidal cells of
different size are seen in this specimen. Short processes come off their tips and lateral surfaces; one
long process comes off the base of the cells. Name the specimen.
A. *Cerebral cortex
B. Spiral organ of the inner ear
C. Retina of the eye
D. Cortex of the cerebellum
E. Spinal ganglion

НЕРВОВА СИСТЕМА. СПИННИЙ МОЗОК. МОЗОЧОК.

2015
(questions are absent)

2014

As a result of an injury, the integrity of the anterior spinal cord root was broken. Specify the
neurons and their processes that had been damaged:
A *Axons of motor neurons
B Motor neuron dendrites
C Axons of sensory neurons
D Dendrites of sensory neurons
E Dendrites of association neurons

As a result of trauma a patient has damaged anterior roots of the spinal cord. What structures have
been affected?
A *Axons of motoneurons and axons of neurons of lateral horns
B Central processes of sensitive neurons of spinal ganglions
C Peripheral processes of sensitive spinal ganglions
D Axons of neurons of lateral horns
E Dendrites of neurons of spinal ganglions

2013

Alcohol intoxication, as a rule, is accompanied by the coordination of movements’ disorder and


imbalance caused by the damage of cerebellum structural elements. The function of what cells of
cerebellum is affected first of all?
A. *Purkinje’s cells
B. Basket cells
43
C. Betz cells
D. Stellate cells
E. Granule cells

Cerebellar cortex is revealed in a specimen impregnated with silver salts. It includes pyriform,
basket, stellate and granule cells. What neurons constitute molecular layer?
A. *Basket, small and large stellate cells
B. Stellate and pyramidal cells
C. Granule cells and large stellate cells
D. Large stellate cells and spindle cells
E. Pyriform cells

Cerebellar cortex is revealed in a specimen impregnated with silver salts. It includes pyriform,
basket, stellate and granule cells. Which from mentioned above cells is efferent neuron of
cerebellum?
A. *Pyriform cells
B. Stellate cells
C. Granule cells
D. Pyramidal cells
E. Spindle cells

An organ of nervous system, which consists of grey and white substances, is represented in a
histological specimen. Grey substance is located in the center and consists of efferent neurons,
projection neurons and interneurons. Name this organ.
A. *Spinal cord
B. Pons cerebelli
C. Cerebral cortex
D. Cerebellum
E. Spinal ganglion

2012

55-year-old patient has disorder of the movements’ coordination and balance as a result of
permanent using of alcohol and developing intoxication. What nervous structures of the CNS have
been disordered?
A. *Purkinje cells of the cerebellum
B. Basket cells of the cerebellum
C. Stellate cells of the cerebellum
D. Motor neurons of the spinal cord
E. Olive of the medulla oblongata

15-year-old patient was admitted to the clinic with diagnosis poliomyelitis. This disease is
accompanied with disorder of movements. What nervous structure destruction can explain this
disorder?
A. *Motor neurons of the spinal cord
B. Sensory neurons of the spinal ganglions
C. Vegetative nucleuses of the spinal cord
D. Substantia gelatinosa
E. Neurons of the cerebellum

A patient with poliomyelitis (which characterized by spinal cord damage) has disorder of skeletal
muscles function. What neurons destruction can explain this disorder?
44
A. *Motor neurons
B. Pseudounipolar
C. Associative (interneurons)
D. Pseudounipolar and associative (interneurons)
E. Interneurons and motor

An organ of nervous system, which consists of grey and white substances, is presented in a
histological specimen. Grey substance is located on the periphery. Neurons in the grey matter form
three layers molecular, Purkinje and granular. What organ has this structure?
A. *Cerebellum
B. Spinal cord
C. Cerebral cortex
D. Medulla oblongata
E. Pons

A specimen, dyed by the method of silver impregnation, is being investigated. Pyriform cells with
2-3 climbing up prominent dendrites are seen in this specimen. Name the specimen.
A. *Cortex of the cerebellum
B. Spiral organ of the inner ear
C. Retina of the eye
D. Cerebral cortex
E. Spinal ganglion

Grey substance was revealed during microscopic investigation of the CNS. Neurons of it form three
layers molecular, Purkinje and granular. Name neurons which form second layer?
A. *Pyriform cells
B. Small stellate cells
C. Granule cells
D. Large stellate
E. Basket cells

In the microspecimen of the spinal cord nucleus neurons of which form motor ending in the skeletal
muscles have to be analyzed. Indicate this nucleus
A. *Nucleus proprius of the ventral horn
B. Nucleus thoracicus
C. Intermediolateral nucleus
D. Nucleus proprius of the dorsal horn
E. Nucleus proprius of grey substance

As a result of trauma anterior roots of the spinal cord were damaged in a 47-year-old patient. What
neurons processes were damaged?
A. *Axons of motor somatic and vegetative nucleuses neurons
B. Axons of sensory pseudounipolar neurons
C. Dendrites of sensory pseudounipolar neurons
D. Dendrites of motor and axons of lateral horns nucleuses
E. Dendrites and axons of sensory pseudounipolar neurons

2005-2011

The VII-X thoracic vertebrae were damaged as a result of injury in patient. The hematoma which
formed by this injury compresses the dorsal roots of spinal cord. Which pathway will be suffer from
the damaging effects?
45
A. *Tractus cuneatus
B. Tractus spino-cerebellaris dorsalis
C. Tractus corticospinalis lateralis
D. Tractus vestibulospinalis
E. Tractus rubrospinalis

Nerve cells that provide in the human body next function were found during the histological
examination of the spinal cord in the anterior horns:
A. *The transmission of nerve impulses to the striated muscle tissue.
B. The transmission of signals from the thermoreceptors to cells of the optic hill.
C. The transmission of nerve impulses in the prevertebral and paravertebral ganglia of the
sympathetic nervous system.
D. The transmission of nerve impulses to the secretory apparatus of the digestive system.
E. Secretion of the neurotransmitters that affect vascular tone.

There are two specimens: of the spinal cord and spinal node. Which glial cells can be seen only in
specimen of the spinal cord?
A *Ependymal cells
B Protoplasmic astrocytes
C Fibrous astrocytes
D Glial macrophages
E Olygodendrocytes

Small cells with short dendrites which have appearance of a “bird’s foot” are visible in the electron
microscope picture of the cerebellar cortex in the granular layer. The mossy fibers form synapses in
the glomeruli of the cerebellum with dendrites of these cells. What are these cells?
A *Granular cells
B Golgi cells
C Fusiform horizontal cells
D Stellate neurons with short dendrites
E Stellate neurons with long dendrites

Large pear-shaped cells were found in the specimen of the CNS impregnated with silver salts. They
are arranged in a single row, their top leaves 3-4 processes that ramify in one plane. Name this cells.
A *Purkinje cells
B Betz cells
C Dogel cells
D Golgi cells
E Martinoti cells

НЕРВОВА СИСТЕМА. ПЕРИФЕРИЧНА НЕРВОВА СИСТЕМА.

2015
(questions are absent)

46
2014
(questions are absent)

2013

Parenchyma of the organ consists of the nervous tissue in which pseudounipolar neurons are
revealed. Perikaryon of the neurons is covered with glial and connective tissue membranes and
located in bunches. Name this organ.
A. *Spinal ganglia (sensory ganglia)
B. Vegetative ganglion
C. Epiphysis
D. Spinal cord
E. Cerebellum

2012
(questions are absent)
2005-2011

The patient of the neurological department has a loss of sensitivity which associated with disorders
of pseudounipolar neurons. Pseudounipolar neurons are a type of bipolar neurons and localized in
only one place of the human body. Name it.
A *Spinal nodes
B Retina
C Spiral ganglion of the ear
D Intramural autonomic ganglia
E Thalamus

НЕРВОВА СИСТЕМА. ВЕГЕТАТИВНА НЕРВОВА СИСТЕМА. ОБОЛОНКИ МОЗКУ.


ГЕМАТО-ЕНЦЕФАЛІЧНИЙ БАР᾿ЄР.

Tests of the “KROK-1” database

2015
(questions are absent)

2014
(questions are absent)

2013

An organ of nervous system, which consists of grey and white substances, is presented in a
histological specimen. Grey substance is located in the center and forms butterfly. Neurons in the
grey matter locate in bunches and form nucleuses. Which nucleus belongs to the central part of the
vegetative nervous system?
A. *Intermediate lateral nucleus
B. Nucleus proprius of the dorsal horn
C. Nucleus proprius of the ventral horn
D. Nucleus thoracicus
E. Intermediate medial nucleus
47
2012
(questions are absent)

2005-2011

An organ of the nervous system which has star-shaped form is presented in a histological specimen.
This organ is covered with connective tissue capsule and contains multipolar neurons which have
different functions. What organ is this?
A *Autonomic ganglion
B Spinal ganglion
C Spinal cord
D Cerebellum
E Medulla oblongata

The autonomic ganglion which is composed of neurons that are surrounded by a glial sheath is
presented in a histological specimen. What neurons according to the morphological classification
are found in the autonomic ganglia?
A *Multipolar neurons
B Unipolar neurons
C Bipolar neurons
D Pseudounipolar neurons
E-

SENSORY ORGANS. Organ of vision.

Tests of the “KROK-1” database

2015
Neural origin cell is represented in the electron micrograph. Terminal part of the cell dendrite is
cylindrically in shape and consists of 1000 membrane enclosed discs. What cell is shown in this
microphotograph?
A.Rod cell
B. Cone cell
C. Spinal ganglion
D.Neuron of the spinal cord
E. Neuron of the cerebral cortex

2014

An infectious disease caused contractive activity of muscles that contract and dilate eye pupil
(paralytic state). What functional eye system was damaged?
A *Accommodative
B Dioptric
C Ancillary
D Photosensory
E Lacrimal apparatus

48
A 14-year-old patient has twilight vision impairment. What vitamin deficiency takes place in the
organism?
A. *A
B. B 1
C. В 6
D. C
E. В 12

During the examination the doctor found that the patient does not distinguish blue and green colors
with normal perception of other colors. What structure is disordered?
A *Cones receptor cells
B Rod receptor cells
C Bipolar neurons
D Amacrine neurons
E Horizontal neurons

2013

Deficiency of the vitamin A results in the impairment of twilight vision. Name the cells that have
the above-mentioned photoreceptor function:
A *Rod receptor cell
B Horizontal neurons
C Cone receptor cells
D Bipolar neurons
E Ganglion neurons

An ophthalmologist has detected that patient has disorder of green color perception during
examination. Which retina cells absence may cause this sight disorder?
A. *Cone receptor cells
B. Rod receptor cells
C. Epithelial pigmented cells
D. Bipolar neuron
E. Ganglionic nerve cell

An eyeball structure without blood vessels was detected in a histological specimen. What structure
is characterized by this morphological sign?
A. *Cornea
B. Ciliary body
C. Choroid
D. Iris
E. Retina

Cells peripheral part of which consists of two segments were revealed in the electron
microphotograph of the sense organ. The outer segment has membrane half discs and the inner one
has ellipsoid. In what organ does this structure locate?
A. *In the organ of vision
B. In the organ of taste
C. In the organ of smell
D. In the vestibular organ
E. In the auditory organ
49
The increased intraocular tension is observed in the patient with glaucoma. Secretion of aqueous
humor by the ciliary body is normal. Injury of what structure of the eyeball wall does the disorder
of outflow from the anterior chamber cause?
A *Venous sinus
B Ciliary body
C Choroid
D Ciliary muscle
E Back epithelium of cornea

2012

Damages of vascular membrane are detected in the histological specimen of a fetus eyeball. What
embryonic material was probably damaged during the development of the eye?
A. *Mesenchyme
B. Ectoderm
C. Endoderm
D. Outer layer of the eyeball
E. Internal layer of the eyeball

In the electron microphotograph revealed cell of neural origin. Terminal part of the cell dendrite has
cylindrical shape and consists of 1000 enclosed membrane discs. What cell is this?
A. *Rod cell
B. Ventral horn of the spinal cord
C. Sensory ganglia neuron
D. Cerebral cortex neuron
E. Cone cell

A lot of people with age have clouding of the lens (phacoscotasmus or cataract). It became opaque
that leading to the partial blindness. What optical and chemical properties of the lens fibers protein
will be disordered?
A. *Crystalline
B. Vitrein
C. Dinein
D. Rhodopsin
E. Iodopsin

In the histological specimen of the eyeball wall, structure which consists of three neurons chain is
revealed. Bodies of these neurons form outer, inner nuclear and ganglion cell layers. Which eye
component has this morphologic structure?
A. *Retina
B. Iris
C. Sclera
D. Choroid
E. Ciliary body

Underdeveloped epithelium of cornea is observed in the histological specimen of a fetus’ eyeball. A


part of what embryonal layer was probably affected in the process of embryogenesis?
A. *Ectoderm
B. Endoderm
C. Mesoderm
D. Outer layer of the eyeball
50
E. Inner layer of the eyeball

Patient with eye trauma consulted a doctor. During examination of corneal epithelium was revealed
changes. What epithelium was damaged?
A. *Stratified squamous non keratinized
B. Simple pseudostratified
C. Stratified squamous keratinized
D. Stratified cuboidal
E. Stratified cylindrical

Transplantation of the cornea was done to the patient. Which peculiarities of the cornea structure
give expectation to engraftment of it?
A. *Absence of blood and typical lymphatic vessels
B. Presence of stratified anterior epithelium
C. A huge innervations
D. Presence of connective tissue
E. Presence of simple squamous epithelium

Patient has appealed to the ophthalmologist with complaints about hurt in his eyes, which revealed
itself after long presence of the patient in the field during dust storm. Doctor diagnosed superficial
injuries of the external corneal epithelium. Which cells provide regeneration of the injured
epithelium?
A. *Basal cells
B. Cells of stratum corneum
C. Cells of stratum granulosum
D. Cells of stratum spinosum
E. Cells of stratum lucidum

Ciliary body was damaged in the patient. Function of what eye apparatus will suffer in this case?
A. *Accommodative
B. Protective
C. Trophic
D. Photosensitive
E. Dioptric

An infectious disease caused contractive activity of muscles that contract and dilate eye pupil
(paralytic state). What functional eye system was damaged?
A *Accommodative
B Dioptric
C Ancillary
D Photosensory
E Lacrimal apparatus

55-year-old man went to the doctor with complains about the significant increase in the distance
that he has the ability to read without glasses. What part of the eye has undergone age-related
changes that have led to this condition?
A. *Lens
B. Cornea
C. Sclera
D. Iris
E. Ciliary body

51
Nutrition of the cornea is carried out:
A. *By diffusion from the fluid of the anterior chamber of the eye
B. From its own blood vessels
C. By diffusion from the fluid of the posterior chamber of the eye
D. From the lymphatic vessels
E. Due to the secret of the lacrimal glands

As a result of petechial hemorrhages to the retina, the patient lost the ability to see objects in the
center of the view field. Where was bleeding in the retina?
A. *Macula lutea
B. Ciliary part of the retina
C. Blind spot
D. Choroid
E. -

A cell of the retina in which one of the processes has a segment with hemidiscs is visible in the
electron microscope picture. What pigment is contained in the structure of the membrane of these
hemidiscs?
A *Iodopsin
B Rhodopsin
C Melanin
D Hemosiderin
E Lipofuscin

The patient attended the ophthalmologist with nyctalopia (night blindness). The doctor examined
the retina and found degenerative changes in the photosensory layer of the rods of the retina. What
vitamin will be prescribed by a doctor?
A *Vitamin А
B Vitamin С
C Vitamin D
D Vitamin В1
E Vitamin В12

Cells of the retina which have outer and inner segments in the dendrites are presented in the
electron microscope picture. The outer segment contains hemidisc and inner segment contains lipid
droplet surrounded by mitochondria. What do these cells?
A *Provide daylight and color vision
B Provide black and white vision
C Refract the light
D It is the source of regeneration of the photoreceptor neurons
E Form the efferent way – optic nerve

ОРГАНИ ЧУТТЯ. ОРГАН СЛУХУ ТА РІВНОВАГИ.

2015
(questions are absent)
52
2014
(questions are absent)

2013
(questions are absent)

In an electronic micrograph of a sense organ hair cells are seen, on their apical part there are short
microvilli – stereocilii and a polar located kinocilium. What sensory organ are such cells typical
for?
A. *Vestibular organ
B. Vision organ
C. Olfactory organ
D. Hearing organ
E. Taste organ

A histological specimen represents a receptor zone of a sensoepithelial sense organ. Cells of this
zone are placed upon the basal membrane and include the following types: external and internal
receptor cells, external and internal phalangeal cell, stem cells, external limiting cells and external
supporting cell. The described receptor zone belongs to the following sense organ:
A *Acoustic organ
B Visual organ
C Gustatory organ
D Equilibrium organ
E Olfactory organ

2012

Cristae ampullaris have been damaged in a 32-year-old patient as a result of head trauma. What
stimuli perception has been disordered?
A. *Angular movement
B. Vibration
C. Gravitation
D. Linear movement
E. Vibration and gravitation

A patient had taken high doses of streptomycin and consequently became deaf. The function of
what cells of the inner ear was damaged in this case?
A. *Hair
B. Phalangeal
C. Pillar
D. Deiters’
E. Connective tissue

2005-2011

The patient lost the ability to hear loud bass sounds after suffering a viral illness. Which cells of the
spiral organ are injured?
A. *Inner hair cells of the upper parts of the cochlear duct
B. Phalangeal cells
C. Outer hair cells of the lower parts of the cochlear duct
D. Inner hair cells of the lower parts of the cochlear duct
53
E. Pillar cells

The supporting cells, which are arranged in 3-4 rows, rest on the basement membrane close to the
external columnar cells are visible in the electron microscope picture of the organ of Corti. Bowl-
shaped recess presents in the upper third of these cells, which includes the basic of external sensory
cells. What cells are these?
A *Outer phalangeal cells
B Outer border cells
C Outer supporting cells
D Inner phalangeal cells
E Inner pillar cells

The perception of the vibration oscillation was impaired in a patient with meningitis. What cells of
the organ of hearing and balance are affected?
A *Hair sensory cells of the macula of spherical sacculi
B Hair sensory cells of the macula of elliptical saculi
C Supporting cells of the macula of spherical sacculi
D Supporting cells of the macula of elliptical sacculi
E Hair sensory cells of the cristae ampullaris

ОРГАНИ ЧУТТЯ. ОРГАНИ НЮХУ, СМАКУ ТА ДОТИКУ.

2015
(questions are absent)

2014
(questions are absent)

2013
(questions are absent)

2012

A 30-year-old man has damaged mucosal membrane which covers upper part of upper conchae as a
result of trauma. What consequences did this lead to?
A. *Disorder of smell substances perception
B. Disorder of air moistening
C. Disorder of secretory activity of goblet cells
D. Disorder of air warming
E. Disorder of air moistening and warming

A boxer has disturbance in smell perception after a trauma of the nose. What cells damage may
cause the loss of smell?
A. *Neurosensory
B. Supporting epithelial cells
C. Basal epithelial cells
D. Microvillous epithelial cells
E. Brush cells

54
On the electron microphotograph neural origin cell is presented. This cell constitutes part of mucosa
membrane epithelium. Distal part of peripheral process of the cell has knob-like expansion from
which arise 10-15 cilia. What cell is it?
A. *Olfactory cell
B. Bipolar neuron of spinal ganglion
C. Sensor epithelial cell of organ of taste
D. Rode cell
E. Cone cell

2005-2011

There are cells that perform a protective function in the epidermis. They are similar in morphology
to the dendritic cells of the lymph nodes, spleen, and thymus and have a monocytic origin. What are
these cells?
A. *Langerhans cells
B. Melanocytes
C. Keratinocytes of the basal layer
D. Keratinocytes of the spinosum layer
E. Keratinocytes of the granular layer

Five-month-old baby was admitted to the pediatric clinic with the changes of the skeletal system
such as: softening of some flat bones of the skull, enlargement of the fontanels, characteristic
swelling of ribs. The baby nutrition was normal but he rarely walked. The baby was diagnosed with
rachitis. What function of the skin was violated in this case?
A. *Synthesis of vitamin D
B. Water-salt metabolism
C. Heat exchange and thermoregulation
D. Deposition of blood
E. Protection from environmental influences

One of the skin diseases is accompanied by the separation of the epidermis from the basement
membrane and the formation of the bubble as a result of autoimmune reaction. Which structure of
the epidermis is damaged in this reaction?
A *Hemidesmosomes
B Desmosomes
C Zonulae occludentes
D Gap junction
E Synapse

There are cells that form so-called Mongolian spot in people of the Mongoloid race in the dermis of
coccygeal area of the skin. It is known that these cells originate from cells of the neural crest and
synthesize melanin pigment. What are these cells?
A *Pigment cells
B Macrophages
C Fibroblasts
D Lipocytes
E Adventitial cells

Pemphigoid was diagnosed in the patient with damage of certain areas of the skin in the form of
bubbles filled with fluid. This is an autoimmune disease which is accompanied with destruction of
the basement membrane of the epidermis that causes its separation from the connective tissue.

55
Which glycoproteins of the basement membrane perform the adhesive role under the normal
condition?
A *Laminin and fibronectin
B Globin
C Actinin and vinculin
D Vimentin
E Tubulin

The palms and soles are covered with thick skin and other parts of the body covered with a thin
skin. Which layer is missing in the thin skin?
A. *Stratum lucidum
B. Stratum basale
C. Stratum spinosum
D. Stratum granulosum
E. Stratum corneum

SKIN MORPHOLOGY

2015
The method of dactylography is used extensively in the forensic medical expertise. This method is
based on the strictly individual picture of the skin surface made by the papillary layer of the derma.
What tissue is forming this layer of the derma?
A * Loose connective tissue
B Dense irregular fibrous tissue
C Dense regular fibrous tissue
D Reticular tissue
E Adipose tissue

A 12-year-old patient has white non-pigmented spots on the skin. The spots appeared after the
patient became 10-year-old, and they constantly grow. This spots appeared due to the lack of the
following skin cells:
A. *Melanocytes
B. Adipocytes
C. Fibrocytes
D. Plasmocytes
E. Labrocytes

2014

Cells with processes and dark brown granules in the cytoplasm were revealed in the skin epidermis
of the biopsy material. What are these cells?
A. *Melanocytes
B. Intraepidermal macrophages
C. Keratinocytes
D. Merkel cells
E. Lymphocytes

Trauma of skin reticular layer was happened. At the expense of what cells differon activity
regeneration of this layer will happen?
56
A. *Fibroblastic
B. Macrophagic
C. Lymphoblastic
D. Neuroblastic
E. -

Under the radiation influence epidermal cells of the stratum basale were damaged. What function of
the epidermis will be weakening or depressed first of all?
A. *Regenerative
B. Protective
C. Absorptive
D. Barrier
E. Dielectric

With age human skin undergoes changes, which may declare themselves by reduction of skin
elasticity. What structures of the connective tissue provide skin elasticity most of all?
A. *Collagen and elastic fibers
B. Ground substance
C. Epidermis cells
D. Connective tissue cells
E. Reticular fiber

In the course of experiment on a frog embryo the external embryonic layer – ectoderm – has been
destroyed. Which of the following morphological structures has not been developed henceforth?
A. *Epidermis
B. Somites
C. Nephrotome
D. Splanchnotome
E. Sclerotome

2013

One of the surgery rules is performing sections along the so-called Langers’ lines (lines of skin
tension). Which from mentioned below tissues form papillary layer (the strongest layer in the
derma)?
A. *Dense irregular connective tissue
B. Reticular connective tissue
C. Loose connective tissue
D. Epithelial tissue
E. Dense regular connective tissue

In an electron micrograph of skin epidermis among the cells of cuboidal form dendritic cells are
detected. In their cytoplasm Golgi apparatus is well-developed; there are a lot of ribosomes and
melanosomes. Name these cells.
A. *Melanocytes
B. Keratinocytes
C. Cells of Langerhans
D. Merkel’s cells
E. Mast cells

57
Study of fingerprints (dactylography) is used by criminalists for personal identification as well as
for diagnostics of genetic abnormalities, particularly Dawn’s disease. What layer of skin determines
individuality of fingerprints?
A *Dermopapillary
B Horny
C Reticular
D Clear (stratum lucidum)
E Basal

2012

A child has abraded skin of the palm when falling down. What epithelium was damaged?
A. *Stratified squamous keratinized
B. Stratified squamous non-keratinized
C. Simple low columnar
D. Transitional
E. Simple squamous

Method of dactyloscopy is largely used in forensic medicine. This method is based on strictly
individual print of the skin surface which determines papillary layer of derma. Which tissue forms
this layer of the derma?
A. *Loose irregular connective tissue
B. Reticular connective tissue
C. Dense irregular connective tissue
D. Adipose tissue
E. Dense regular connective tissue

There is histological specimen of the skin epidermis biopsy sample taken from the healthy adult.
Cells division can be seen in the stratum basale. What process does provide these cells?
A. *Physiologic regeneration
B. Differentiation
C. Adaptation
D. Reparation
E. Apoptosis

Some layers are absent on a limited area of epidermis after a trauma. Only germinative layer is
preserved. Name the cells, which will become the main source of its regeneration.
A. *Layer of basal cells
B. Layer of spinosum cells
C. Layer of granulosum cells
D. Layer of spinous and granular cells of undisturbed area
E. Cells of lucidum layer of undisturbed area

The central part of epiblast cells (ectoderm) sags and neurulation process begins during the third
week of embryogenesis. In which direction will the remaining ectodermal cells differentiate?
A. *Skin
B. Somites
C. Gut
D. Chord
E. Yolk sac

58
Stratified structure organ which covered with stratified squamous keratinized epithelium is
represented on the histological specimen. There is loose connective tissue which forms convexity in
the form of papilla located under epithelial basal membrane. Dense irregular connective tissue
located under it and form reticular layer. What organ has these morphological peculiarities?
A. *Skin
B. Tongue
C. Esophagus
D. Tonsil
E. Cervix of the uterus

There are next layers in the skin histological specimen: stratum basale, spinosum, granulosum,
lucidum and corneum. Which part of the human body does this epithelium belong to?
A. *Palm skin
B. Face skin
C. Hairy part of the head skin
D. Shoulder skin
E. Thigh skin

Skin malignant tumor was revealed at 30-years old patient. What epidermis cells take place in the
immune response?
A. *T-lymphocytes
B. Keratinocytes
C. Keratinocytes and Merkel cells
D. Merkel cells
E. Stratum spinosum cells

Trauma of derma reticular layer was happened. At the expense of what cell differon activity
regeneration of this layer will happen?
A. *Fibroblasts
B. Macrophages
C. Lymphoblasts
D. Mast cells
E. Plasma cells

Process of the dorsal mesoderm segmentation and somites formation was disordered in the embryo.
In what part of the skin is disorder development possible?
A. *Derma
B. Hair
C. Sebaceous glands
D. Epidermis
E. Sweat glands

2005-2011

There are cells that perform a protective function in the epidermis. They are similar in morphology
to the dendritic cells of the lymph nodes, spleen, thymus and have a monocytic origin. What are
these cells?
F. *Langerhans cells
G. Melanocytes
H. Keratinocytes of the basal layer
I. Keratinocytes of the spinosum layer
J. Keratinocytes of the granular layer
59
Five-month-old baby was admitted to the pediatric clinic with the changes of the skeletal system
such as: softening of some flat bones of the skull, enlargement of the fontanels, characteristic
swelling of ribs. The baby nutrition was normal but he rarely walked. The baby was diagnosed with
rachitis. What function of the skin was violated in this case?
F. *Synthesis of vitamin D
G. Water-salt metabolism
H. Heat exchange and thermoregulation
I. Deposition of blood
J. Protection from environmental influences

One of the skin diseases is accompanied by the separation of the epidermis from the basement
membrane and the formation of the bubble as a result of autoimmune reaction. Which structure of
the epidermis is damaged in this reaction?
A *Hemidesmosomes
B Desmosomes
C Zonulae occludentes
D Gap junction
E Synapse

There are cells that form so-called Mongolian spot in people of the Mongoloid race in the dermis of
coccygeal area of the skin. It is known that these cells originate from cells of the neural crest and
synthesize melanin pigment. What are these cells?
A *Pigment cells
B Macrophages
C Fibroblasts
D Lipocytes
E Adventitial cells

Pemphigoid was diagnosed in the patient with damage of certain areas of the skin in the form of
bubbles filled with fluid. This is an autoimmune disease which is accompanied with destruction of
the basement membrane of the epidermis that causes its separation from the connective tissue.
Which glycoproteins of the basement membrane perform the adhesive role under the normal
condition?
A *Laminin and fibronectin
B Globin
C Actinin and vinculin
D Vimentin
E Tubulin

The palms and soles are covered with thick skin and other parts of the body covered with a thin
skin. Which layer is missing in the thin skin?
F. *Stratum lucidum
G. Stratum basale
H. Stratum spinosum
I. Stratum granulosum
J. Stratum corneum
ШКІРА. ПОХІДНІ ШКІРИ.

2014
(questions are absent)

60
2013
(questions are absent)

2012

A patient complains about dryness of head skin, itching, fragility and loss of hair. He was diagnosed
with seborrhea after examination. Disturbed activity of which cells caused this condition?
A *Cells of sebaceous glands
B Cells of sudoriferous glands
C Epithelial cells
D Adipocytes
E Melanocytes

2005-2011

A man was admitted to the traumatological department with burns of the scalp. All epidermal cells
of all layers down to the basement membrane died as a result of it. What structures will provide the
epithelization of the affected areas?
A *Cells of the hair follicles
B Cells of the papillary layer of dermis
C Cells of the reticular layer of dermis
D Cells of the hair bags
E Cells of the hair papilla

The hair shaft of long hair has been destroyed as a result of injury of the victim. What structures
will the renewal of hair provide?
A *Hair bulb
B Dermal papilla
C External root sheath
D Internal root sheath
E Cuticle of the hair

A part of the nail plate was removed in patient as a result of trauma. What structures will provide
the renewal of it?
A *Nail matrix
B Nail cushion
C Eponychium
D Nail fold (groove)
E Nail root

CARDIOVASCULAR SYSTEM. ARTERIES. MICROCIRCULATION.

Tests of the “KROK-1” database

2015

61
Tunica media consists of 1-2 layers of smooth muscle cells which have spiral orientation in the slide
of the vessel of microvascular bed. Tunica adventitia is a thin, sheath of loose connective tissue.
What vessel is it?
A. *Arteriole
B. Venule
C. Capillary
D. Postcapillary
E. Arterio-venous shunt

2014

A histological specimen represents an artery. One of the membranes of its wall has flat cells lying
on the basal membrane. What type of cells is it?
A *Endothelium
B Mesothelium
C Smooth myocytes
D Fibroblasts
E Macrophages

Large arteries during systole stretch out and return in previous condition during diastole providing
stability of bloodstream. What elements of vessel wall will explain this?
A. *Elastic fibers
B. Muscle fibers
C. Reticular fibers
D. Collagen fibers
E. Large amount of fibroblasts

Wall of the vessels often have huge morphological difference in the structure of the tunica media.
What is the reason of appearance such specific peculiarities in the structure of this tunica in
different vessels?
A. *Hemodynamic condition
B. Influence of endocrine system organs
C. Regulation from central nervous system
D. Inductive influence of vegetative ganglion neurons
E. Large content of catecholamines in the blood

A histological specimen shows a blood vessel. Its inner coat is composed by endothelium,
subendothelium and internal elastic membrane. The middle coat is enriched with smooth myocytes.
Such morphological characteristics are typical for the following vessel:
A *Muscular-type artery
B Elastic-type artery
C Capillary
D Non-muscular vein
E Muscular-type vein

2013

Fenestrae between endothelial cells and partial or total absence of basal lamina underlying the
endothelium are revealed in the electron micrograph of a capillary. What type of capillary is it?
A. *Sinusoidal
B. Somatic
C. Visceral
62
D. Atypical
E. Shunt

Multiple capillaries with penetrated through their walls mature blood cells were revealed in the
microspecimen of the red bone marrow. What type of capillaries is it?
A *Sinusoidal
B Fenestrated
C Somatic
D Visceral
E Lymphatic

2012-

The artery is presented in the histological specimen. Squamous cells that lying on the basement
membrane are defined in one of the layer of its wall. Name this type of cells.
A *Endothelium
B Mesothelium
C Smooth muscle cells
D Fibroblasts
E Macrophages

About 50 thick membranes have been detected in a histological specimen of tubular organ dyed
with orcein. They have wiggly appearance and formed tunica media of this organ. Name this organ.
A. *Aorta
B. Muscular artery
C. Esophagus
D. Trachea
E. Heart wall

Vessels which have thick layer of smooth muscle cells in the tunica media were revealed during
investigation of skin bioptat in the derma. What is the name of these vessels?
A. *Muscular artery
B. Capillaries
C. Arterioles
D. Venules
E. Arterio-venous shunts

In the vessel slide one found prominent internal and external elastic membranes and a lot of smooth
muscle cells in the tunica media. What type of vessel is it?
A. *Muscular artery
B. Small artery (mixed)
C. Large vein (muscular)
D. Elastic artery
E. Extraorganic lymphatic system

Fragment cells which rests on the basal membrane and connected with each other by means of
desmosomes and zonulae occludentes revealed in an electron micrograph of a tunica intima. Name
these cells.
A. *Endothelium
B. Mesothelium
C. Epidermis
63
D. Epithelioreticular cells
E. Macrophages

Tunica intima of a vessel is lined with epithelium from within. What epithelium is this?
A. *Endothelium
B. Mesothelium
C. Epidermis
D. Transitional epithelium
E. Pseudostratified epithelium

2005-2011
Wall blood clot was revealed in the blood vessel in the patient during an X-ray isotope scanning.
Damaging of what epithelium leads to the clotting formation?
A. *Endothelium
B. Mesothelium
C. Simple cuboidal epithelium
D. Simple columnar epithelium
E. Stratified squamous epithelium

The 45-year-old patient had a heart attack, which was accompanied by intensive squeezing
retrosternal pain that lasted more than 30 minutes, radiated to the lower jaw, left arm under the left
shoulder scapula and did not pass after taking nitroglycerin during physical activity. Subendocardial
myocardial infarction was diagnosed in the clinic. Which structures have undergone specific
changes in the first place in this case?
A. *Endothelium
B. Fibroblasts
C. Myocytes
D. Fibrocytes
E. Macrophages

The endothelial cell is presented in the electron micrographs. There are pores that are bridged by a
diaphragm in the cytoplasm of this cell. Which vessel has such endothelial cell?
A. *Fenestrated capillaries
B. Somatic capillaries
C. Sinusoidal capillaries
D. Artery
E. Vein

Preparation of the aorta was investigated in the corpse of 74-year-old man. What age-related
changes can be detected in the aortic wall?
A. *The growth of the collagen fibers in the tunica intima and tunica media
B. The growth of elastic fibers in the tunica media
C. Proliferation of smooth muscle cells in the tunica media
D. Hypertrophy of smooth muscle cells in the tunica media
E. Proliferation of endothelial cells

A piece of the soft tissue of the thigh was investigated in the experiment of the animal in two weeks
after incision through the skin and muscles. Newly formed blood vessels were revealed. Which
structures of the vascular wall provide the formation of capillaries in the area of injury?
A. *Proliferation of endothelial cells
B. Proliferation of pericytes
C. Proliferation of adventitial cells
64
D. Hypertrophy of fibroblasts
E. Proliferation of smooth muscle cells

Which vessels have the highest total surface area that creates optimal conditions for bilateral
exchange of substances between tissues and blood?
A. *Capillaries
B. Arteries
C. Veins
D. Arterioles
E. Venules

The patient has aortic aneurysm. Which of the following structures of the aortic wall will be
damaged?
A. *Elastic membranes
B. Endothelium
C. Subendothelial layer
D. Smooth muscle cells
E. Tunica adventitia

Short capillary-type vessels that connected arterioles and venules are visible in a histological
specimen. What is the name of these anastomoses?
A. *Hemishunts
B. Genuine simple shunt
C. Genuine anastomoses secured the contractile structures
D. Genuine simple epithelioid-type
E. Genuine complex epithelioid-type anastomoses

Ultrasound examination of the patient revealed the damage of the wall of arteries of elastic type.
Which function of the vessels may be violated?
A *Transportation of blood
B Participation in the formation of tissue fluid
C Regulation of blood supply to organs
D Ensuring nutrition of organs and tissues
E Drainage function

There is a capillary in electron micrographs of the organ. There are numerous thinning of the
cytoplasm in the endothelium of this capillary. The basement membrane is continuous. Which
organ has this capillary?
A *Hypophysis
B Cerebral cortex
C Liver
D Spleen
E Skeletal muscle

The histological specimen of the vessel with dominating elastic fibers which form fenestrated
membranes in the middle layer was presented for medical examination. Obliquely oriented
myocytes are well expressed between the elastic membranes. Name the vessel from which it was
prepared the histological specimen.
A. *Pulmonary artery
B. Renal artery
C. Capillary
D. Femoral vein
65
E. Vena cava inferior

One of the malformations of the cardiovascular system - cleft arterial duct (patent ductus arteriosus)
was revealed in a one-year-old child. Name the place of its localization.
A. *The vessel connecting the pulmonary artery and aorta
B. Interatrial septum
C. Interventricular septum
D. The vessel connecting the umbilical cord and the vena cava inferior
E. The vessel connecting the portal vein and vena cava inferior

One can see that the structure of the wall of arteries significantly differs from the structure of the
wall of veins during the study of histological specimen of blood vessels. What cause these
differences?
A. *Different hemodynamic conditions
B. The different in diameter
C. The different in length
D. Different functions
E. Different distance from the heart

What type of capillaries is characteristic for the microvascular bed of the hematopoietic organs?
A. *Discontinuous capillaries
B. Fenestrated capillaries
C. Somatic capillaries
D. Continuous capillaries
E. Lacunes

The inner surface of the blood vessels is covered with an epithelium that synthesizes substances that
prevent the blood clotting in vessels. What kind of epithelium is this?
A *Simple squamous epithelium (endothelium)
B Stratified squamous nonkeratinized epithelium
C Stratified squamous keratinized epithelium
D Simple squamous epithelium (mesothelium)
E Simple pseudostratified columnar epithelium

The contraction of smooth muscle cells of arterioles occurs upon admission into the blood of
epinephrine from the adrenal medulla. What features of the structure have these cells?
A * Presence of perforations in the basement membrane and inner elastic membrane
B A single location of smooth muscle cells
C Presence of effector endings on pericytes
D Presence of contacts between endothelium and pericytes
E Presence of narrowing in the place of origin of capillary from arterioles due to the circularly
arranged smooth muscle cells

During the systole aorta is stretched out and returns to its original state during diastole, thus
providing stability to the blood stream. What structural elements of the vessel wall provide it?
A *Elastic membranes
B Muscle fibers
C Reticular fibers
D Collagen fibers
E A large number of fibroblasts

A large number of elastic fibers in all layers and fenestrated elastic membranes in the tunica media
66
are detected in the wall of the blood vessel. What factors contribute to these structural features of
blood vessels?
A *High blood pressure
B Low blood pressure
C High speed of blood flows
D Low speed of blood flows
E Osmotic pressure

I.M. Sechenov called arterioles “taps” of the cardiovascular system. What structural elements
provide this function of arterioles?
A *Circular oriented muscle cells
B Longitudinal oriented muscle cells
C Elastic fibers
D Longitudinal oriented muscle fibers
E Collagen fibers

Cells with uneven wavy edges were revealed as a result of impregnation with silver salts of tunica
intima of blood vessel. Name these cells.
A *Endothelial cells
B Stellate cells
C Myocytes
D Fibroblasts
E Adipocytes

The venom of spiders and snakes that contained hyaluronidase easily penetrates through the wall of
capillaries. What structural component is associated with permeability of capillary walls?
A *Basement membrane
B Fenestrae
C A layer of glycoproteins which covered endothelial cells
D Pericytes
E Adventitial cells

The patient with hypertension has increase in pressure up to 180/100 mm Hg. What receptors in the
wall of the vessel are embraced this change?
A *Mechanoreceptors
B Osmoreceptors.
C Chemoreceptors.
D Thermoreceptors.
E Photoreceptors.

Arterioles play an important role in the blood supply of the organs functional units. Which of these
structures perform this function?
A *Myocytes
B External elastic membrane
C Internal elastic membrane
D Special connective tissue cells
E Endothelial

From 40 to 60 fenestrated elastic membranes were found at the histological preparations stained by
the orcein. Name this vessel.
A *Artery of elastic type
B Artery of muscular type
67
C Artery of mixed type
D Vein of muscular type
E Vein of unmuscular type

The contraction of the smooth muscle cells of arterioles occurs after the adrenalin releasing from
the adrenal medulla into the blood. What are the features of these vessels structure?
A * Availability of perforations in the endothelium basement membrane and the internal elastic
membrane
B Single position of the smooth muscle cells
C Presence of the effector endings on the pericytes
D Presence of the contacts between the pericytes
E Availability of precapillary sphincters

Obliterating atherosclerosis causes changes in the vessels of the lower extremities. A histological
specimen of such vessel evidently presents both internal and external elastic membranes; middle
membrane contains a lot of smooth muscle cells. What vessel is affected in case of this disease?
A *Artery of muscular type
B Artery of elastic type
C Artery of mixed type
D Vein with strongly developed muscles
E Lymph node

Intralobular capillaries of a liver specimen have wide irregular lumen. Basal membrane is absent in
the major part of the capillary. What type of capillaries is it?
A *Sinusoidal
B Visceral
C Somatic
D Precapillaries
E Postcapillaries

A histological specimen shows a blood vessel. Its inner coat is composed by endothelium,
subendothelium and internal elastic membrane. The middle coat is enriched with smooth myocytes.
Such morphological characteristics are typical for the following vessel:
A *Muscular-type artery
B Elastic-type artery
C Capillary
D Non-muscular vein
E Muscular-type vein

CARDIOVASCULAR SYSTEM. VEINS. LYMPHATIC VASSELS.

Tests of the “KROK-1” database


2015

An organ, the wall of which is formed by three tunics is determined in a histological specimen. The
tunica intima consists of endothelium and a thin subendothelial layer. Tunica externa is the thickest.
Which organ is represented in a histological specimen?
A. Vein
B. Artery
C. Urethra
D. Heart
68
E. Uterus

2014

Blind ended vessels which have shape of oblate tubes are present in the histological slide. They
don’t have basal membrane and pericytes. The outer surface of the endothelium is attached to the
surrounding connective tissue by anchoring fibers. What type of vessel is it?
A. *Lymph capillary
B. Blood capillary
C. Arterioles
D. Venules
E. Arterio-venous shunts

A histological specimen of spleen shows a vessel with a wall consisting of endothelium and
subendothelial layer, tunica media is absent, and tunica adventitia attached to the layers of spleen
connective tissue. What vessel is it?
A *Vein of non-muscular type
B Vein of muscular type
C Artery of muscular type
D Arteriole
E Capillary

Histological specimen presents a vessel the wall of which consists of endothelium, basal membrane
and loose connective tissue. What type of vessel is it?
A *Vein of non-muscular type
B Artery
C Vein of muscular type
D Hemocapillary
E Lymphocapillary

2013

A specimen of pia mater includes a vessel whose wall doesn’t have the tunica media, the tunica
externa is adherent to the surrounding tissues, the intima is composed of a basement membrane and
endothelium. What vessel is it?
A *Nonmuscular vein
B Muscular vein with underdeveloped muscular elements
C Muscular artery
D Arteriole
E Artery of mixed type

A specimen of the pia mater shows a vessel without tunica media in its wall, its tunica adventitia
adheres to the surrounding tissues, and the tunica intima is made up of the basal membrane and
endothelium. Specify this vessel:
A *Fibrous vein
B Muscular vein with weakly developed muscular elements
C Muscular artery
D Arteriola
E Mixed artery

2012
69
Morphological examination revealed in histological specimen of biopsy material an irregular-
shaped vessel. Its middle layer is formed by bundles of smooth myocytes and layers of connective
tissue. What type of vessel is it?
A *Vein of muscular type
B Artery of muscular type
C Lymphatic vessel
D Venule
E Arteriole

2005-2011

The medical examiner found the venous sinuses that are adherent to the surrounding tissues during
the study of meninges of the brain. What type of veins is this?
A *Fibrous type of vein
B Veins with weak development of muscle elements
C Veins with middle development of muscle elements
D Veins with high development of muscle elements
E Venules

Thrombophlebitis of deep veins of the shin was revealed in a 63-year-old man. Which layer of these
vessels is damaged?
A * Endothelial layer
B Subendothelial layer
C Layer of elastic fibers
D Layer of smooth muscle cells
E Connective tissue layer

The varicose veins of the lower extremities often develop in people who are forced to work standing
for a long time. What caused changes in the structure of the walls of veins?
A *Weak development of elastic elements of the walls
B Weak development of striated muscle tissue
C Weak development of collagen fibers
D The thinning of the endothelial layer
E Weak development of reticular fibers

A histological specimen shows a blood vessel. Its inner coat is composed by endothelium and
subendothelial layer. The middle coat consists of smooth muscle cells bundles. Tunica adventitia
well developed and consists of loose connective tissue and some smooth muscle cells. Such
morphological characteristics are typical for which vessel?
A *Muscular vein (large vein)
B Muscular artery
C Non-muscular vein
D Mixed artery (small artery)
E Elastic artery

During morphological investigation in a histological specimen one can see irregularly shaped vessel
tunica media of which formed by bundles of smooth muscle cells and layer of connective tissue.
What type of vessel is it?
A *Large vein (muscular type)
B Muscular artery
70
C Lymphatic vessel
D Venule
E Arteriole

CARDIOVASCULAR SYSTEM. HEART.

Tests of the “KROK-1” database

2015
(questions are absent)

2014

The epithelial tumor of pericardium was diagnosed at a 53-year-old patient. What kind of
epithelium was the source of this tumor?
A *Simple squamous
B Simple pseudostratified
C Transitory
D Stratified keratinizing
E Stratified non-keratinizing

2013

Large cells with light cytoplasm and eccentrically located nucleus are revealed in the histological
specimen of the heart wall between endocardium and myocardium. What kind of the heart cells
have following morphologic signs?
A. *Purkinje cells
B. Pacemaker cells
C. Cardiac muscle cells
D. Endocrine cells
E. Adipose cells

2012

Concentration of calcium is increased in sarcoplasma of cardiac muscle cells during myocardium


contraction. Which structure participate in calcium storage
A. *L systems
B. Lysosomes
C. Ribosomes
D. T-systems
E. Nucleolus

As a result of thrombosis of left coronary artery the group of contractive cardiac muscle cells have
been destroyed. At the expense of which cells reparation in the area of damage will occur?
A. *Fibroblasts
B. Cardiac muscle cells
C. Myosymplast
D. Myosatellitocytes
E. Smooth muscle cells

71
There are detected cells of squared shape, 80-120 micrometers in size, with a centrically positioned
nucleus and well-developed myofibrils connected with the help of intercalated discs in a heart
specimen. What function is connected with these cells?
A. *Heart contraction
B. Nerve impulses conduction
C. Endocrine
D. Protective
E. Regenerative

In a slide revealed organ of cardio-vascular system. One of it coats is build by fibers which formed
anastomosis between each other. They formed by cells which connected with the help of
intercalated disks. Name this organ of cardio-vascular system.
A. *Heart
B. Large vein (muscle type)
C. Muscle artery
D. Elastic artery
E. Arteriole

In a histological specimen is represented heart wall. In one of its layers are found contractive,
conducting and secretory muscle cells, endomysium and blood vessels. Which layer of the heart is
it?
A. *Myocardium of the atrium
B. Endocardium of the ventricles
C. Epicardium
D. Adventitia
E. Pericardium

Morphological structure of the heart wall was restored after myocardial infarction. At the expense
of which tissue did regeneration occur?
A. *Connective
B. Smooth muscle
C. Striated muscle
D. Epithelial
E. Nervous

A 40 year-old patient had myocardial infarction of left ventricle. What morphological components
of the heart wall will replace this damage?
A. *Proliferation of connective tissue cells
B. Intracellular regeneration of contractive cardiac muscle cells
C. Proliferation of contractive cardiac muscle cells
D. Proliferation of conducting cells
E. Proliferation of contractive cardiac muscle cells and conducting cells
2005-2011

A 55-year-old patient consults a cardiologist with complaints about abnormal heart rhythm. What
structures of the heart are damaged in this case?
A. *Cardiac conducting cells
B. Contractile cardiomyocytes
C. Coronary vessels
D. Heart valves
E. -

72
The myocardium does not receive arterial blood during the systole. Which inclusions of
cardiomyocytes provide delivery of oxygen to them?
A. *Pigment.
B. Trophic.
C. Excretory.
D. Secretory.
E. Incretors.

A 45-year-old patient consults a doctor with complaints about leg swelling and pain. Varicose veins
and redness of the skin were observed during the examination. What structures of the subcutaneous
veins have changed most likely?
A. *Valves
B. Endothelium
C. Smooth muscle cells
D. Vasa vasorum
E. Collagen fibers

Impaired development of the myocardium of the ventricle was diagnosed in a newborn child. What
embryonic source have impaired development in this pathology?
A. *Myoepicardial plate.
B. Parietal splanchnopleura
C. Endoderm
D. Ectoderm
E. Mesenchyme

Portion of the heart muscle was injured as a result of myocardial infarction. It was accompanied by
massive loss of cardiomyocytes. What cells will provide the substitution of the formed defect in the
structure of the myocardium?
A. * Fibroblasts
B. Cardiomyocytes
C. Satellite cells
D. Epithelial cells
E. Smooth muscle cells

There are several layers in the wall of blood vessels and the heart wall. Which of the layers of the
heart is similar to the wall of blood vessels in histogenesis and tissue structure?
A. *Endocardium
B. Myocardium
C. Pericardium
D. Epicardium
E. Epicardium and myocardium

Accumulation of fluid in the pericardium is observed in the patient with pericarditis. Which cells of
the pericardium have violated activity?
A. *Mesothelium cells
B. Endothelial cells
C. Smooth muscle cells
D. Fibroblasts
E. Macrophages

One of walls of the heart is similar in the histogenesis and tissue structure to the blood vessel walls.
What is its source of development?
73
A. *Mesenchyme
B. Splanchnotom
C. Endoderm
D. Ectoderm
E. Somites

Changes in cardiomyocytes were revealed during microscopic examination of the heart of a dead
fetus. Development disorder of what source leads to this changes?
A. * Myoepicardial plate
B. Myotome
C. Endoderm
D. Ectoderm
E. Mesenchyme

One distinguishes star-shaped cardiomyocytes with a centrally located nucleus, developed rough
endoplasmic reticulum, Golgi apparatus and specific granules in a histological specimen of the
heart. What function is provided by these cells?
A. *Endocrine function
B. Contraction
C. Conduction of impulses
D. Protective function
E. Regenerative function

One of the organs of the cardiovascular system is built of cells that are connected to each other by
intercalated discs. Name this organ.
A *Heart
B Muscular type of vein
C Mixed type of artery
D Muscle type of artery
E Aorta

Branched cells containing few organelles but well developed rough endoplasmic reticulum and
secretory granules are visible in electron micrographs of the myocardium. Name these cells.
A *Secretory cardiomyocytes.
B Ventricular cardiomyocytes.
C Pacemaker cells.
D Transitional atypical cells.
E Cells of the AV bundle (of His).

One can see elongated cells with a small number of randomly located organelles and myofibrils and
peripherally positioned nucleus in a histological specimen of the heart wall under the endocardium.
Name these cells.
A. *Striated muscle cells
B. Contractile cardiomyocytes
C. Secretory cardiomyocytes
D. Smooth muscle cells
E. Cardiac conducting cells

Cardiomyocytes form the main part of the myocardium in a histological specimen of the heart wall.
They form muscle fibers due to intercalated discs. What type of junction provides electrical
connection of neighboring cells?

74
A. *Gap junction.
B. Desmosomes.
C. Hemidesmosomes.
D. Zonula occludens.
E. Simple contact.

There are atrial cardiomyocytes that produce natriuretic factor, which relax smooth muscle cells of
the wall of blood vessels and inhibit the secretion of aldosterone and vasopressin among various
types of cardiomyocytes. What type of cardiomyocytes is this?
A *Secretory cardiomyocytes
B Atypical cardiomyocytes
C Typical cardiomyocytes
D Fibrous cardiomyocytes
E-

It is known that the contractile cardiomyocytes are interconnected with intercalated disks in the
formation of which involved the junctions of three types. Indicate the correct answer.
A *Desmosomes, gap junction, zonula adherens
B Adherent junction, desmosome
C Synapse, desmosome, zonula occludens
D Synapse, desmosome, gap junction
E Zonula occludens, gap junction, zonula adherens

In a 45-year-old patient a doctor by EKG identified a violation of the synchronism of the cardiac
cycle phases, the rapid conduction of excitation through myocardium. Which of the following
structures that provide electrical connections between cardiomyocytes are damaged?
A. *Gap junction.
B. Hemidesmosomes.
C. Places of interweave of myofibrils in sarcolemma.
D. Intercalated disk.
E. Simple adherent junction.

A distinguish feature of the structure of Pacemaker cells of the cardiac conducting system is:
A. *Presence of a large amount of free calcium in the cytoplasm of cells
B. Presence of T-systems
C. Presence of a large number of myofibrils
D. Compact arrangement of myofilaments in the structure of myofibrils
E. Presence of a large number of mitochondria

Cells which are arranged in the form of light bands have a small number of myofibrils and
inclusions of glycogen are determined In a histological specimen of the heart. These cells belong to:
A. *AV bundle (of His)
B. Pacemaker cells
C. Conduction transitional cells
D. Endocrine cells
E. Contractile cells

Myocardiodystrophy is accompanied by a violation of the metabolism of cardiomyocytes. What


source of development is damaged in this pathology?
A. *Myoepicardial plate
B. Myotome
C. Endoderm
75
D. Ectoderm
E. Mesenchyme

Myoepicardial plate was damaged in embryo. What tissue will undergo the impaired development?
A. *Cardiac muscle tissue
B. Connective tissue
C. Smooth muscle tissue
D. Skeletal muscle tissue
E. Epithelial tissue

The disorder of heart rhythm (the rhythm is not sinus) on the electrocardiogram was revealed in 32-
year-old woman after suffering from myocarditis. What cardiomyocytes have impaired function?
A. *Pacemaker cells
B. Contractile cardiomyocytes
C. Conducting transitional cardiomyocytes
D. Cardiac conducting cells of AV bundle
E. -

A 54-year-old patient has a disorder of heart rhythm. What cells of the heart are affected?
A. *Pacemaker cells
B. Conducting transitional cells
C. Purkinje fibers
D. Typical cardiomyocytes
E. Secretory cardiomyocytes

Intensive therapy was performed to the patient with myocardial infarction in the intensive care unit.
Prognosis to the future myocardial function is poor. What can be the reason?
A. *Violation of the structure of smooth myocytes of the myocardium
B. Violation of the structure of contractile cardiomyocytes
C. Violation of the structure of myofibroblasts of the myocardium connective tissue
D. Violation of the structure of myosymplast of the myocardium
E. Changes in the structure of connective tissue of the myocardium

Impaired development of the ventricle myocardium was diagnosed to a newborn child. What source
of embryonic development is violated in this pathology?
A. *Visceral splanchnopleura
B. Parietal splanchnopleura
C. Endoderm
D. Ectoderm
E. Mesenchyme

The pathology of the valvular apparatus of the inner lining of the heart is revealing in the patient
with endocarditis. What tissues form the valves of the heart?
A. *Dense connective tissue, endothelium
B. Loose connective tissue, endothelium
C. Cardiac muscle tissue, endothelium
D. Hyaline cartilage, endothelium
E. Elastic cartilage, endothelium

The heart block occurred as a result of myocardial infarction. Atria and ventricles contracted
asynchronously. What structures were damaged?
A. *Cells of AV bundle ( of His )
76
B. Pacemaker cells of sinus-atrial node
C. Contractile myocytes of ventricles
D. Nerve fibers of nervus vagi
E. Sympathetic nerve fibers

A 45-year-old patient had a heart attack, which was accompanied by intensive squeezing
retrosternal pain that lasted more than 30 minutes, radiated to the lower jaw, left arm under the left
shoulder blade and did not pass after taking nitroglycerin, during physical activity. Subendocardial
infarction was diagnosed. What structures have undergone specific changes in the first place in this
case?
A *Endothelium
B Fibroblast
C Myocytes
D Fibrocytes
E Macrophages

The death of the group of contractile cardiomyocytes (myocardial infarction) occurred as a result of
thrombosis of the left coronary artery. What mechanism will ensure the regeneration in the
damaged area?
A. *Proliferation and secretory activity of fibroblasts
B. Proliferation of surviving cardiomyocytes
C. Proliferation of myosymplast
D. Proliferation and secretory activity of myosatellitocytes
E. Proliferation of smooth muscle cells

The total weight of the myocardium differs in several times in children and in adults. What is the
basic process that provides an increase of the weight of myocardium with age?
A. *Polyploidy of cardiomyocytes
B. Division of cardiomyocytes
C. Activation of macrophages
D. Proliferation of fibroblasts
E. Proliferation of smooth muscle cells

The fibers of the myocardium composed of cells which in places of connections form intercalated
discs. What tissue forms this layer?
A * Cardiac muscle tissue
B Skeletal muscle tissue
C Smooth muscle tissue
D Loose connective tissue
E Dense irregular connective tissue

A 56-year-old patient consulted a doctor with complaints about shortness of breath, palpitations,
swelling of hands and feet. She was diagnosed with congestive heart failure as a result of violation
of trophism of cardiac muscle. What intercellular structures provide storage of trophic material in
physiological conditions?
A *Glycogen, lipids
B Smooth endoplasmic reticulum
C Rough endoplasmic reticulum
D Pinocytotic vesicles
E Т -system

Atrioventricular block associated with a violation of conducting electrical impulses to the working
77
cardiomyocytes of the ventricle was revealed in the patient during ECG-examination. What
structural components of cardiac muscle tissue provide a consistent contraction in the working
cardiomyocytes of atria and ventricles?
A *Cardiac conducting cells
B Pacemaker cells
C Nerve fibers
D Collagen fibers
E Contractile cardiomyocytes

The patient was admitted to the hospital with tangential gunshot wound of the pericardium. What
kind of epithelium is damaged as a result of injury?
A *Simple squamous epithelium
B Simple columnar epithelium
C Simple cuboidal epithelium
D Stratified squamous keratinized epithelium
E Stratified squamous nonkeratinized epithelium

Visceral layers of mesoderm adjacent to the mesenchyme tube were destroyed in the experiment the
animal embryo. What structures of the heart will undergo impaired development?
A *Myocardium and epicardium
B Myocardium and endocardium
C Epicardium and endocardium
D Endothelium of the endocardium
E Mesothelium of the epicardium

A 56-year-old patient consulted a doctor with complaints about shortness of breath, palpitations,
swelling of hands and feet. She was diagnosed with congestive heart failure as a result of violation
of trophism of cardiac muscle. What intercellular structures provide storage of trophic material in
physiological conditions?
A *Glycogen, lipids
B Smooth endoplasmic reticulum
C Rough endoplasmic reticulum
D Pinocytotic vesicles
E Т-system

CENTRAL HEMATOPOIETIC ORGANS

Tests of the “KROK-1” database

2015

Reduction of the gland mass to 3.0 g was revealed in a 5-year-old child who died from acute
destructive staphylococcal pneumonia during the study of the thymus. Histological examination of
the thymus showed a decrease in the number of lobules of the gland, a significant decrease in the
number of lymphocytes with the collapse of the stroma of lobules, inversion of layers, cyst-like
enlargement of Hassall’s corpuscles. Which of the following diagnoses is most likely?
A. *Accidental involution of thymus
B. Thymus hypoplasia
C. Thymomegaly
D. Thymus dysplasia
78
E. Thymus agenesis

During autopsy of a 9-month-old girl’s body, which died due to severe pneumonia complicated with
sepsis, lack of thymus is observed. In the lymph nodes the lymphoid follicles and cortical substance
are absent; follicles of spleen are reduced in size with no light zones and plasma cells. What is the
cause of such structural changes?
A. *Thymus aging
B. Accidental involution of thymus
C. Thymus hypoplasia
D. Thymus atrophy
E. Thymus aplasia

A child received a hit to the retrosternal region during a game. As a result of this trauma an organ
located behind the retrosternal was damaged. Name this organ:
A. *Thymus
B. Thyroid gland
C. Heart
D. Pericardium
E. Larynx

2014

Histological examination of a 40 y.o. mans’ thymus revealed decreased share of parenchymatous


gland elements, increased share of adipose and loose connective tissue, and its enrichment with
thymus bodies. The organs mass was unchanged. What phenomenon is it?
A *Age involution
B Accidental involution
C Hypotrophy
D Dystrophy
E Atrophy

An electron microphotograph shows a macrophagic cell with erythrocytes at different stages of


differentiation located along its processes. This is the cell of the following organ:
A *Red bone marrow
B Thymus
C Spleen
D Tonsil
E Lymph node

2013

Amount of epithelioreticular cells and Hassall’s corpuscles are increased and area of medulla
became larger in the lobules of thymus during infections and intoxication. Give name of these
changes in the thymus.
A. *Accidental involution
B. B immunodeficiency
C. Thymico-lymphatic condition
D. Age involution
E. T immunodeficiency

79
The specimen represents sections of hemopoietic and immunologic organ. Organ has lymph tissue
forming different structures (lymph nodes, lobules, bars). In what organ does antigen-independent
proliferation and differentiation take place?
A *Thymus
B Lymphatic nodes
C Spleen
D Hemolymph nodes
E Tonsil

An organ which is made up of lobules is presented in a histological specimen. Each lobule has a
cortex and medulla. Parenchyma of lobules are formed of lymphoid tissue, which contains T-
lymphocytes at different stages of proliferation and differentiation. Microenvironment is presented
by epithelioreticular cells. The Hassall’s corpuscles are found in the medulla. What organ of the
human body has such morphological structure?
A * Thymus.
B Red bone marrow
C Lymph node.
D Adrenal gland.
E Spleen.

A teenager was irradiated with high radiation dose that resulted in serious damages of lymphoid
system, lysis of many lymphocytes. Restoration of normal hemogram is possible due to the
functioning of the following gland:
A *Thymus
B Thyroid
C Liver
D Pancreas
E Adrenal

One can find star shaped cells with oxyphilic cytoplasm which contacts with each other by means of
their processes in the slide of the human red bone marrow smear between myeloid row of cells and
adipose cells. Name these cells.
A. *Reticular
B. Fibroblasts
C. Macrophages
D. Dendrite cells
E. Osteocytes

Slide of hematopoietic organ is under investigation. This organ consists of differently shaped
lobules. In each of the lobule presents cortex and medulla. What organ possesses these
morphological characteristics?
A. *Thymus
B. Lymph node
C. Spleen
D. Tonsils
E. Appendix

2012

2012
80
An organ of hematopoiesis and immune response is presented in a histological specimen. This
organ is made up of lobules that are surrounded by layers of connective tissue; in the periphery of
lobules the number of cells is much higher than in the center. Lymphatic nodules are absent. What
organ of the body is represented?
A * Thymus
B Red bone marrow
C Lymph node
D Spleen
E Tonsil

A newborn baby has underdevelopment of the thymus. What line of hematopoiesis will be violated?
A*Lymphopoiesis
B Monocytopoiesis
C Erythropoiesis
D Granulopoiesis
E Platelet formation

Antibodies against thymosin were revealed in the patient. What cells have impaired differentiation
in the first place?
A * Т-lymphocytes
B В-lymphocytes
C Monocytes
D Macrophages
E–

Antibody against thymus hormones was injected under condition of experiment into the body of
investigated animal. Which cells differentiation will be affected first of all?
A. *T lymphocytes
B. Monocytes
C. Plasma cells
D. Macrophages
E. B lymphocytes
A parenchyma of the hematopoietic organ in which was found megakaryocytes was taken in a sick
person with diagnostic purpose. Name this organ.
A *Red bone marrow
B Spleen
C Thymus
D Lymph node
E Tonsil

The child was born with immunodeficiency. Cell mediated immunity has been affected that has
caused often viral infections. What organ has been damaged?
A. *Thymus
B. Red bone marrow
C. Lymph node
D. Spleen
E. Tonsil

The student was given specimens of two smears. Red blood cells cover the entire field of view in
the first specimen. Blood cells of different maturity are seen in the second specimen. What are the
kinds of smears?
81
A *Human blood and red bone marrow
B Blood and lymph
C Blood of frog and human blood
D Blood and smear of yellow bone marrow
E Smears of red and yellow bone marrow

Antigen independent proliferation and differentiation of T-lymphocytes was studied in the child
with impaired immune reactivity. What organ was investigated?
A * Thymus.
B Spleen.
C Lymph node.
D Red bone marrow.
E Palatine tonsils.

Medullary substance of a hemopoietic organ lobule is lighter colored and contains epithelial bodies
in a histological specimen. What organ are these morphological properties typical for?
A *Thymus
B Lymph node
C Spleen
D Liver
E Kidney

The cell with processes is visible in electron micrographs. There are lymphocytes at the stage of
differentiation in recesses of the plasmolemma of this cell. What organ of the human body has such
structure?
A * Thymus
B Red bone marrow
C Spleen
D Tonsil
E Liver

Developing cells are located in the form of islets in the red bone marrow. Some islets are associated
with macrophages. What blood cells develop in these islets?
A *Erythrocytes.
B Precursors of T- and B-lymphocytes.
C Monocytes.
D Platelets.
E Basophilic granulocytes.

Decreasing of the thymus parenchyma, increasing of adipose and loose connective tissue, enriching
of Hassall’s corpuscles, with remaining the same weight of the organ was revealed in a 40-year-old
man’s’ thymus during histological investigation. What is the name of this phenomenon?
A. *Age involution of the thymus
B. Accidental involution of the thymus
C. Thymus hypotrophy
D. Thymus dystrophy
E. Thymus atrophy

2005-2011

82
Decreased number of erythrocytes in one liter of the blood has been found in a patient as a result of
massive bleeding. After a short period of time the level of red blood cells became physiological.
What additional sources of hematopoiesis provided restoration of normal level of red blood cells?
A. *Foci of hematopoiesis of the yellow bone marrow of long bone diaphysis
B. Red bone marrow of long bones diaphysis
C. Red bone marrow of flat bones
D. Foci of hematopoiesis of the yellow bone marrow of long bone epiphysis
E. Red bone marrow of long bone epiphysis

The patient has symptoms of autoimmune damage of organs that may be the results of a violation of
the structure and function of the blood thymic barrier. What structures are involved in its formation?
A Endothelial cells and basement membrane
B Macrophages
C Hassall’s corpuscles
D Postcapillary venules of medulla
E Connective tissue trabeculae

A 46-year-old patient was admitted to the hematological department. It was found that he had
disorder of Granulocytopoiesis and thrombocytogenesis processes. In what organ does this
pathological process take place?
A *Red bone marrow
B Thymus
C Spleen
D Lymphatic ganglion
E Palatine tonsil

In the red bone marrow the blood cells, which develop, are located by the islands. Some of the
islands associated with macrophages. What blood cells are developed in these islands?
A * Erythrocytes
B Precursors of T- and B-lymphocytes
C Monocytes
D Platelets
E Basophilic granulocytes

Patient has disordered processes of erythropoiesis, granulocytopoiesis, monocytopoiesis and


thrombocytopoiesis. Which hematopoietic organ had been affected?
A. *Red bone marrow
B. Thymus
C. Spleen
D. Lymph node
E. Tonsil

One can see macrophage with erythrocytes at different stages of differentiation situated along
processes its processes in the electron microphotograph. What organ is it?
A. *Red bone marrow
B. Thymus
C. Spleen
D. Tonsil
E. Lymph node

83
One can see the organ stroma of which consists of reticular tissue, adipose cells, macrophages and
osteogenic cells in the slide. What organ is represented in the slide?
A. *Red bone marrow
B. Spleen
C. Thymus
D. Lymph node
E. Tonsil

One can see an organ which consists of lobules and stroma includes epitheliocytes with processes in
the specimen. What organ is represented in the slide?
A. *Thymus
B. Red bone marrow
C. Spleen
D. Tonsil
E. Lymph node

PERIFERAL HEMATOPOIETIC ORGANS

Tests of the “KROK-1” database

2014

In a microscopic specimen is a bean-shaped organ which has cortical and medullar substance.
Cortical substance is represented by separate spherical nodules 0,5-1 mm in diameter, medullar
substance – by medullary cords. What organ is this?
A. *Lymph node
B. Kidney
C. Thymus
D. Adrenal gland
E. Spleen

One has done histological section through lymph node. In the slide one can see enlargement of it
paracortex. Proliferation of what cells of lymph node have caused this process?
A. *T lymphocytes
B. Dendritic cells
C. Plasma cells
D. Macrophages
E. Reticular cells

Student got 2 histological specimens. They both have lymphatic nodules. First slide has only
follicles but second one has follicles with eccentrically positioned vessel. Determine these slides.
A. *First-lymph node, second-spleen
B. First-red bone marrow, second -spleen
C. First thymus, second- spleen
D. First liver, second -lymph node
E. First-liver, second -spleen

In a histological specimen parenchyma of an organ is represented by lymphoid tissue that forms


lymph nodes; the latter are arranged in a diffuse manner and enclose a central artery. What
anatomic formation has such morphological structure?
A *Spleen
84
B Tonsil
C Lymph node
D Thymus
E Red bone marrow

2013

The increase in the amount of lymphoid tissue, which may indicate the activation of immune
response, is determined in histological specimens of spleen and lymph node. Name the place where
the antigen-dependent proliferation and differentiation of B-lymphocytes (B-zone) are performed in
these organs.
A * Germinal center of lymphatic nodules
B Mantle zone.
C Paracortical zone.
D Medullary sinuses.
E Peryarterial zone

A specimen shows an organ covered with the connective tissue capsule with trabeculae radiating
inward the organ. There is also cortex containing some lymph nodules, and medullar cords made of
lymphoid cells. What organ is under study?
A *Lymph node
B Thymus
C Spleen
D Red bone marrow
E Tonsils

Morphological investigation of the spleen revealed activation of immune reactions in the organism.
In which structures of this organ do antigen depended proliferation of T lymphocytes begin?
A. *Periarterial sheath of white pulp
B. Central zone of white pulp
C. Germinal center
D. Marginal zone of white pulp
E. Red pulp

Focuses of increased formation of plasma cells were revealed in the biopsy sample of the lymph
node. Antigen depended stimulation of which immune cells have caused their formation?
A. *B lymphocytes
B. T lymphocytes
C. Macrophages
D. Dendrite cells
E. Interdigital cells

2012
Vital dye to the afferent vessel of the lymph node was injected the animal in the experiment. Which
cells of the lymph node will contain particles of the dye?
A *Macrophages.
B Reticular endothelial cells.
C В-lymphocytes.
D Plasma cells.
E Т-lymphocytes

85
The patient has an enlarged spleen and a reduced level of erythrocytes in peripheral blood. Which
cells of spleen have enhanced activity in this case?
A *Macrophages
B Lymphocytes
C Dendritic cells
D Plasma cells
E Reticular cells

In a histological specimen parenchyma of an organ is represented by lymphoid tissue that forms


lymph nodes; the latter are arranged in a diffuse manner and enclose a central artery. What
anatomic formation has such morphological structure?
A *Spleen
B Tonsil
C Lymph node
D Thymus
E Red bone marrow

A histological specimen represents an organ that has both cortical and medullary substance. Cortical
substance consists of an external zone that contains lymph nodules as well as of a paracortical zone.
Medullary substance contains medullary cords, sinuses and trabeculae. What organ possesses these
morphological signs?
A *Lymph node
B Spleen
C Kidney
D Thymus
E Adrenal glands

One can see an organ where lymphocytes formed 3 types of lymphoid structures such as lymphatic
nodules, medullary cords and sinuses in the specimen. What organ is it?
A. *Lymph node
B. Spleen
C. Thymus
D. Tonsil
E. Red bone marrow

In the specimen revealed an organ in the reticular stroma of which situated blood formed elements
and seen lymphoid formation. What organ is this?
A. *Spleen
B. Lymph node
C. Tonsil
D. Thymus
E. Red bone marrow

In a specimen revealed roundish formation of the lymphocytes with a central artery in the center.
What organ is it?
A *Spleen
B kidney
C Lymph node
D Thymus
E Red bone marrow

86
2005-2011

A doctor noted an enlarged spleen during the examination of a 20-year-old patient. The level of
erythrocytes was 3,2х1012 /L in the analysis of the blood. What cells of the spleen have enhanced
function that caused such changes?
A. *Macrophages
B. Endothelial cells
C. Dendritic cells
D. Microphages
E. Follicular dendritic cells

Examination of a patient who was exposed to the ionizing radiation revealed damage of white pulp.
What cells of white pulp undergo pathological changes?
A *Lymphocytes
B Neutrophilic leukocytes
C Basophilic leukocytes
D Monocytes
E Tissue basophils

There are cells that produce substances stimulating the proliferation of T-lymphocytes and have lost
ability to phagocytosis in a histological specimen of the lymph node on the border of cortex and
medulla. Name these cells.
A *Follicular dendritic cells
B Т – lymphocytes
C В – lymphocytes
D Macrophages
E Plasma cells

The spleen is not a vital organ but its rupture in trauma can cause the death of a person. It is
associated with such its function as:
A. *Deposition of blood
B. Deposition of iron
C. Elimination of red blood cells and platelets
D. Formation of B-lymphocytes
E. Synthesis of splenin

In the slide which was made from the spleen one can see white and red pulp in the base of which is
rest special tissue which formed their stroma. What tissue is it?
A. *Reticular connective tissue
B. Dense connective tissue
C. Adipose tissue
D. Muscle tissue
E. Nervous tissue

In the histological cross section of the lymph node in the experimental animal after antigen
stimulation in the medullary cords one can find huge amount of cells with intensively basophilic
cytoplasm, eccentrically positioned nucleus with chromatin giving the illusion of the spokes of the
wheel and light area of cytoplasm near it. Name these cells.
A. *Plasma cells
B. Macrophages
C. Fibroblasts
D. Adipose cells
87
E. Mast cells

15-year-old patient has enlarged tonsils because of the tonsillitis. Which histological structures of
these organs take place in immune protection of the body at a response of streptococcus invasion?
A. *Lymphatic nodules
B. Stratified squamous keratinized epithelium
C. Stratified squamous non keratinized epithelium
D. Loose connective tissue
E. Crypt

INTRACELLULAR CONNECTIONS AT IMMUNE REACTIONS

Tests of the “KROK-1” database

2015

The patient consulted an immunologist with complaints about diarrhea, weight loss within a few
months, low-grade fever, and swollen lymph nodes. The doctor suggested that the patient has HIV
infection. Which cells have to be examined in the first place?
A. Т-helpers
B. Т-suppressors
C. В-lymphocytes
D. Monocytes
E. Plasma cells
2014

Rejection of transplant developed in the patient after transplantation of heterogeneous kidney.


Name main effector cells, which take place in this immune reaction?
A. *T killer
B. B lymphocytes
C. T suppressor
D. T helper
E. Plasma cells

2013

Donor skin transplantation was performed to a patient with extensive burns. On the 8-th day the
graft became swollen and changed color; on the 11-th day graft rejection started. What cells take
part in this process?
A *T-lymphocytes
B Erythrocytes
C Basophils
D Eosinophils
E B-lymphocytes

In a patient with clinical signs of immunodeficiency the number and functional activity of T and B
lymphocytes are not changed. Defect with dysfunction of antigen-presentation to the
immunocompetent cells was found during investigation on the molecule level. Defect of what cells
is the most probable?
A *Macrophages, monocytes
B Т-lymphocytes, В-lymphocytes
C NK-cells
88
D Fibroblasts, Т-lymphocytes, В-lymphocytes
E 0-lymphocytes

A female patient underwent liver transplantation. Her condition became worse because of reaction
of transplant rejection 1,5 month after it. What factor of immune system plays the leading part in
this reaction?
A *T-killers
B Interleukin-1
C Natural killers
D B-lymphocytes
E T-helpers

2012

Humoral immune response developed in response of the introduction of foreign antigens during
vaccination of the child. Specify the main spleen cells involved in the immune response.
A * Macrophages, Т-helpers, В-lymphocytes
B Т-killers, Т-helpers
C В-lymphocytes
D Т-suppressors and T-helpers, macrophages
E В-lymphocytes

Rejection of transplant was revealed during heterotransplantation of an organ. What blood cells will
ensure this process?
A. *T killer
B. T helper
C. T suppressor
D. T 0 lymphocytes
E. T memory cells

2005-2011

Fabrician’s bag was removed from the chicken embryo. What disorder of immunity will occur in
this case?
A *The lack of humoral responses
B Deficiency of T-killers
C Deficiency of T-helpers
D Hyper production of T-suppressors
E The lack of T- and B-lymphocytes

The same antigen was injected twice after a certain period of time to the animal in the experiment.
In this case secondary immune response developed faster and was more intense. What cells provide
this rapid response?
A Memory cells T- and B- types and immunoblasts
B Plasma cells
C NK-cells
D Т-killers
E Т-suppressors

The patient was diagnosed with AIDS after a positive reaction enzyme immunoassay. What cells of
the peripheral blood are affected in first place?
A *Т-helpers
89
B Т-suppressors
C Т-killers
D Т-memory cells
E Macrophages

There are mutations of somatic cells in the body by the impact of unfavorable factors. What cells
participate in the destruction of genetically modified cells?
A * NK-cells
B Т-helpers
C Plasma cells
D Monocytes
E В-lymphocytes

Cells having the receptors of T cells (TCR+), enhance the activity of other T- and B-lymphocytes
are distinguished among the subpopulations of lymphocytes. Name these cells.
A *Т-helper
B Т-cytotoxic lymphocytes (Т-killer)
C Т-suppressor
D Т-memory cell
E NK-cells

Cells having receptors of T cells (TCR+) which are not immunoglobulins; recognize antigens
associated with major histocompatibility complex; produce perforins and other proteins that destroy
other cells infected by viruses and some tumor cells are detected among the subpopulations of
lymphocytes by immunological methods. Name these cells?
A *Т-killer
B Т-helper
C Т-suppressor
D Т-memory cell
E NK-cell

The cells which have membrane receptors for the IgM, under the influence of specific antigens
become activated, proliferate by mitotic division, differentiate into plasma cells that produce
antibodies (immunoglobulins) are distinguished among subpopulation of lymphocytes. What are
these cells?
A *В-lymphocytes
B Т-memory cells
C Т-killers
D Т-suppressors
E В-memory cells

To prevent epidemic of California virus vaccine (heterogeneous protein) was injected into the
organism of human. What cells will take place in specific immunity?
A. *Lymphocytes
B. Adipose cells
C. Pigmentocytes
D. Fibroblasts
E. Adventitial cells

Burn wound was covered with pig skin (heterotransplantation). Call effector cells, which will reject
transplant (pig skin).
A. *T killer
90
B. T helper
C. T suppressor
D. B lymphocytes
E. Natural killer

ENDOCRINE SYSTEM. HYPOPHYSIS. HYPOTHALAMUS.

Tests of the “KROK-1” database

2015

To the doctor turned a 27-year-old man. An enlargement of hands, feet and lower jaw were
revealed during the examination. Moreover the deformation of joints and hormonal disorders
(impotence, testicular atrophy) were observed. What gland has impaired function?
A. Anterior hypophysis
B. Adrenal glands
C. Pineal gland
D. Thyroid gland
E. Parathyroid gland

2014

The patient has a significant increase in daily urine output. Which hormone of the hypothalamus has
impaired secretion?
A *Vasopressin
B Oxytocin
C Liberins
D Statins
E Thyroid

25-year-old woman consulted a doctor with complaint about reduction in milk production a month
after delivery. What hormone is produced in insufficient amount?
A *Prolactin
B Adrenocorticotropic hormone
C Somatostatin
D Insulin
E Glucagon

The parenchyma of the anterior hypophysis is presented by trabecules which are formed by
glandular cells. There are cells with granules that stain with basic dyes and contain glycoproteins
among cells of adenohypophysis. What are these cells?
A *Gonadotropes, thyrotropes
B Somatotropes
C Melanotropes
D Mammotropes
E Chromophobes

Pocket of Rathke, which is directed to the base of the future brain, is formed from the ectodermal
epithelium lining the upper part of the oral fossa of the human embryo. What develops from this
embryonic rudiment?
A *Adenohypophysis
91
B Neurohypophysis
C Medial eminence
D Hypophyseal stalk
E Anterior hypothalamus

Roentgenological examination of skull base bones revealed enlargement of sellar cavity, thinning of
anterior clinoid processes, destruction of different parts, and destruction of different parts of sella
turcica. Such bone destruction might be caused by a tumor of the following endocrine gland:
A *Hypophysis
B Epiphysis
C Thymus gland
D Adrenal glands
E Thyroid gland

An endocrine gland with parenchyma consisting of epithelium and neural tissue is under
morphological examination. Epithelial trabeculae have two types of cells: chromophilic and
chromophobic. Identify this organ:
A *Hypophysis
B Adrenal glands
C Hypothalamus
D Thyroid gland
E Parathyroid gland

2013

Epithelial strands, which are composed of chromophilic (acidophilic, basophilic) and chromophobic
cells are revealed in a histological specimen of endocrine glands. What organ is presented in a
histological specimen?
A *Adenohypophysis
B Adrenal gland
C Neural hypophysis
D Thyroid gland
E Pineal gland

A 32-year-old patient consulted a doctor about the absence of lactation after parturition. Such
disorder might be explained by the deficit of the following hormone:
A *Prolactin
B Somatotropin
C Vasopressin
D Thyrocalcitonin
E Glucagon

Examination of a patient revealed overgrowth of facial bones and soft tissues, tongue enlargement,
wide interdental spaces in the enlarged dental arch. What changes of the hormonal secretion are
possible?
A *Hypersecretion of the somatotropic hormone
B Hyposecretion of the somatotropic hormone
C Hypersecretion of insulin
D Hyposecretion of thyroxin
E Hyposecretion of insulin

92
A 40-year-old patient complains about intensive heartbeats, sweating, nausea, visual impairment,
arm tremor, hypertension. From his anamnesis: 2 years ago he was diagnosed with
pheochromocytoma. What hormones hyper production does the given pathology cause?
A *Catecholamines
B Aldosterone
C Glucocorticoids
D ACTH
E Thyroidal hormones

A man has suddenly reduced dieresis after loss of 1,5 liter of the blood . The increased secretion of
what hormone caused such diuresis alteration?
A *Vasopressin
B Corticotropin
C Natriuretic
D Cortisol
E Parathormone

The aim of the morphological study was to investigate an endocrine gland with parenchyma
consisting of epithelium and neural tissue. In the epithelial trabeculae the study revealed two types
of cells: chromophile and chromophobe. Identify this organ:
A *Pituitary gland
B Adrenal gland
C Hypothalamus
D Thyroid gland
E Parathyroid gland

2012
The patient was diagnosed with acromegaly in endocrinology department. What cells of the
pituitary gland are hyperactive, which causes this disease?
A *Somatotropes
B Gonadotropes
C Chromophobes
D Mammotropes
E Thyrotropes

The milk secretion is reduced in the woman, who is breast-feeding a baby. The secretory process in
lactotropic cells is not violated. What hormone is insufficient?
A *Prolactin
B Oxytocin
C Progesterone
D Estrogens
E Follicle stimulating hormone

45-year-old man consulted a doctor with complaints about increase in size of hands, feet, brow,
cheekbones, and nose. The patient was diagnosed with acromegaly. What cells of hypophysis are
hyperactive?
A *Somatotropes
B Adrenocorticotropes
C Gonadotropes
D Thyrotropes
E Mammotropes

93
Proportionally built child has growth retardation. Which cells of the anterior hypophysis are
affected?
A *Somatotropes
B Mammotropes
C Gonadotropes
D Adrenocorticotropes
E -

The patient with acromegaly has the enlargement of extremities and bony skeleton of the face. What
cells causing this disease are hyperactive?
A *Somatotropes
B Mammotropes
C Gonadotropes
D Adrenocorticotropes
E -

Muscles of the myometrium don’t contract sufficient in the woman during childbirth. This is
manifested by weakness of labor. What nuclei of hypothalamus have hypofunction?
A *Paraventricular
B Arcuate
C Suprachiasmatic
D Supraoptic
E Dorsomedialis

Axons of neurosecretory cells of supraoptic nucleus of the hypothalamus were cut in the animal
during an experiment. What hormone of the pituitary gland has impaired storage?
A *Vasopressin
B Somatotropin
C Prolactin
D Adrenocorticotropin
E Lipotropin

Reduce of uterine contractive activity appears in the woman during childbirth. Which hormone of
the hypothalamus can increase the contractile activity of the uterus in this situation?
A *Oxytocin
B Vasopressin
C Liberins
D Statins
E -

The woman had a weak contraction of the myocytes of uterus during childbirth. Insufficiency of
what hypothalamic hormone can cause this condition?
A *Oxytocin
B Folliberin
C Prolactoliberin
D Somatoliberin
E Vasopressin

Both sex glands were removed from the woman during surgical intervention for malignant ovarian
tumors. What will happen with gonadotropic cells of the pituitary gland in this case?
A *Hypertrophy
B Atrophy
94
C Necrosis
D Malignization
E Lysis

A 40-year-old woman has weak labor activity caused by weak contraction of myometrium. What
hormone should be injected to help this woman?
A. *Oxytocin
B. Hydrocortisone
C. Dexamethasone
D. Aldosterone
E. Prednisolone

Increasing of the cavity of sella turcica and thinning of processus clinoideus anterior and damaging
of different areas of sella turcica were revealed in the base of the cranium bones during X-ray
examination. Tumor of what endocrine gland can cause such damaging of bones?
A. *Hypophysis
B. Thymus
C. Pineal gland
D. Thyroid gland
E. Adrenal gland

For morphological investigation endocrine gland was represented. Parenchyma of this gland
consists of epithelium and nervous tissue. In epithelial trabeculae revealed two types of cells
chromophiles and chromophobes. Name this organ.
A. *Hypophysis
B. Adrenal gland
C. Hypothalamus
D. Thyroid gland
E. Parathyroid gland

2005-2011

The patient complaints about sharp increase in urine output (up to 5-7 L of urine per day).
Examination showed that the patient has a reduced secretion of vasopressin. Which cells are
showing a lack of secretory activity?
A *Neurosecretory cells of the hypothalamus
B Endocrine cells of the anterior pituitary
C Endocrine cells of intermediate part of the hypophysis
D Pituicytes
E Cells of tuberal part of the hypophysis

The very tall patient with long thick fingers, large lower jaw and protruding lower lip consulted a
doctor. What hormone probably has an increased secretion?
A *Growth hormone of the anterior pituitary
B Gonadotropic hormone of the anterior pituitary
C Glucocorticoids of fascicular zone of the adrenal gland
D Thriiodothyronine of thyroid follicles
E Somatostatin of D-cells of Langerhans islets

An endocrine gland, the parenchyma of which is formed by the epithelial and nervous tissues is
presented in a histological specimen. In the epithelial tissue are determined chromophilic and
95
chromophobic cells. There are cells secretory granules of which provide basophilic staining of the
cytoplasm among chromophiles. Name these cells
A *Thyrotropes, gonadotropes, adrenocorticotropes
B Mammotropes, adrenocorticotropes, gonadotropes
C Somatotropes, mammotropes
D Thyrotropes, somatotropes, gonadotropes
E Mammotropes and gonadotropes

One can see cells, the cytoplasm of which is stained basophilic in a histological specimen of
anterior pituitary gland among endocrine cells. These cells synthesize follicle-stimulating and
luteinizing hormones. What are these cells?
A. *Gonadotropes
B. Thyrotropes
C. Mammotropes
D. Somatotropes
E. Pituicytes

Student made an inaccuracy by telling that synthesis of glucocorticoids from cortical region of
adrenal gland is stimulated by hypophysis hormones during characterization of stress. What
clarification should he make?
A. *Adrenocorticotropic hormone
B. Somatotropin
C. Gonadotropic hormone
D. Mammotropic hormone
E. Thyrotropic hormone

Diuresis has significantly increased in a 50-year-old patient after infectious disease of the brain.
Level of glucose was 4,1 moll/L in the blood. The doctor decided that it is inadequate function of
endocrine system. What hormone is most likely insufficient?
A. *Vasopressin
B. Glucagon
C. Insulin
D. Cortisol
E. Aldosterone

Experimental animal produces big amount of urine and have strong thirst. Urine doesn’t have sugar.
What cells disordered?
A. *Neurosecretory cells of supraoptic nucleus
B. Follicular endocrine cells of the thyroid gland
C. Principal cells
D. Endocrine cells of the zona glomerulosa of adrenal gland
E. Endocrine cells of the medullary region of adrenal gland

Patient has suffered from hypothyroidism for 7 years. Deficiency of thyrotropic hormones was
revealed. What cells of adenohypophysis will be changed?
A. *Thyrotropes
B. Gonadotropes
C. Corticotropes
D. Somatotropes
E. Mammotropes

96
Increased amount of follicle stimulating hormone was revealed on the background of deficiency of
sex hormones in a 30-year-old female. What cells synthesize this hormone?
A. *Gonadotropes
B. Thyrotropes
C. Corticotropes
D. Somatotropes
E. Mammotropes

One can see cells with oxyphilly stained cytoplasm in the specimen of adenohypophysis between
endocrine cells. These cells secrete prolactin. Name these cells.
A. *Mammotropes
B. Thyrotropes
C. Adrenocorticotropes
D. Gonadotropes
E. Pituicytes

Examination of a 32-year-old patient revealed disproportional skeleton size, enlargement of


superciliary arches, nose, lips, tongue, jaw bones and feet. What gland function was disturbed?
A *Hypophysis
B Epiphysis
C Pancreas
D Thyroid
E Suprarenal

A 35-year-old patient complains about permanent thirst, bad appetite. He drinks every day 9 L of
fluid. Daily diuresis increased, urine discolored. Most probable reason of such pathology
development in this patient is:
A. *Hypothalamic nucleuses
B. Epithelia of nephron tubules
C. Adenohypophysis
D. Pineal gland
E. Basal membrane of glomerulus capillaries

A 27-year-old patient has bronze color of the skin which typical for Addison disease after sepsis.
Mechanism of hyper pigmentation is based on increasing of hormone secretion.
A. *Melanostimulating
B. Somatotropic
C. Gonadotropic
D. B lipotropic
E. thyrotropic

Cessation of bleeding connects with action of oxytocin to the uterus wall after parturition. Which
layer of the organ does react at the action of this hormone?
A. *Myometrium
B. Endometrium
C. Perimetrium
D. Parametrium
E. Submucosa

A 50-year-old patient complains about enlargement of ears, nose and hands size. Hyperfunction of
which gland does these symptoms give?
A. *Hypophysis
97
B. Thyroid gland
C. Sex glands
D. Adrenal glands
E. Pineal gland

ENDOCRINE SYSTEM. EPIPHYSIS. ADRENAL GLAND.

Tests of the “KROK-1” database

2015

Endocrine system organ is represented in the slide. It surrounded by connective tissue capsule
which extends trabeculae in the center of the organ and formed lobules. Each lobule contains two
types of cells. Neurosecretory pinealocytes are polygonal cells with processes located in the center
and glial cells (astrocytes) located in the periphery. What organ is this?
A. *Pineal gland
B. Hypophysis
C. Hypothalamus
D. Thyroid gland
E. Medullary region of the adrenal gland

Cortex and medulla, which are separated by a layer of connective tissue, are distinguished in a
histological specimen of an endocrine gland. Parenchyma cells of the cortex form three zones: in
the superficial zone there are round clusters, in the middle – parallel strands, in a deep – cell cords
form a network. What gland is this?
A. Adrenal glands
B. Thyroid gland
C. Hypophysis
D. Pineal gland
Е. Hypothalamus

2014

Endocrinologist observed a 40-year-old patient, who has insufficient function of the adrenal glands
cortex, which is manifested by a reduction of the level of aldosterone in the blood. What cells of the
cortex have impaired function?
A *Cells of the zona glomerulosa
B cells of the zona fasciculata
C Cells of the zona reticularis
D Cells of the intermediate zone
E Cells of X-zone

It is known that aldosterone regulates amount of sodium in the body. What cells of the adrenal
gland do these hormones synthesize?
A. *Cells of zona glomerulosa
B. Chromaffin cells produce epinephrine
C. Cells of zona reticularis
D. Cells of zona fasciculata
E. Chromaffin cells produce norepinephrine

98
In the specimen parenchymal organ is represented. External layer of the cortex of it is formed by
glomeruli created by endocrine cells. What organ is it?
A. *Adrenal gland
B. Lymph node
C. Spleen
D. Thyroid gland
E. Ovary

A patient has been given high doses of hydrocortisone for a long time. This caused atrophy of one
of the adrenal cortex zones. What zone is it?
A *Fascial
B Glomerular
C Reticular
D Glomerular and reticular
E-

2013

A histological specimen represents parenchymal organ, which has cortex and medulla. Cortex
consists of epitheliocytes bars with blood capillaries between them; the bars form three zones.
Medulla consists of chromaffinocytes and venous sinusoids. Which organ has these morphological
features?
A *Adrenal gland
B Kidney
C Lymph node
D Thymus
E Thyroid

Large cells which impregnate by solution of potassium dichromate were found in the study of
removed during surgery adrenal gland. What hormone is synthesized by these cells?
A *Adrenalin
B Aldosterone
C Secretin
D Thyroxin
E Cholecystokinin

One of the adrenal glands was removed from the patient for medical reasons. How will change the
structure of the second adrenal gland?
A *Hypertrophy of cells of the cortex and medulla
B Atrophy of the medulla cells
C Atrophy of the cortex cells
D Necrosis of the medulla cells
E Lysis of the cortex cells

2005-2011

As a result of stress the relaxation of smooth muscle cells of the intestine occurred in adult. Result
of action of what substance can cause this condition?
A. *Adrenaline
B. Histamine
C. Gastrin
99
D. Cholecystokinin
E. Serotonin

The adrenal gland is a complex endocrine gland with many areas that produce complexes of various
hormones. What hormones are produced by endocrinocytes of the zona fasciculate of the adrenal
cortex?
A *Glucocorticoids
B Androgens, female sex hormones
C Aldosterone
D Noradrenalin, adrenalin
E Renin

Precocious puberty occur in the experimental animals after endocrine glands removal. By what
gland removal is this happened?
A. *Pineal gland
B. Hypophysis
C. Adrenal gland
D. Thyroid gland
E. Parathyroid gland

In a histological specimen of adrenal cortex there are small polygonal cells that form roundish
clusters and contain some lipid inclusions. What part of adrenal gland is represented in this
histological specimen?
A *Glomerulosa zone
B Intermedia zone
C Fasciculata zone
D Reticularis zone
E-

Microscopic examination of a parenchymatous organ revealed that its epithelial cords formed
glomerular, fascicular and reticular zones. The central part of the organ was presented by
accumulations of Chromaffin cells. Specify this organ:
A *Adrenal gland
B Thyroid gland
C Epiphysis
D Liver
E Hypophysis

ENDOCRINE SYSTEM. THYROID AND PARATHYROID GLAND.

Tests of the “KROK-1” database

2015
(tests are absent)

2014
There are large endocrine cells, secretory granules of which are osmiophilic and argyrophilic in the
wall of follicles and interfollicular layers of connective tissue of the thyroid gland. Name these
cells.
A *Calcitoninocytes
100
B Thyrocytes
C Parathyrocytes
D Pinealocytes
E Pituicytes

A child has abnormal formation of tooth enamel and dentin as a result of low concentration of
calcium ions in blood. Such abnormalities might be caused by deficiency of the following hormone:
A *Parathormone
B Thyrocalcitonin
C Thyroxin
D Somatotropic hormone
E Triiodothyronine

One can see roundish structures in the endocrine gland specimen. Their wall is formed with one
layer of epithelial cells resting on the basal membrane. These structures contain homogeneous non
cellular mass in the middle of them. What gland is it?
A. *Thyroid gland
B. Adrenal gland, cortex
C. Parathyroid gland
D. Anterior part of hypophysis
E. Posterior part of hypophysis

2013

A 2-year-old child experienced convulsions because of lowering calcium ions concentration in the
blood plasma. Function of what structure is decreased?
A *Parathyroid glands
B Hypophysis
C Adrenal cortex
D Pineal gland
E Thymus

Parodontitis is treated with calcium preparations and a hormone that stimulates tooth mineralization
and inhibits tissue resorption. What hormone is it?
A *Calcitonin
B Parathormone
C Adrenalin
D Aldosterone
E Thyroxine

A patient suffering from thyrotoxic symptoms of vegetoasthenic syndrome was revealed. What of
the following would show the histological appearance of a thyroid gland being stimulated by
thyroid-stimulating hormone (TSH)?
A *Columnar-shaped follicular cells
B Decreased numbers of follicular cells
C Increased numbers of parafollicular cells
D An abundance of colloid in the lumen of the follicle
E Decreased numbers of parafollicular capillaries

Kidneys of a man under examination show increased resorption of calcium ions and decreased
resorption of phosphate ions. What hormone causes this phenomenon?
A *Parathormone
101
B Thyrocalcitonin
C Hormonal form D3
D Aldosterone
E Vasopressin

2012

Columnar shape of thyrocytes, the increased number and height of microvilli, increased number of
plasmolemma invaginations on the basal surface are determined in a histological specimen of the
thyroid gland. What functional condition is characterized by such histological picture?
A *Enhancing of the functional activity
B Inhibition of the functional activity
C Normal function
D Involutional age-related changes
E -

A 30-year-old patient was diagnosed with thyroid gland hyperfunction. What shape do follicular
cells have in the follicles?
A. *Columnar
B. Polygonal
C. Squamous
D. Spindle
E. Cuboidal

A 42-year-old patient after resection of thyroid gland had convulsions. After injection of calcium
preparation she got relief. What endocrine glands disorder has caused this condition?
A. *Parathyroid
B. Adrenal
C. Ovary
D. Hypophysis
E. Pineal

2005-2011

Large argyrophilic cells located in the wall of follicles are visible in a histological specimen of the
thyroid gland after impregnation with silver salts. Which hormone is synthesized by these cells?
A *Calcitonin
B Thyroxin
C Parathyrin
D Aldosterone
E Adrenaline

Stroma and parenchyma are visible in a histological specimen of the thyroid gland. What type of
tissue forms the stroma of the gland?
A *Loose connective tissue
B Dense connective tissue
C Adipose tissue
D Muscle tissue
E Nervous tissue

102
Microscopic study of an endocrine gland revealed that its parenchyma consisted of follicular
structures. Their wall was formed by simple cuboidal epithelium, and their cavity was filled with
oxyphilic substance. What hormone is secreted by this gland?
A *Thyroxin
B Aldosterone
C Cortisol
D Parathormone
E Oxytocin

Tetany characterized by spasms of the striated muscles was developed in the patient after surgery of
the thyroid gland. Histological examination of the removed material revealed the formation under
capsule shared with thyroid gland. This formation is made up of cell trabeculae connected to each
other by means of desmosomal junctions. There are some with basophilic cytoplasm, other with
acidophilic cytoplasm among these cells. What formation is this?
A *Parathyroid gland
B Clusters of parafollicular cells
C Clusters of proliferating thyrocytes
D Interfollicular islet
E Tumor of the thyroid gland

A high level of basic metabolism without hyperthyroidism may be a manifestation of disease which
at the molecular level is characterized by the damage of oxidative phosphorylation. What cellular
component is damaged in this case?
A *Mitochondria
B Microtubules
C Lysosomes
D Golgi apparatus
E Peroxisomes

A 35-year-old patient has increase in body weight, decrease in body temperature, dry skin, hair loss,
depression functions of central nervous system, bradycardia. What gland has impaired function?
A. *Thyroid gland
A. Parathyroid gland
B. Pancreas
C. Adrenal gland
D. Gonads

Endocrine gland was removed by mistake in the patient during operation. It caused decreasing of
the calcium blood level. What gland was removed?
A. *Parathyroid
B. Hypophysis
C. Adrenal
D. Thyroid
E. Pineal

A 40-year-old patient consulted a doctor with complains about tachycardia, exophthalmos,


fatigability, reduced weight of the body. What cells increasing function does this condition connect?
A. *Follicular cells
B. Parafollicular cells
C. Parathyroid cells
D. APUD cells
E. Acidophil endocrine cells
103
Under action of harmful environmental factors lysosomes formation were disordered in follicular
cells. What part of hormone production in the thyroid gland will be disordered?
A. *Proteolysis of phagocytized colloid from follicles
B. Colloid synthesis
C. Iodination of colloid
D. Resorption of colloid
E. Thyroglobulin synthesis

Young woman consulted an endocrinologist with complains about sleepiness, depression, fatigue,
bad appetite and increasing of the body weight. Disorder of what endocrine gland can cause such
problems?
A. *Thyroid
B. Pancreas
C. Adrenal cortex
D. Adrenal medulla
E. Ovary

A patient has the sudden decrease of Са 2+ content in the blood. What hormone secretion will
increase?
A *Parathormone
B Thyrocalcitonin
C Aldosterone
D Vasopressin
E Somatotropin

A 9 y.o. boy was admitted to the endocrine department. This boy has already had several fractures
of wrist extremities due to bone brittleness. The function of the following endocrine gland is
disturbed:
A *Parathyroid
B Thyroid
C Thymus
D Adrenal
E Epiphysis

Clinical examination of a female patient revealed reduction of basal metabolism by 40%, gain in
body mass, drop of body temperature, face puffiness, sexual dysfunctions, inertness and apathy,
lowered intelligence. These symptoms are caused by dysfunction of the following endocrine gland:
A *Hypofunction of thyroid gland
B Hypofunction of parathyroid glands
C Hypophysis hyperfunction
D Epiphysis hypofunction
E Hyperfunction of thyroid gland

After a surgical procedure an experimental animal died from intense convulsions. What endocrine
glands were extracted?
A *Parathyroid
B Thyroid
C Adrenal
D Ovaries
E Testicles

104
During operation at thyroid gland two from four parathyroid glands were mistakenly removed. It
caused decreasing of the calcium blood level. What cells are target for parathyroid gland hormone
which increase level of calcium?
A. *Osteoclasts
B. Osteocytes
C. Osteoblasts
D. Fibroblasts
E. Chondrocytes

A two-year-old child had convulsions as a result of decreasing concentrations of calcium ions in the
plasma. This is caused by declining function of:
A. *Parathyroid glands
B. Hypophysis
C. Adrenal cortex
D. Pineal gland
E. Thymus

DIGESTIVE SYSTEM. ORAL CAVITY.

Tests of the “KROK-1” database

2015
A 30-year-old patient consulted a doctor with complaints about fever up to 38 C, weakness, sore
throat. Objectively: there is white coating on the tongue. What histological structure of the tongue is
involved in the formation of this coating?
A. Epithelium of the filiform papillae
B. Epithelium of the foliate papillae
C. Epithelium of the fungiform papillae
D. Epithelium of the circumvallate papillae
Е. Connective tissue base of all the lingual papillae

2014

A 22-year-old female student consulted a physician about fever up to 38oC, weakness, sore throat.
Objectively: there is white coating on the tongue. What histological structures of the tongue are
involved in the formation of this coating?
A *Epithelium of the filiform papillae
B Epithelium of the foliate papillae
C Epithelium of the fungiform papillae
D Epithelium of the circumvallate papillae
E Connective-tissue base of all the lingual papillae

2013

Changes in the uvula and soft palatine were detected during patient examination for diphtheria.
What epithelium was damaged?
A. *Stratified squamous
B. Pseudostratified
C. Simple squamous
D. Simple columnar
105
E. Cuboidal

Crypts of the tonsil, the epithelium of which is infiltrated by leukocytes are identified in histological
specimen. What kind of the epithelium does this organ cover?
A. *Stratified squamous nonkeratinized
B. Simple columnar
C. Stratified cuboidal
D. Stratified squamous keratinized
E. Ciliated

In order to speed up healing in the wound of oral mucosa a patient was prescribed a drug. It is
thermally stable protein which occurs in tears, saliva, and mothers’ milk as well as in a new-laid
hen’s egg. It is known that this protein is a factor of natural resistance of an organism. How is this
protein called?
A *Lysozyme
B Complement
C Interferon
D Interleukin
E Imanine

While examining the oral cavity a dentist revealed inflammation of papillae on the border of the
median and posterior third of the back of the tongue. What papillae are inflamed?
A *Papillae circumvallate
B Papillae fungiform
C Papillae foliate
D Papillae filiform
E Papillae conical

The reason of occurrence of some diseases of an oral cavity is connected with structural
peculiarities of its mucous membrane. What morphological attributes characterize these features?
A *No muscularis mucosa, stratified squamous epithelium
B Transitional epithelium, no submucosa
C Simple columnar ciliated epithelium
D Well developed muscularis, no submucosa
E Transitional epithelium, no muscularis mucosa

2012

Phenomenon of “coated tongue” was revealed in the patient with chronic gastritis during external
examination. This phenomenon is caused by cornification process. In what papillae of the tongue
does epithelium keratinize?
A. Filiform papillae
B. Fungiform papillae
C. Circumvallate papillae
D. Foliate papillae
E. -

It can be seen the anterior surface is lined with stratified squamous non-keratinized epithelium, and
the posterior one – with pseudostratified ciliary epithelium in a histological specimen of an oral
cavity organ. What organ is this?
A. Soft palate
B. Gum (gingiva)
106
C. Hard palate
D. Lip
E. Cheek

Chemical burn of the tongue dorsal surface happened in a 15-year-old girl. What epithelium was
damaged?
A. Stratified keratinized
B. Simple low cuboidal
C. Pseudostratified villous
D. Transitional
E. Simple squamous

2005-2011

One of the organs of the oral cavity consists of several folds of the mucosa, in the lamina propria of
which there are numerous lymphoid follicles. What is this organ?
A. *Palatine tonsil
B. Tongue
C. Parotid gland
D. Sublingual gland
E. Submandibular gland

The child has damaged a lateral surface of the tongue. Which papillae of the tongue are probably
damaged?
A. *Foliate papillae
B. Conical papillae
C. Circumvallate papillae
D. Filiform papillae
E. Fungiform papillae

The student remembered that the epithelium of the oral cavity mucosa is stratified squamous
nonkeratinized. In a histological specimen of cheeks he saw that on both sides of them epithelium
are keratinized. What should a student remember?
A. *The intermediate zone of buccal mucosa may be keratinized
B. The maxillary zone of buccal mucosa may be keratinized
C. The cheek area near the excretory duct of the parotid gland may be keratinized
D. The entire epithelium of the cheek mucosa may be keratinized
Е. -

The cause of some diseases of the oral cavity is connected with structural features of the mucous
membrane. What are the morphological traits which characterize these features?
A. *The absence in some places of the submucosa and muscularis mucosa
B. Mucous membrane is covered with transitional epithelium, the submucosa is absent
C. Mucous membrane is covered with simple columnar ciliated epithelium
D. Well-developed muscularis mucosa, submucosa is absent
E. Mucous membrane is covered with transitional epithelium, muscularis mucosa is absent

An employee of the chemical laboratory burned the right edge of the tongue due to the careless
handing with acids. What papillae of the tongue are probably damaged?
A. *Foliate
B. Conical
C. Circumvallate
107
D. Filiform
E. Fungiform

The dentist instructed the intern to examine the patient areas of the oral cavity mucosa which may
be keratinized. Where are these areas?
A. *On the hard palate, free edges of gums and along the intermediate part of the cheek
B. On the hard palate, tongue, gums
C. On the cheeks, lower lip, back of the tongue
D. On the hard palate and diaphragm of the oral cavity
E. On the tongue, gingival papilla, tonsils

Numerous excretory ducts of the minor salivary gland, the topography and location of which are not
typical for glands open on the tongue surface. Specify the location of the salivary glands of the
tongue.
A. *Muscularis externa
B. Mucous membrane
C. Submucosa
D. Epithelium of the upper surface of the tongue
E. Epithelium of the lower surface of the tongue

What are the structural features of the mucosa of the hard palate?
A. *Tightly adherent to the periosteum
B. Covered with transitional epithelium
C. Well developed muscularis mucosa
D. Well developed submucosa
E. Covered with ciliated columnar epithelium

What are the main structural features of the mucosa of gums?


A. *Tightly adherent to the periosteum
B. Accommodates many small salivary glands
C. Located on the submucosa
D. Lamina propria and muscularis mucosa are absent
E. Covered with transitional epithelium, submucosa is absent

ТРАВНА СИСТЕМА. ЗУБИ.

Tests of the “KROK-1” database

2015 р.
(тестові завдання з даної теми відсутні)

2014 р.
(тестові завдання з даної теми відсутні)

2013 р.
(тестові завдання з даної теми відсутні)

2012 р.
(тестові завдання з даної теми відсутні)

2005-2011 рр.
108
What is the structural component of the tooth that provides the trophism of dentine?
A. *Pulp
B. Periodontum
C. Enamel
D. Cellular cementum
E. Acellular cementum

Dark areas of the dentin were found on the section of the extracted tooth. Which process formed
these sites?
A. *Destruction of dentinoblasts
B. Proliferation of dentinoblasts
C. Hypertrophy of dentinoblasts
D. Atrophy of dentinoblasts
E. Proliferation of fibroblasts

Radial light bands were found in the dentin on the tooth section of elderly person. Such sites in the
dentin are called:
A. *Transparent dentin
B. “Dead ways”
C. Secondary dentin
D. Tertiary dentin
E. Irregular dentin

The dentin tubules are visible on the section of tooth crown. What is contained in these tubules?
A. *Processes of dentinoblasts
B. Processes of enameloblasts
C. Bodies of dentinoblasts
D. Fibroblasts
E. Elastic fibers

Dental caries is characterized by a decalcification of the enamel. What is the sequence of the
process of decalcification?
A. *Shell of enamel rods, interred enamel, enamel rods
B. Enamel rods, interred substance, shell of enamel rods
C. Interrod substance, enamel rods, shell of enamel rods
D. Enamel rods, interrod substance
E. Interrod substance, enamel rods

The cuboidal shape cells with well-developed rough endoplasmic reticulum are determined in a
histological specimen in the area of the tooth root on the outer surface of the dentine. What tissue
do they form?
A. *Cementum
B. Dentin
C. Enamel
D. Loose connective tissue
E. Dense connective tissue

There is adding of calcium into enamel after the eruption of the tooth. What stage of enamel
formation has these characteristics?
A. *Tertiary mineralization
B. Secondary mineralization
109
C. Secretion stage
D. Primary mineralization
E. Maturation

The interprismatic enamel is represented in a histological specimen. What cells form it?
A. *Secretory active ameloblasts
B. Secondary ameloblasts
C. Preodontoblasts
D. Primary ameloblasts
E. Secretory active odontoblasts

The germ of the tooth is presented in the histological specimen of the embryo oral cavity. Specify
from what elements it consists of?
A. *Enamel organ, dental papilla, dental follicle
B. Enamel organ and dental papilla
C. Dental papilla and dental follicle
D. Enamel organ and dental follicle
E. Enamel organ

Linear striation in the form of concentric circles, which are directed at the angle to dentinoenamel
junction, was found during histological examination of enamel cross-section during histological
examination of enamel cross-section. Name these structures.
A. *Contour lines of Retzius
B. Hunter-Schreger bands
C. Enamel lamellae
D. Enamel tufts
E. Enamel spindles

In the process of development of the tooth in the periodontum are stored the remains of the
embryonic Hertwig’s root sheath that are called epithelial cells rests of Malassez. They can be a
source of development of cysts or tumors in the tooth root region. What cells formed Hertwig’s root
sheath?
A *Cells of enamel organ
B Mesenchyma cells
C Cells of the pulp
D Odontoblasts
E Cementoblasts

DIGESTIVE SYSTEM. SALIVARY GLANDS.

Tests of the “KROK-1” database

2015 р.
(тестові завдання з даної теми відсутні)

2014 р.
(тестові завдання з даної теми відсутні)

2013

Only serous end parts are detected in a histological specimen of the glandular organ. In interlobular
connective tissue ducts are seen, lined with stratified epithelium. Determine this structure.
110
A *Parotid gland
B Submandibular salivary gland
C Pancreas
D Sublingual salivary gland
E Liver

A microspecimen of the submandibular salivary gland shows some basket-shaped cells


concentrated around the acinus and excretory ducts. These cells surround bases of the serous cells
and are called myoepitheliocytes. These cells relate to the following tissue:
A *Muscle tissue
B Epithelial tissue
C Nervous tissue
D Special connective tissue
E Loose fibrous connective tissue

2012

Parotid salivary gland has acinar pieces which are formed by serous cells. What organelles of these
cells provide synthesis and secretion of the saliva components?
A. Granular endoplasmic reticulum and Golgi complex
B. Golgi complex
C. Smooth endoplasmic reticulum and Golgi complex
D. Mitochondria, Golgi complex
E. Lysosomes

2005-2011

Damage of the epithelium of excretory ducts is observed in chronic inflammatory processes of the
salivary glands. What kind of epithelium will be damaged in the striated ducts of the major salivary
glands?
A. *Columnar epithelium with basal striations
B. Squamous epithelium with basal striations
C. Cuboidal epithelium with basal striations
D. Pseudostratified epithelium with basal striations
E. Stratified cuboidal epithelium

Lobules ducts with a large number of orderly arranged mitochondria in the basal part of the
epithelium are visible in a histological specimen of the parotid gland. What kind of ducts is this?
A. *Striated ducts
B. Common ducts
C. Interlobular ducts
D. Intercalated ducts
E. Intralobular ducts

Terminal secretory acini with serous cells that synthesize mostly enzymes are distinguished in a
histological specimen of the parotid gland. What kind of glands according to chemical composition
of their secretion are they?
A *Protein
B Mucous
C Protein and mucous
D Sebaceous
E Sweat
111
It is known that submandibular salivary gland has mucous secretory acini consisting of mucous
cells. What features are characteristic for these cells?
A *Flattened nucleuses and light cytoplasm
B Basophilic cytoplasm
C Rounded nucleus in the center of the cell
D Microvilli
E Basal striations

Processes of secretion and excretion are violated during acute inflammation of the parotid gland.
Which cells are suffered in this case?
A *Serous and myoepithelial cells
B Protein, serous, mucous cells
C Serous cells, cells with basal striations, stellate cells
D Protein and mucous cells
E Cells with basal striations

It is known that submandibular salivary gland has mucous secretory acini consisting of mucous
cells. What feature is characteristic for mucous cells?
A. *Oxyphilic cytoplasm
B. Basophilic cytoplasm
C. Rounded nucleus in the cell center
D. Microvilli
E. Basal striations

The secret of paired parotid glands is realized into the mouth. What acini is the part of parotid
gland?
A. *Protein secretory acini
B. Mucous secretory acini
C. Epithelial trabeculae
D. Follicles
E. Mixed secretory acini

The protein synthesizing system of salivary gland cells was blocked using actinomycin D. What
component will be absent in saliva? What cells will cease to secrete its secret?
A. *Enzyme amylase, serous cells
B. Enzyme amylase, mucous cells
C. Enzyme lipase, serous cells
D. Enzyme pepsin, serous cells
E. Enzyme lipase, mucous cells

Cells with round nuclei and basophilic cytoplasm are presented in a specimen of parotid glands
secretory acini. What structures of the gland do they form?
A. *Protein secretory acini
B. Mucous secretory acini
C. Sebaceous secretory acini
D. Follicles
E. Mixed secretory acini

The damage of the epithelium of excretory ducts is observed during chronic inflammatory processes
in the parotid gland. What kind of epithelium covers interlobular ducts of the parotid gland in
normal condition?
112
A. *Pseudostratified, then – stratified squamous
B. Simple cuboidal, then - columnar
C. Simple columnar, then – simple squamous
D. Stratified ciliated, then - glandular
E. Stratified squamous, then – stratified columnar

The student was asked a test about the structure of the salivary glands. In the test says that in the
salivary gland except secretory cells there are myoepithelial cells, which are able to contract. What
is the source of development for myoepithelial cells?
A *Epidermal
B Neural
C Mesenchymal
D Celomic
E Somatic

Secretory acini are visible in a histological specimen. It is known that these cells synthesize and
secrete protein. How is stained cytoplasm of these cells?
A *Basophilic
B Oxyphilic
C Not stained
D Metachromatic
E Polychromatic

DIGESTIVE SYSTEM. ESOPHAGUS.

Tests of the “KROK-1” database

2015 р.
(тестові завдання з даної теми відсутні)

2014

There is presented a transverse section of the wall of a hollow organ whose mucosa is covered with
stratified squamous non-keratinized epithelium in a histological specimen. What organ is this?
A. Esophagus
B. Duodenum
C. Large intestine
D. Uterus
E. Appendix

2013
(тестові завдання з даної теми відсутні)

2012

The process of epithelium cornification is detected during the morphological analysis of the mucous
coat of esophagus biopsy material. What epithelium covers the mucous coat of esophagus?
A. Stratified squamous non-keratinized
B. Simple squamous
C. Pseudostratified villous
D. Simple columnar
E. Stratified squamous keratinized
113
Mucosal epithelium was damaged in the patient as a result of burns of the esophagus by vinegar.
What cellular structure of the epithelium is the source of reparative regeneration?
A *Basal cells
B Squamous cells
C Spiny cells
D Ciliated cells
E Endocrine cells

2005-2011
It was found that in 78% of cases the location of ulcers was typical in patients with ulcers of the
esophagus in the surgical department during the retrospective study of case histories. The researches
attributed this to the presence in these areas of cardiac glands. What areas of the esophagus are
often damaged?
A. *Level of cricoid cartilage of the larynx
B. Level of the sixth cervical vertebra
C. At the transition of striated muscle tissue into smooth muscle tissue
D. In place of the esophagus passage through the diaphragm
E. -

Microscopic examination of a hollow organ revealed that its mucous membrane is covered with
stratified squamous nonkeratinized epithelium and the lamina propria contains simple tubular gland,
secretory acini of which consist mainly of mucous cells and small amount of parietal cells. What
organ is this?
A *Esophagus
B Stomach
C Small intestine
D Trachea
E Urethra

Striated muscle tissue of the hollow organ of the digestive system is represented for microscopic
study. What organ was taken for biopsy?
A. *Esophagus
B. Stomach
C. Small intestine
D. Ileum
E. Appendix

DIGESTIVE SYSTEM. STOMACH.

Tests of the “KROK-1” database

2015 р.
(тестові завдання з даної теми відсутні)

2014

Relatively large cells with acidophilic cytoplasm in the glands of the histological section of the
stomach bottom can be seen. One can see in these cells complicate system of intercellular canaliculi
in electron microscopy. What gastric juice component is formed due these cells activity?
114
A. Hydrochloric acid
B. Mucus
C. Pepsinogen
D. Gastrin
E. Serotonin

A first-year-old child has disorders of mother’s milk suction. With activity disorder of what gastric
proper (fundic) glands cells is it connected?
A. Chief cells
B. Mucous neck cells
C. Enteroendocrine cells
D. Parietal cells
E. Undifferentiated cells

Areal damaging of gastric epithelium happened under influence of harmful factors. With what cells
help possible its regeneration?
A. Mucous neck cells
B. Parietal cells
C. Chief cells
D. Enteroendocrine cells
E. Mucous cells of the glands (Undifferentiated cells)

The damage of endothelial layer of mucous membrane was found during endoscopic examination of
the stomach. What cells provide the reparative regeneration?
A *Undifferentiated cells
B Accessory cells
C Chief cells
D Parietal cells
E Glandular epithelium

A patient has malignant anemia developing after gastrectomy. Absence of what gastric glands cells
causes this pathology?
A. Parietal cells
B. Chief cells
C. Neck mucous cells
D. Enteroendocrine cells
E. Goblet cells

A patient with chronic gastritis went for endogastric pH-metry that reveal decreased acidity of the
gastric juice. It indicates diminished function of the following cells:
A *Parietal cells
B Chief cells
C Enteroendocrine cells
D Mucous neck cells
E Accessory cells

Examination of a 43 y.o. patient revealed that his stomach has difficulties with digestion of protein
food. Gastric juice analysis revealed low acidity. Which gastric cells function is disturbed in this
case?
A *Parietal cells
B Chief cells
C Mucous neck cells
115
D Enteroendocrine cells
E Mucous neck cells

It was found out that glands contain very small amount of parietal cells or they are totally absent
during histological examination of the stomach. What part of the stomach mucosa was studied?
A *Pyloric part
B Fundus of stomach
C Cardiac part
D Body of stomach
E-

2013

Section of alimentary canal organ was revealed in the histological specimen. Relief of this organ is
represented by foveolae (gastric pits). Pits surface were covered with epithelium in which all the
cells lays on the basal membrane, have columnar shape and apical part of the cells is filled with
droplets of mucous secrete. Determine, which organ has this epithelium?
A. *Stomach
B. Small intestine
C. Large intestine
D. Esophagus
E. Appendix

Alimentary canal organ is revealed in the histological specimen. Wall of this organ constitutes 4
layers: mucosa, submucosa, muscularis externa and serosa. Mucosa has fields and rugae (folds).
Determine which organ has this relief.
A. Stomach
B. Duodenum
C. Large intestine
D. Esophagus
E. Appendix

An organ was revealed in the histological specimen. Simple tubular glands located in the lamina
propria of this organ mucosa. These glands include mainly chief, parietal, mucous neck and
endocrine cells. Name the type of glands.
A. *Fundic glands of the stomach
B. Pyloric glands of the stomach
C. Cardiac glands of the stomach
D. Esophageal glands propria
E. Esophageal cardiac glands

Signs of hypochromic anemia were found in the patient with chronic atrophic gastritis. Disorder of
what stomach glands cells function can explain development of anemia?
A. *Parietal
B. Chief
C. Mucous neck
D. Endocrine
E. Undifferentiated

A 20-year-old patient has been prescribed a prolonged aspirin intake because of rheumatism. What
structural component of the stomach mucosa mostly provides its defense from the damage?
A. *Simple cuboidal glandular epithelium
116
B. Connective
C. Muscle
D. Stratified villous
E. Stratified squamous non keratinized

Large oval shaped cell is detected in the electron microphotography of the stomach fundic gland.
There is a lot of intercellular secretory canaliculus, huge amount of mitochondria. Name this cell.
A. *Parietal
B. Chief
C. Undifferentiated
D. Mucous
E. Exocrine

Mucosal epithelium is damaging in the stomach during inflammatory diseases. What epithelium is
damaged?
A. *Simple columnar glandular
B. Simple squamous
C. Simple cuboidal microvillous
D. Simple cuboidal
E. Stratified cuboidal

Amount of glandulocytes (cells of the stomach) with oxiphylic cytoplasm were increased in the
bioptat of patient suffering from ulcer during histological examination of the stomach mucosa.
What component of gastric juice provides these cells?
A. *Hydrochloric acid
B. Mucus
C. Pepsinogen
D. Gastrin
E. Secretin

Surgical removal of a part of stomach resulted in disturbed absorption of vitamin BI 12. It is


excreted with feces. The patient was diagnosed with anemia. What factor is necessary for
absorption of this vitamin?
A *Gastromucoprotein
B Gastrin
C Hydrochloric acid
D Pepsin
E Folic acid

A newborn develops dyspepsia after the milk feeding. When the milk is substituted by the glucose
solution the dyspepsia symptoms disappear. The newborn has the subnormal activity of the
following enzyme:
A *Lactase
B Invertase
C Maltase
D Amylase
E Isomaltase

2012
Pernicious anemia caused by the damage of cells that produce antianemic factor was developed in
48-year-old patient after radiotherapy for cancer of the stomach. What cells of the gastric glands are
affected in this case?
117
A *Parietal cells
B Chief cells
C Mucous neck cells
D Endocrine cells
E Accessory cells

A 45-year-old patient is hospitalized with the complaint about pain in the stomach. Gastroscopy has
detected small ulcers in the area of gastric fundus. What cells of the stomach impairment function
became a reason for damage of mucosa?
A. Cells of superficial epithelium with mucous secretion (mucous neck cells)
B. Parietal cells of stomach glands that secrete chlorides and ions of hydrogen
C. Chief cells that secret pepsinogen
D. Endocrine cells which secrete somatostatin
E. Endocrine cells which secrete serotonin

Total hyperacidity was identified in a 56-year-old woman during pH-metry of the gastric juice.
What cells of the gastric glands have impaired function?
A *Parietal cells
B Chief cells
C Mucous neck cells
D Accessory cells
E Endocrine cells

When the pH level of the stomach lumen decreases to less than 3, the antrum of the stomach
releases peptide that acts in paracrine fashion to inhibit gastrin release. This peptide is:
A *GIF
B Acetylcholine
C Gastrin-releasing peptide (GRP)
D Somatostatin
E Vasoactive intestinal peptide (VIP)

2005-2011

There is high content of mucus, which impedes digestion in the stomach. What cells have the
violation of the functional activity?
A. *Mucous cells
B. Neck cells
C. Mucous cells of the propria stomach glands
D. Endocrine cells
E. Parietal cells

There is a reduce amount of HCL and pepsin in the analysis of the gastric juice. What hormone of
the gastric glands has impaired production?
A *Gastrin
B Somatostatin
C Serotonin
D Histamine
E Glucagon

The patient has ulcerative defect of the stomach wall as a result of uncontrolled intake of
nonsteroidal anti-inflammatory drugs. What cells are affected?
A. *Glandular cells
118
B. Goblet cells
C. Chief cells
D. Parietal cells
E. Ehrlich cells

45-year-old man consulted a doctor with complaints about disorder of the stomach function. It was
found a neoformation from epithelial tissue during a comprehensive examination. Which cells most
likely given the development of the tumor?
A. *Mucous neck cells
B. Chief cells
C. Endocrine cells
D. Accessory cells
E. Parietal cells

Insulin injection to assess completeness of vagotomy is accompanied by a significant increase of


acidity of gastric juice. What cells of the gastric glands provide this phenomenon?
A. *Parietal cells
B. Endocrine cells
C. Chief cells
D. Mucocytes
E. Neck cells

Insufficient amount of mucus covering the mucosa was revealed in the patient during gastroscopy.
What cells of the stomach have impaired function?
A. *Cells of columnar glandular epithelium
B. Parietal cells of the stomach glands
C. Chief cells
D. Neck cells
E. Endocrine cells

Small cells with a high nuclear-cytoplasmic ratio and basophilic cytoplasm are detected during
histological examination of the neck of the fundic stomach glands. Specify the function of these
cells.
A. *Regeneration of glandular epithelium
B. Protective function
C. Endocrine function
D. Secretion of chloride ions
E. Secretion of pepsinogen

The skin epithelium originates from the ectoderm. It is stratified or pseudostratified epithelium.
What localization is not typical for it?
A *Stomach
B Oral cavity
C Esophagus
D Vagina
E Cornea

Analysis of biopsy material of human stomach mucosa in a patient with gastritis showed a sharp
decrease in the number of parietal cells. How is it reflected on the following components of gastric
juice?
A. *Hypoacidity
B. Hyperacidity
119
C. Hyperproduction of gastric juice
D. Hypoproduction of gastric juice
E. Reduction of mucus production

60-year-old patient treated chronic gastritis for a long time. Changes in the epithelium of the
mucosa are revealed during endoscopy of the stomach. What kind of epithelium was changed?
A. *Simple columnar glandular
B. Simple columnar ciliated
C. Pseudostratified
D. Simple squamous
E. -

The patient, who among other complaints has heartburn, was made a biopsy of the gastric mucosa.
Numerous cells with oxyphilic cytoplasm with some cells containing two nuclei are determined in a
histological specimen. What are these cells?
A *Parietal cells
B Chief cells
C Mucocytes
D Epithelial cells
E Endocrine cells

Large irregular, spherical cell in the cytoplasm of which one can determine the intracellular
canaliculi, surrounded by a large number of mitochondria is presented in electron micrographs of a
fragment of the fundic stomach gland. What does this cell produce?
A *Ions of Сl- і H+
B Mucus
C Pepsinogen
D Renin
E Gastrin

Muscularis externa consisting of three layers of smooth muscle cells is identified in the preparation
of a hollow organ. The mucous membrane is covered with a simple columnar glandular epithelium
and has a three-layer muscular plate. Name this organ.
A *Stomach
B Heart
C Uterus
D Esophagus
E Urinary bladder

DIGESTIVE SYSTEM. INTESTINE.

Tests of the “KROK-1” database

2015

It was revealed during autoradiography examination that total regeneration of small intestine
epithelium has been taking place during 3 days as a result of active proliferation of undifferentiated
cells. Indicate their localization.
A. Bottom of crypts
B. Villi apex
C. Villi base
120
D. Lateral side of villi
E. Lamina propria of mucosa

2014

Clustered cells were found in a histological specimen of small intestine wall at the bottom of crypts.
There are large acidophilic secretory granules in the apical part of the cells. Cytoplasm is colored
basophilic. What cells are these?
A. Paneth cells
B. Intermediate cells
C. Endocrine cells
D. Goblet cells
E. Enterocytes

Terminal parts of serous glands are located in a histological specimen of small intestine in the
submucosal base. What part of the intestine is presented in the specimen?
A. Duodenum
B. Small intestine
C. Ileum
D. Appendix
E. Jejunum

2013

Disorder of digestion and absorption of proteins in the small intestine has been found in the patient
with chronic enterocolitis (intestine inflammation) as a result of insufficient quantity of dipeptidase
in the intestine juice. In what cells synthesis of these enzymes are disordered?
A. *Paneth cells
B. Enterocytes
C. Intermediate cells
D. Goblet cells
E. Enteroendocrine cells

Absorptive function suffers during diseases of small intestine mucosa. What epithelium is
responsible for this function?
A. *Simple columnar with striated border
B. Simple cuboidal
C. Simple columnar villous
D. Stratified squamous
E. Stratified cuboidal

Disorder of parietal and membranous digestion was found during examination of the patient with
small intestine disease. What function cells disorder is connected with this?
A. *Enterocytes
B. Intermediate cells
C. Goblet cells
D. Paneth cells
E. Enteroendocrine cells

Absence of specific structures of small intestine relief has been seen in the patient with chronic
enterocolitis (intestine inflammation) during endoscopic examination. What components designate
peculiarities of this organ mucosa relief?
121
A. *Plicae circulares, villi and crypts
B. Fields, rugae and pits
C. Haustrae, villi and crypts
D. Obliquely circular folds
E. Fields, villi

Certain diseases of small intestine are connected with the function disorder of exocrine cells with
acidophil granules (Paneth cells). Where are these cells located?
A. *In the bottom of intestinal glands (crypts)
B. In the apical part of intestinal villi
C. On lateral surfaces of intestinal villi
D. In the apical part of intestinal glands (crypts)
E. In the place of villi transition into the glands

The proportion between epithelial cells of mucosa changed in case of certain diseases of large
intestine. What types of cells dominate in crypts epithelium of large intestine in normal condition?
A. *Goblet cells
B. Enterocytes
C. Enteroendocrine cells
D. Paneth cells
E. Undifferentiated cells

Proctoscopy has shown a tumor proceeding from the mucosa of caudal part of rectum. Of what
epithelium has this tumor formed?
A. *Stratified squamous non-keratinized
B. Simple columnar glandular
C. Simple cuboidal
D. Simple cuboidal with microvilli
E. Transitional

2012

An accumulation of spherical cells with large basophilic nuclei, which are surrounded by a narrow
rim of cytoplasm were found in the preparation of small intestine in the lamina propria of the
mucosa. The central part is light and contains fewer cells than peripheral in most of these clusters.
What morphological structures are these clusters?
A *Lymphatic nodules
B Nerve ganglion
C Adipose cells
D Blood vessels
E Lymphatic vessels

Ulcer of small intestine was developed in 39-year-old patient after radiotherapy because of the
hepatoma. Ulcer was caused by the inhibition of mytotic activity of the cells, which are responsible
for regeneration of small intestine surface epithelium. Inhibition of what cells mitotic activity does
this patient have?
A. *Crypt columnar cells without margins
B. Columnar cells
C. Endocrine cells
D. Goblet cells
E. Enterocytes with acidophilic granules
122
The rate of small intestine epithelium renovation of a patient is reduced. With the damage of what
cells is it connected?
A. Columnar non-border epithelial cells (intermediate cells)
B. Paneth cells
C. Endocrine cells
D. Goblet cells
E. Enterocytes

An infectionist has detected an acute enterocolitis syndrome with infringement of the processes of
breakdown and absorption of nutritive materials of a patient. Damage of what intestinal epithelium
cells cause this disorder?
A. Columnar epitheliocytes with microvilli (enterocytes)
B. Non-microvilli epitheliocytes (intermediate cells)
C. Goblet cells
D. Paneth cells
E. Endocrine cells

Significant infringement of the regeneration process of the small intestine mucosa epithelial layer of
a cancer patient is detected by means of morphological research after X-ray therapeutics. What
epithelial layer cells are damaged?
A. Columnar non-border epithelial cells (intermediate cells)
B. Enterocytes
C. Goblet cells
D. Endocrine
E. Paneth cells

There are mucosal layer of an organ in the histological specimen. Enterocytes and goblet cells can
be detected on the villi surface of epithelial lining. What organ consists from these cells?
A. Small intestine
B. Stomach
C. Large intestine
D. Urethra
E. Bronchi

2005-2011
A 70-year-old patient for a long time treated chronic duodenitis. Changes in the mucosal epithelium
of the duodenum were revealed during endoscopic examination. What kind of epithelium was
changed?
A *Simple columnar
B Stratified squamous nonkeratinized
C Simple squamous
D Pseudostratified ciliated
E Simple columnar glandular

Histological slice of the small intestine was made. Simple columnar epithelium covering the villi is
defined in the preparation. What functions of the small intestine will be violated by its damage?
A *Covering, absorption
B Excretion
C Pinocytosis
D Bowel motility
E Gas exchange
123
A part of mucosa was taken during biopsy research of the small intestine wall. What kind of
epithelium covers the surface of this organ mucosa?
A. *Simple columnar epithelium with striated border
B. Simple cuboidal
C. Simple columnar ciliated
D. Simple columnar glandular
E. Stratified squamous nonkeratinized

Granules in which by histochemical methods were identified dipeptidase and lysozyme were found
in the cytoplasm of crypts epithelial cells of the small intestine. Specify these cells.
A. *Paneth cells
B. Enterocytes
C. Goblet cells
D. А- cells
E. S- cells

Electron-dense granules at the basal pole of the cell are detected in the electron micrographs of
duodenum enterocytes. What do they contain?
A. *Secretin
B. Trypsin
C. Absorbed proteins
D. Adipose inclusions
E. Lipofuscin

Electron-dense granules at the basal pole are clearly defined in electron micrographs of the
duodenal epithelium cells. What are these cells?
A. *Endocrine cells
B. Columnar cells with striated border
C. Undifferentiated cells
D. Goblet cells
E. Parietal cells

The wall of an organ of the digestive system, in the lamina propria and submucosa of which there
are numerous lymphoid nodules is presented in a histological specimen. Name this organ.
A. *Appendix
B. Stomach
C. Duodenum
D. Jejunum
E. Colon

Intramural ganglion of the Auerbach’s plexus of the small intestine was removed in the experiment.
What changes will occur in the functional activities of the small intestine?
A *Disorder of the motor function
B Disorder of malabsorption of nutrients
C Disorder of the endocrine function
D Disorder of the secretory function
E Disorder of all functions

A portion of the small intestine was removed from the patient during surgery for tumor. What cells
will provide the regeneration of muscle tissue of the intestinal wall in the region of the seam?
A *Smooth muscle cells
124
B Adipose cells
C Myosatellitocytes
D Fibrocytes
E Fibroblast

It was revealed that most often the inflammation of the appendix (appendicitis) happens after viral
infections. What morphological features of the appendix wall structure contribute to it?
A. *Contains many clusters of lymphoid tissue
B. Contains a large number of goblet cells in the epithelium
C. Has a highly developed muscular layer
D. Absence of the muscularis mucosa
E. Has a well-developed villi

DIGESTIVE SYSTEM. LIVER.

Tests of the “KROK-1” database

2015

Cells of a healthy liver actively synthesize glycogen and proteins. What organelles are the most
developed in these cells?
A. Granular and agranular endoplasmic reticulum
B. Cell center
C. Lysosomes
D. Mitochondria
E. Peroxisomes

A specimen of a parenchymal organ shows poorly delineated hexagonal lobules surrounding a


central vein, and the interlobular connective tissue contains embedded triads (an artery, a vein and
an excretory duct). What organ is this?
A. Liver
B. Pancreas
C. Thymus
D. Spleen
E. Thyroid

2014
A parenchymal organ with lobular structure is presented in a histological specimen. Each lobule is
roughly hexagonal block of tissue consisting of anastomosing plates with sinusoidal capillaries.
They are lying between the plates and radially converging to the central vein. What anatomic organ
has such morphological structure?
A. *Liver
B. Pancreas
C. Thymus
D. Spleen
E. Lymph node

Ultramicroscopic examination of “dark”; hepatocyte population in the cell cytoplasm detected well
developed granular endoplasmic reticulum. What function has this organelle in these cells?
A *Synthesis of blood plasma proteins
125
B Carbohydrate synthesis
C Detoxification
D Bile production
E Calcium ion depositing

A viral infection has damaged cells that form walls of bile capillaries. This stimulated conditions for
inflow of bile into the blood of sinusoidal capillaries. What cells are damaged?
A *Hepatocytes
B Kupffer’s cells
C Ito cells
D Pit-cells
E Endotheliocytes

2013

Disorder of blood circulation in classic lobule has been seen in the liver parenchyma during
connective tissue overgrowth as a result of chronic diseases. What is the direction of blood
circulation in these lobules?
A. *From periphery to the center
B. From center to the periphery
C. Around lobule
D. From apex to the bottom
E. From bottom to the apex

Anomaly of the liver development has been found during examination of the patient. What
embryonal source was damaged?
A. *Endoderm of the middle part of the primary gut
B. Endoderm of posterior wall of the gut
C. Endoderm of the anterior gut
D. Mesonephral duct
E. Endoderm of the posterior gut

Parenchymal organ has been seen in the histological slide. Lobules are structural and functional unit
of it. They don’t have clear margins, possess central vein in the center, radial oriented stacks and
intralobular sinusoidal capillaries. Lobule is limited by interlobular arteries, veins and bile ducts.
Name, what organ has these morphological signs.
A. *Liver
B. Thyroid gland
C. Pancreas
D. Parotid gland
E. Kidney

2012

There is a large quantity of carbohydrates in the dietary intake of a human. What structures will be
seen in the cytoplasm of hepatocytes?
A. *Glycogen granules
B. Lipid droplets
C. One large lipid drop
D. Lipofuscin inclusions
E. Increasing of ribosome quantity

126
2005-2011
Reduced in size liver, kidneys and heart were found during autopsy of a 67-year-old patient, who
died from lung cancer. The liver on the section was with brown shade. The golden-brown granules
of lipofuscin were found during microscopic examination of hepatocytes. Name the structure that
forms the granules of lipofuscin.
A *Autophagolysosomes
B Golgi apparatus
C Ribosomes
D Endoplasmic reticulum
E Mitochondria

A diffuse growth of connective tissue, dilatation of bile ducts, excessive accumulation of bilirubin
granules in the cytoplasm of hepatocytes and its necrosis were found during histological
examination of sectioned material (liver) of a 64-year-old patient with diagnosis of cancer of the
pancreatic head. What organelles damage can cause the delay of bilirubin excretion by hepatocytes?
A *Golgi apparatus
B Lysosomes
C Plasma membrane
D Endoplasmic reticulum
E Nucleus

Violation of the integrity of the membrane of lysosomes in most cells of the liver is observed in the
patient, who was poisoned with CCL4. What will be the effect of the poison on the liver cells?
A. *Developing autolysis that will lead to the cell death
B. Not affected
C. Will occur phagocytosis
D. Will occur pinocytosis
E. Will occur exocytosis

The bile can get into the bloodstream, causing jaundice due to the damage junctions between
hepatocytes which arising from some pathological processes. What types of intercellular junctions
are damaged in this case?
A. *Zonula occludens, gap junction, desmosomes
B. Plasma membrane infoldings (finger-like) and gap junction
C. Synapses and desmosomes
D. Desmosomes and gap junction
E. Gap junction and zonula occludens

The cell in the cytoplasm of which there are granules with a density similar to the fruit bone is
represented in electron micrographs of intralobular sinusoid of the liver. It is known that this cell is
a natural killer. What kind of cell is represented?
A. *Ріt- cell
B. Hepatocyte
C. Endothelial cell of the sinusoid capillary
D. Stellate macrophage
E. Perisinusoidal lipocyte

Damaged rough endoplasmic reticulum was found in hepatocytes as a result of exposure of


hepatotropic poison. Synthesis of what substances will be disordered in the liver epithelium?
A. *Albumins and fibrinogen
B. Phospholipids
C. Glycogen
127
D. Cholesterol
E. Vitamins

As a result of stab wounds of the liver, hepatic artery was cut, however in liver lobules the blood
continued to flow. What vessel provided the flow of blood into lobules?
A. *Perilobular vein
B. Interlobular vein
C. Perilobular artery
D. Sublobular vein
E. Hepatic vein

Intralobular capillaries have wide irregular lumen throughout its length in a specimen of the liver.
The basement membrane in most of the capillary wall is absent. What type of capillaries are these?
A *Sinusoidal
B Visceral
C Somatic
D Precapillaries
E Postcapillaries

Golgi apparatus was destroyed in hepatocytes as a result of the damaging effects of hepatotropic
poison. Synthesis of what substances will be disordered in the liver?
A *Bile
B Glycogen
C Albumins and fibrinogen
D Vitamins
E Cholesterol

DIGESTIVE SYSTEM. PANCREAS.

Tests of the “KROK-1” database

2015

Glucose is transported inside the cell from extracellular space through plasma membrane before it
utilization. This process is stimulated by the following hormone:
A. Insulin
B. Glucagon
C. Thyroxin
D. Aldosterone
E. Adrenalin

2014
A 14-year-old patient has diabetes mellitus. What cells of the pancreas do not function?
A *В -cells
B А - cells
C D - cells
D D1- cells
E РР - cells

128
55-year-old patient is observed by endocrinologist apropos decreased function of the pancreas that
manifested by decreasing of hormone glucagon in the blood. What cells function is disturbed in this
case?
A. α-cells of Langerhans islets
B. β- of Langerhans islets
C. D-cells of Langerhans islets
D. PP-cells of Langerhans islets
E. EC-cells of Langerhans islets

2013

50-year- old patient complains of increased appetite, thirsty, decreased body weight and fatigue.
Increased blood glucose level was revealed during laboratory examination. What cells function
disorder is connected with development of this disease?
A. *β-cells
B. α-cells
C. Thyrocytes
D. Acinar cells
E. Lipotropocytes

There is gland in the histological slide. There are acini in the lobules, secretory cells of which have
2 zones: basal – homogeneous basophilic and apical – zymogen oxiphylic. What organ has these
morphological signs?
A. *Pancreas
B. Liver
C. Parotid salivary gland
D. Submandibular salivary gland
E. Sublingual salivary gland

Certain cells of pancreas are in the permanent condition of exertion in people that incline to
excessive using of sweet. What cells are these?
A. *β-cells
B. α-cells
C. D cells
D. PP cells
E. EC cells

The β cells of endocrine portion of pancreas are selectively damaged by alloxan poisoning. How
will it be reflected in blood plasma?
A *The content of sugar increases
B The content of fibrinogen decrease
C The level of sugar decreases
D The content of globulins decreases
E The content of albumins decreases

2012

Massive damage of centroacinar cells was detected in the analysis of a patient after acute
pancreatitis. With help of what cells is its regeneration possible?
A. Intercalated duct cells
B. Islets Langerhans cells
C. Cells of the stroma
129
D. Vessel endothelium
E. Interlobular duct cells

2005-2011

The substance, which selectively damages A-cells of the pancreatic islet (cobalt salts), was injected
to the animal. What function will be disrupted?
A. *Synthesis of glucagon
B. Synthesis of insulin
C. Synthesis of somatostatin
D. Synthesis of cholecystokinin
E. Synthesis of pancreozymin

The alloxon, which selectively damage B-cells of pancreatic islets, was injected to the animal. What
function of the pancreas will be disrupted?
A *Synthesis of insulin
B Synthesis of glucagon
C Synthesis of somatostatin
D Synthesis of pancreatic polypeptide
E Synthesis of vasointestinal polypeptide

A group of cells is defined in a histological specimen of the pancreas. Some of them are located
centrally and have basophilic secretory granules. Their secretion regulates carbohydrate
metabolism. Name these cells
A. *В- cells
B. РР- cells
C. А- cells
D. Adipose cells
E. Д- cells

RESPIRATORY SYSTEM. RESPIRATOTY AIRWAYS.

Tests of the “KROK-1” database

2015
Cleaning of the mucous membrane of the respiratory tract from dust and microorganisms
occurs due to the mucociliary transport – the movement of mucus by the epithelium
surface. What cells provide the clearing mechanism?
A.Ciliated and goblet cells
A.Brush cells
B. Bronchiolar exocrinocytes
C. Endocrine and basal cells
Е. Dendritic cells

2014

The changes of the epithelium are observed at the patient after prolonged inflammation of the nasal
cavity mucous membrane. What epithelium has been changed?
A * Simple pseudostratified
B Simple squamous
130
C Stratified squamous
D Stratified cuboidal
E Stratified columnar

The glands, cartilage islands and pseudostratified columnar ciliated epithelium were founded during
the histological examination of the bronchus wall. What kind of bronchus was examinated?
A * Middle bronchus
B Large bronchus
C Primary bronchus
D Small bronchus
E Terminal bronchioles

Pathological process mainly localized in the bronchus was found during the autopsy of the 65-year-
old dead man, who suffered from pulmonary disease. The glands, cartilage islands and
pseudostratified columnar ciliated epithelium were clearly visible at the histological examination of
the bronchus. What kind of bronchus was injured?
A * Middle bronchus
B Large bronchus
C Primary bronchus
D Small bronchus
E Terminal bronchioles

The mucous membrane that covers the upper part of the upper nasal turbinate was damaged as a
result of the nasal trauma of a 30-year-old man. What consequence it will result to?
A * Disorders of odorant perception
B Disorders of air humidification
C Disorders of goblet cells secretory activity
D Disorders of air warming
E Disorders of air warming and humidification

A patient was admitted to the hospital with an asphyxia attack provoked by a spasm of smooth
muscles of the respiratory tracts. This attack was mainly caused by alterations in the following parts
of the airways:
A *Small bronchi
B Median bronchi
C Large bronchi
D Terminal bronchioles
E Respiratory part

2013

Real diphtheritic croup is accompanied with deposition of fibrous membranes on the true vocal
folds which are tightly connected with epithelium. What type of epithelia covers mucosa of these
cords?
A. *Stratified squamous nonkeratinized
B. Stratified squamous keratinized
C. Pseudostratified ciliary
D. Simple squamous
E. Simple cuboidal

The child breathed a button, which was removed from the right main bronchus by the
bronchoscope. What epithelium of bronchus was probably damaged by the foreign objects?
131
A * Pseudostratified ciliated
B Stratified squamous nonkeratinized
C Simple cuboidal
D Transitional
E Simple squamous

The ciliary and goblet cells, which form the mucociliary complex, are located in epithelium which
covers respiratory airways. What is the function of this complex?
A * Clearing of the air from the dust particles
B Secretion of the hormones
C Warming of the air
D Humidification of the air
E Respiration

There is an increased quantity of slime in respiratory passages of a chemical production worker


after breathing with poisonous steams. What of respiratory tract epithelial cells participate in
mucosa moistening?
A *Goblet cells
B Fibroblasts
C Endocrine cells
D Langerhans cells
E Intercalated cells

A pathological process in bronchi resulted in epithelium desquamation. What cells will provide
regeneration of bronchial epithelium?
A *Basal
B Intercalary
C Ciliate
D Endocrinal
E Goblet

A 35-year-old patient applied to a doctor with complaints about having intense rhinitis and loss of
sense of smell for a week. Objectively: nasal cavity contains a lot of mucus that covers mucous
membrane and blocks olfactory receptors. In what part of nasal cavity are these receptors situated?
A *Superior nasal turbinate
B Median nasal turbinate
C Inferior nasal turbinate
D Common nasal meatus
E Vestibule of nose

2012

The patient consulted a doctor with complaints of nasal cavity dryness. The dysfunction of the nasal
cavity mucous glands was founded during the investigation. What layer of the nasal cavity mucous
membrane contains these glands?
A * Lamina propria of mucosa
B Epithelial lamina of mucosa
C Lamina muscularis of mucosa
D Submucosa layer
E Fibrous cartilage lamina

132
The organ is represented in the histological slide. Its wall consists of mucosa, submucosa, fibro-
cartilage and adventitia tunics. It is covered with pseudostratified ciliated epithelium. There are
glands with mucous-protein secretion in the submucosa layer. Hyaline cartilage forms large plates.
What organ has the same morphological characteristics?
A * Large bronchus
B Esophagus
C Trachea
D Larynx
E Small bronchus

The malignant epithelial tumor of middle bronchus was diagnosed at 66-year-old patient. What kind
of epithelium was the source of this tumor?
A * Pseudostratified ciliated
B Stratified squamous nonkeratinized
C Stratified squamous keratinized
D Transitional
E Simple cuboidal

The malignant epithelial tumor of trachea was diagnosed at the patient 56 years old. What kind of
epithelium was the source of this tumor?
A * Pseudostratified ciliated
B Stratified squamous nonkeratinized
C Stratified squamous keratinized
D Transitional
E Simple cuboidal

The wall of the trachea was damaged during intubation. What kind of epithelium was damaged?
A * Pseudostratified ciliated
B Stratified squamous nonkeratinized
C Stratified squamous keratinized
D Transitional
E Simple cuboidal

The terminal bronchioles are revealed at the histological examination of the lung. What kind of
epithelium covers these bronchioles?
A * Simple cuboidal ciliated
B Stratified squamous nonkeratinized
C Pseudostratified ciliated
D Simple cuboidal
E Transitional

A tubular organ was revealed on the histological preparation of the respiratory system. It is covered
with a low epithelium, has well developed muscular layer. The glands and cartilage are absent.
Name this organ.
A * Small bronchus
B Trachea
C Larynx
D Main bronchus
E Middle bronchus

133
A tubular organ was revealed on the histological preparation of the respiratory system. The wall of
it consists of simple cuboidal ciliated epithelium, muscle lamina is composed by smooth muscle
cells, and mucosa folds are absent. What is the organ?
A *Terminal bronchus
B Small bronchus
C Middle bronchus
D Large bronchus
E Main bronchus

The desquamation of epithelium occurs as result of the pathological process in bronchus. What cell
will make the bronchial epithelium regeneration?
A * Basal
B Mucus
C Ciliary
D Endocrine
E Goblet

The death of the bronchus ciliated epithelial cells developed at the chemical industry worker after
inhaling of the caustic miasma. What cell will provide the bronchial epithelium regeneration?
A * Basal
B Mucus
C Ciliary
D Endocrine
E Goblet

Low cells of the oval or triangular shape are visible in a histological specimen of the trachea in the
structure of pseudostratified ciliated epithelium. Its top does not reach the apical surface of the
epithelium in some cells are visible mitotic figures. What is the role of these cells?
A *Source of regeneration
B Part of the mucociliary complex
C Secret mucus
D Secret surfactant
E Produce biologically active substances

2005-2011

The patient was admitted to the hospital with asthma attack that started at night. There are cyanosis
and pallor of the patients’ face. Name the divisions of the airways, which are associated with
disruption of the normal functioning of lungs in this case.
A *Small sized bronchi
B Medium sized bronchi
C Large bronchi
D Terminal bronchioles
E Respiratory part

The patient with bronchial asthma has an attack of difficulty breathing. Where in the bronchial tree
one can find a spasm of muscle cells which cause this phenomenon?
A *Small sized bronchi
B Terminal bronchioles
C Medium sized bronchi
D Large bronchi
134
E Respiratory bronchioles

Inflammation of the lungs is frequently observed in a 2-year-old child. What organelles of the
epithelium of bronchi have impaired function?
A *Cilia
B Mitochondria
C Endoplasmic reticulum
D Microvilli
E Lysosomes

A contraction of the muscle wall of one part of the bronchial tree is observed with bronchial asthma
under the influence of biologically active substances. Which bronchus responds to this allergic
reaction?
A *Small bronchus
B Principal bronchus
C Large bronchus
D Terminal bronchioles
E Alveolar ducts

The loss of ciliated epithelial cells of bronchi has occurred in the worker of chemical manufacture
after inhalation of poisonous vapor. What cells of the mucosa will be provide the regeneration of
ciliated cells?
A *Basal cells
B Goblet cells
C Endocrine cells
D Langerhans cells
E Fibroblasts

What part of the airways has such characteristic: pseudostratified ciliated epithelium is gradually
transformed into simple, cartilaginous islands and glands are absent, well-developed muscularis
mucosa?
A *Small bronchi
B Medium sized bronchi
C Large bronchi
D Terminal bronchioles
E Trachea

Attacks that are characterized by difficult expiration – expiratory dyspnoea are observed in the
patients with bronchial asthma under the influence of allergens. Specify the place of the action of
allergens.
A *Small bronchi
B Terminal bronchioles
C Respiratory bronchioles
D Medium sized bronchi
E Large bronchi

Low cells of oval or triangular shape are detected in the histological preparation of the trachea, in
the structure of the surface epithelium of the mucosa. Cells have high nuclear-cytoplasmic ratio, its
apex does not reach the surface of the epithelium, and in some cells are detected mitotic figures.
Name these cells.
A *Basal cells
B Ciliated cells
135
C Endocrine cells
D Goblet cells
E Clara cell

Pseudostratified columnar epithelium, composed of cells forming the mucociliary apparatus is


determined in a histological specimen of the trachea. Identify these cells.
A *Ciliated and goblet cells
B Basal and goblet cells
C Basal and ciliated cells
D Goblet and Clara cells
E Clara cells and ciliated cells

Wall of the trachea was damaged during intubation. What layers of the trachea wall are damaged in
this case?
A *Mucosa, submucosa, fibro-cartilaginous, adventitial
B Mucosa, submucosa, muscular, adventitia
C Mucosa, fibro-cartilaginous, muscular, adventitia
D Mucosa, fibro-cartilaginous, adventitia
E Mucosa, submucosa, fibro-cartilaginous, serous

RESPIRATORY SYSTEM. RESPIRATOTY AIRWAYS.

Tests of the “KROK-1” database

2015

It is known that work in a mine causes inhalation of large amounts of coal dust. Inhaled coal dust
can be detected in the following pulmonary cells:
A. * Alveolar macrophages
B. Respiratory epithelial cells
C. Secretory epithelial cells
D. Capillary endothelial cells
E. Pericapillary cells

Alveolar space of acinus was invaded by bacteria that interacted with the surfactant. This led to the
activation of the cells that localized in the alveolar walls and on the surface. Name these cells:
A. * Alveolar macrophages
B. Alveolocytes type I
C. Endothelial cells
D. Clara cells
E. Alveolocytes type II

2014

The auscultation of a patient with dry pleurisy has revealed sound of pleural friction. What
epithelium type can cause such signs?
A. * Simple cuboidal epithelium
B. *Simple squamous epithelium
C. Transitional epithelium
D. Simple columnar epithelium
E. Pseudostratified
136
The cells that take place at the blood-air barrier were found at the alveoli microphotography. What
are these cells?
A * Respiratory epithelial cells of the alveoli
B Secretory epithelial cells of the alveoli
C Alveolar macrophages
D Clara’s cell
E Ciliated epithelial cells

The respiratory distress syndrome develops at the premature newborn. Lack of what air-blood
barrier component is related with this disease?
A * Surfactant
B Endothelium of capillaries
C Endothelial basement membrane
D Basement membrane of alveolocyte
E Alveolocyte

There are cells with a domed apical part and microvilli on it at the respiratory epithelium. The
synthetic apparatus of this cell is well developed. Apical part of this cell contains the secretory
granules. Name this cell.
A * Clara’s cell
B Goblet cell
C. Endocrine cell
D Basal
E Cambial

The structures in the form of open vesicles are represented on the microphotography. The inner
surface of it is covered by a simple epithelium that consists of respiratory and secretory cells. What
structure is this?
A * Alveolus
B Bronchioles
C Acinus
D Alveolar ducts
E Terminal bronchus

Lung of premature infant is presented on electronic photomicrography of biopsy material. Collapse


of the alveolar wall caused by the deficiency of surfactant was revealed. Dysfunction of what cells
of the alveolar wall caused it?
A *Alveolocytes type II
B Alveolocytes type I
C Alveolar macrophages
D Secretory cells
E Fibroblasts

Electronic microphotography of pulmonary alveoli’s wall presents a big cell. Its cytoplasm has a lot
of mitochondria, developed Golgi apparatus, osmiophil lamellated corpuscles. What is the main
function of this cell?
A *It produces surfactant
B It is a component of blood-air barrier
C It warms the air
D It purifies the air
E It absorbs microorganisms
137
2013 р.

It is known that an important component of the air-blood barrier is alveolar surfactant complex,
which prevents the falling of the alveoli during the expiratory. What cell is synthesizing the
phospholipids surfactant membranes formation?
A * Epitheliocyte type II
B Respiratory cells
C Ciliated epithelial cells
D Alveolar macrophages
E Capillary endothelium

The structure that consist of surfactant, alveolocyte type I, basement membrane and the fenestrated
endothelium of the capillaries was represented on the microphotography. Which barrier between
blood and tissue was represented?
A *Air-blood
B Blood brain
C Blood thymic
D Filtration apparatus
E Blood testis

The bacteria penetrated into the alveolar space and affected the surfactant. This activated the cells
localized in the walls of the alveoli and on their surface. What are these cells?
A Alveolar macrophages
B Alveolocyte type I
C Endothelial
D Clara’s cells
E Alveolocyte type II

2012

One was unable to induce the first breath in the newborn in the maternity ward. Analyzing the
causes of death it was established that the airways are free, but lungs are not expanded. What is the
cause of non expansion of the lungs in this case?
A *Lack of surfactant
B Constriction of the bronchi
C Rupture of the bronchi
D Pleural thickening
E Increase in size of the alveoli

The injury of the cells responsible for respiratory function was found in the microscopic lung
preparation of human suffering from pneumonia. What cells are these?
A * Alveolocyte type I
B Alveolocyte type II
C Macrophages
D Clara’s Cells
E Lymphocytes

A benign epithelial tumor of the visceral pleura was diagnosed in a 48-year-old man. What
epithelium is a source of this tumor development?
A * Simple squamous
B Stratified nonkeratinized
138
C Pseudostratified ciliated
D Transitional
E Stratified keratinized

2005-2011

Processes of the alveolar epithelium differentiation impaired in a 8-year-old child during intensive
formation of lung tissue under the influence of diseases. What consequences it will lead to?
A *Violation of large alveolocytes proliferation and production surfactant
B Violated production of surfactant
C Violation of refractory epithelial cells proliferation
D Violation of proliferation of ciliary cells proliferation
E-

X-ray examination of the patient with anthracosis revealed areas of darkening and in the sputum –
the presence of dust particles. What cells of the alveoli capture dust particles?
A *Alveolar macrophages
B Respiratory epithelial cells
C Ciliated cells
D Brush cells
E-

Preterm infants often have pneumonia because the second type (secretory) alveolocytes do not
perform their function. What is the role of the secretory alveolocytes?
A *Synthesis of surfactant membranes
B Phagocytosis
C Synthesis of enzymes that break down the surfactant
D Synthesis of hormones
E Respiratory function

Name cells that participate in the formation of the surfactant alveolar complex.
A *Second type alveolocytes
B First type alveolocytes
C Epithelial cells of the respiratory bronchioles
D Clara cells
E Ciliated cells

Areas of collapsed lung tissue as a result of insufficient surfactant system are possible during
inflammatory lung diseases. Name the areas where surfactant covers the inner surface of the
respiratory tract?
A *Alveolus
B Terminal bronchiole
C Lobular bronchus
D Segmental bronchus
E Lobar bronchus

It is known that preterm infants, who were born earlier than 34 weeks of development, die more
often with symptoms of respiratory distress than babies born after 36 weeks of development. How
can we explain this fact?
A *Insufficient amount of surfactant
B Formation of insufficient number of acini
C Impaired function of macrophages
139
D Hypoplasia of the vascular system of the lungs
E Violation of innervation

URINARY SYSTEM. MORPHOLOGY OF THE KIDNEY

Tests of the “KROK-1” database

2015
(questions are absent)

2014

An important part of the renal filtration barrier is a three-layer basement membrane, which has a
special mesh structure of its middle layer. Where is situated this basement membrane?
A * In the renal corpuscles
B In the capillaries of the peritubular capillary net
C In the proximal tubules
D In the thin tubules
E In the distal tubules

Electron microscopy of kidneys revealed tubules, which are lined by cuboidal epithelium. There are
light and dark cells in the epithelium. Light cells have a few organelles. The cytoplasm forms folds.
These cells provide the reabsorption of water from primary urine into the blood. Dark cells in
structure and function resemble parietal cells of the stomach. What tubules are presented on the
electronogram?
A *Collecting renal tubules
B Proximal tubules
C Distal tubules
D Ascending tubules loop of Henle
E Descending tubules loop of Henle

The blood cells and fibrinogen was detected in the urine of 35-year-old woman with kidney disease.
It was probably associated with the renal filter function. What is the structural composition of this
filter?
A * Endothelium of glomerular capillaries, three-layered basement membrane, podocytes
B Three-layer basement membrane
C Endothelium of capillaries, basement membrane
D Podocytes, basement membrane
E Endothelium, podocytes

A histological specimen of kidney shows a structure consisting of a glomerulus of fenestrated


capillaries and a bilayer epithelial capsule. Specify this structure:
A *Renal corpuscle
B Proximal tubule
C Distal tubule
D Henle’s loop
E Receiving tube

2013
140
The leached erythrocytes were detected in the patient urine. What part of the nephron was
damaged?
A * Renal corpuscle membrane
B Proximal tubule
C Loop of Henle
D Distal tubule
E Composite renal tube

Biopsy material of the kidney is examined by the method of electron microphotography. The
fenestrated endothelium with basement membrane is revealed on the microphotography. There are
dendritic epithelial cells from the outer side of the basement membrane. What formation of the
kidney is represented on the microphotography?
A * Filtrating barrier
B Renal corpuscle
C Proximal part of nephron
D Distal part of nephron
E Loop of nephron

2012

The presence of albumin (albuminuria) and glucose (glucosuria) in the secondary urine is observed
in a patient with suspected glomerulonephritis for two weeks. What part of the kidney function is
disordered?
A * Proximal tubules
B Distal tubules
C Thin tubule
D Collecting tubules
E Juxtaglomerular apparatus

The injury of the distal nephron part epithelium was made at the experimental model on the rats.
What functional processes in the kidney will be weakened?
A * Reabsorption of water and electrolytes
B Reabsorption of glucose
C Reabsorption of sodium and glucose
D Reabsorption of protein
E Filtration

Residues of proteins and glucose are detected in the analysis of urine of the 27-year-old patient.
What part of the nephron is affected?
A *Proximal tubule
B Ascending tubule loop of Henle
C Descending tubule loop of Henle
D Distal tubule
E Nephron

30 g of glucose was found in the patient urine in case of normal its amount in the blood. What
structural and functional mechanism of kidney was damaged?
A * Process of reabsorption in the proximal part of the nephron
B Process of filtration
C Process of reabsorption in the distal part of the nephron
D Process of reabsorption in the thin tubules
141
E Process of reabsorption in the distal part as a result of ADH insufficiency

The structures covered by cuboidal epithelium with the brush border and deep plasmolemma folds
in the basal part ware determined at the cortex of the kidney. A large mitochondria number is
situated between the plasmolemma folds. What part of nephron is described?
A * Proximal tubule
B Straight distal tubule
C Tortuous distal tubule
D Loop of Henle
E Renal corpuscles

The damage of the first four branches of somits to the right happened in the process of
embryogenesis. What organ will have the impaired development?
A *Development of pronephros
B Development of liver
C Development of pancreas
D Development of right adrenal gland
E Development of spleen

The podocytes can be damaged as result of kidney disease. What functional changes will develop as
result of it?
A * Filtration of protein will increase
B Filtration of protein will decrease
C Secretion of renin will increase
D Secretion of renin will decrease
E Secretion of prostaglandins will increase

The large epithelial cell with large and small processes was represented in the electron
microphotography of the kidney fragment. The large and small processes are attached to the
basement membrane of capillaries. What cell is this?
A * Podocytes
B Juxtavascular cell
C Smooth myocyte
D Endotheliocyte
E Mesangial cell

The blood cells don’t appear in the urine in normal condition. What nephron structure prevent their
appearance in the urine?
A * Basement membrane of glomerular capillaries
B Juxtavascular cell
C Mesangial cell
D Outer layer of the glomerulus capsule epithelium
E Loop of Henle epithelium

The subacid urine reaction was revealed at the patient. What kidney cells provide such urine
reaction?
A * Secretory cells of collecting tubules
B Juxtaglomerular cells of cortical nephrons
C Juxtavascular cells of cortical nephrons
D Macula densa cells
E Interstitial cells of stroma
142
The nephrons, which are situated on the border between the cortex and medulla and have the same
diameter and extension afferent and efferent arterioles, were revealed on the kidney preparation.
What function of kidney will be disordered in case of these nephrons injury?
A * Blood shunting at intensification of blood supply
B Synthesis of renin
C Synthesis of prostaglandins
D Synthesis of erythropoietin
E Sodium receptor activity
The cuboidal shaped cells, the apical surface of which contains brush border and basal surface has
basal striations with mitochondria arranging between invaginations of cytolemma are determined in
electron micrographs of one part of the nephron. Name the part of the nephron.
A *Proximal tubule
B Collecting renal tubules
C Distal tubule
D Thin tubule
E Glomerulus of the nephron

A threat of development of ascending infection of the urinary tract appeared in the patient after
exogenous intoxication as a result of the loss of acid urine reaction. What cells of kidneys were
damaged in this case?
A *Dark cells of collecting tubules
B Light cells of collecting tubules
C Epithelial cells of proximal tubules
D Squamous cells of loop of Henle
E Epithelial cells of distal tubules

2005-2011

The patient’s analysis of urine shows a significant amount of protein. What part of the nephron has
disordered processes of urine formation?
A *Proximal convoluted tubule
B Collecting tubules
C Distal convoluted tubules
D Thin tubule
E distal straight tubule

0,1 – 0,25 mm renal corpuscles surrounded by two leaflets, which limits wide, compared to the
norm, slit cavity having a shape of a bowel were revealed in the preparation kidneys removed
during surgery. Which structure would manifest these changes?
A *Capsule of the renal corpuscle
B Renal corpuscle
C Vascullar glomerulus
D Afferent arteriole
E Efferent arteriole

Constant hypercalcemia, hypercalciuria and hypophosphatemia were found in a 38-year-old patient


with nephrolithiasis. The disease was complicated by bilateral aggravation of chronic
pyelonephritis. On the background of progressive chronic renal failure the patient was died.
Histological examination discovered lime deposits in the convoluted tubule of kidneys and in lungs.
Determine ultrasrtuctures in which calcium is deposited.
A *Mitochondria
143
B Ribosomes
C Endoplasmic reticulum
D Golgi apparatus
E Nucleus

Fenestrated endothelium, three-layered basement membrane, on the outer surface of which located
branched epithelial cells with filtration slits between the branches are determined in the kidney’s
biopsy material during electron microscopic study. Name this formation.
A *Filtration barrier
B Nephron
C Juxtaglomerular apparatus
D Mesangium
E Medullary rays

СЕЧОВИДІЛЬНА СИСТЕМА. СЕЧОВИВІДНІ ШЛЯХИ. ЕНДОКРИННИЙ АПАРАТ


НИРКИ.

Тестові завдання з бази даних «КРОК-1»

2015 р.

Histological specimen of a kidney demonstrates cells closely adjoined to the renal corpuscle in the
distal convoluted tubule. Their basement membrane is extremely thin and has no folds. These cells
sense the changes in sodium content of urine and influence renin secretion occurring in
juxtaglomerular cells. Name these cells:
A. Macula densa cells
B. Juxtaglomerular cells
C. Mesangial cells
D. Podocytes
E. Glomerular capillary endothelial cells

2014

A histological specimen of a kidney shows a part of the distal tubule going between the afferent and
efferent arteriole. The cells constructing the tubule wall have dense nuclei; basal membrane is
absent. Such structural formation is called:
A *Macula densa
B Juxtaglomerular cells
C Mesangial cells
D Juxtavascular cells
E-

The stable increase in blood pressure was made in the experiment by the constriction of the renal
artery. The function of what kidney cells can results this effect?
A * Juxtaglomerular cells
B Podocytes
C Endothelial
D Interstitial cells
E Macula densa cells
144
The cells are tightly adjacent to the renal corpuscles were revealed at the histological preparation of
kidney distal convoluted tubule. Their basement membrane is very thin and do not form the folds.
These cells feel the changes of sodium content in the urine and affect the renin secretion by the
juxtaglomerular cells. What are these cells?
A * Macula densa cells
B Juxtaglomerular cells
C Mesangial cells
D Podocytes
E Endothelium glomerular capillaries

2013

The electronic microphotography of kidney fragment has exposed an afferent glomerular arteriole,
which has giant cells under its endothelium, containing secretory granules. Name the type of these
cells:
A *Juxtaglomerular
B Mesangial
C Smooth muscular
D Juxtavascular
E Interstitial

Examination of a 43 y.o. anephric patient revealed anemia symptoms. What is the cause of these
symptoms?
A *Reduced synthesis of erythropoietin
B Enhanced destruction of erythrocytes
C Iron deficit
D Vitamin B 12 deficit
E Folic acid deficit

Considerable increase of systematic arterial pressure was observed a month after surgical
constriction of rabbit’s renal artery. What of the following regulation mechanisms caused the
animals’ pressure change?
A *Angiotensin-II
B Vasopressin
C Adrenaline
D Noradrenaline
E Serotonin

The cells with large secretory granules in the cytoplasm were found at the kidney electron
microphotography in the wall of afferent and efferent arteries. Identify the structural formation of
the kidney, which consists of these cells.
A * Juxtaglomerular apparatus
B Renal corpuscle
C Proximal part of nephron
D Distal part of nephron
E Loop of nephron

The low specific gravity of the secondary urine (1002) was found out in the sick person. What is the
most distant part of nephron where concentration of secondary urine takes place?
A *In the collecting duct
B In the nephron’s glomerulus
145
C In proximal tubule of nephron
D In ascending part of loop of Henle
E In distal tubule of nephron

A patient has a decreased vasopressin synthesis that causes polyuria and as a result of it evident
organism dehydratation. What is the mechanism of polyuria development?
A *Reduced tubular reabsorption of water
B Reduced tubular reabsorption of Na ions
C Reduced tubular reabsorption of protein
D Reduced glucose reabsorption
E Acceleration of glomerular filtration

2012

Anemia was developed at a 50-year-old patient with chronic nephritis. What is the reason of anemia
development at this patient?
A * Reduced production of erythropoietin
B Absence of cancer
C Lack of vitamin B12
D Disorders of porphyrin synthesis
E Immunological injury of the cells - precursors of erythropoiesis

The patient with kidney disease has increased blood pressure. What structures of kidneys cause
these symptoms?
A *Juxtaglomerular cells
B Cells of proximal tubules
C Cells of distal tubules
D Cells of the macula densa
E Cells of the loop of Henle

2005-2011
Very often high blood pressure is one of the symptoms of kidney damage. What cells responsible
for the implementation of these symptoms?
A *Juxtaglomerular cells
B Juxtavascular cells
C Cells of the macula densa
D Interstitial cells
E Fibroblasts

Hypertension with elevated renin in the blood was revealed in a 40-year-old patient. What cells of
kidneys produce this hormone?
A *Juxtaglomerular cells
B Cells of the macula densa
C Juxtavascular cells
D Mesangial cells
E Interstitial cells

MALE REPRODUCTIVE SYSTEM. TESTIS.

146
Tests of the “KROK-1” database

2015
(questions are absent)

2014

The changes in the nucleus and cytoplasm of spermatids are observed during the one of the
spermatogenesis phases that leads to the formation of mature gametes. What is the phase of
gametogenesis?
A * Formation
B Maturation
C Growth
D Reproduction
E Proliferation

A disorder of the tests epithelium was developed as result of the scrotum mechanical trauma. What
epithelium was damaged?
A * Simple cuboidal
B Ciliated
C Simple columnar
D Stratified
E Transitional

The organ that is covered by albuginea and serous membranes from outside was represented in the
histological section. Organ stroma consists from loose connective tissue that contains the Leydig
cells. The parenchyma is represented by the tubules covering with spermatogenic epithelium from
the inner surface. What organ is this?
A * Testes
B Epididymis
C Prostate
D Mammary gland
E Ovaries

2013

Destroyed wall of convoluted tubules is marked at man after the testis mechanical trauma. What
will it lead to?
A * Aspermatogenesis
B Polyspermia
C Increasing of testosterone synthesis
D Monospermia
E Reduction of testosterone synthesis

2012

The insufficient number of germ cells was found in the seminal fluid of a 25-year-old patient during
the study. Which cells of male gonads provide a sufficient number of sperm for fertilization?
A *Spermatogonia
B Sustentacular cells
C Supporting cells
D Sertoli Cells
147
E Leydig cells

The couple complains of inability to have children. Injury of testes spermatogenic epithelium that
leads to the absence of sperm in the semen and infertility was found after the man examination.
What part of the testis was injured?
A * Seminiferous tubuli (Ductuli seminiferi contorti)
B Rete testis
C Tubuli recti (Ductuli seminiferi recti)
D Ductuli efferentes testis
E Ductus epididymis

The cells of male sex glands begin to produce the male sex hormone testosterone that causes the
appearance of secondary sex characteristics during the puberty. What cells of male gonads produce
this hormone?
A * Leydig cells
B Sustentacular cells
C Sertoli Cells
D Supporting cells
E Spermatozoa

The production of sperm was disordered as result of orchitis at 43-year-old men. What part of the
testis was injured?
A * Seminiferous tubuli (Ductuli seminiferi contorti)
B Rete testis
C Tubuli recti (Ductuli seminiferi recti)
D Ductuli efferentes testis
E Ductus epididymis

2005-2011

What factors contribute to divergence of development of the reproductive system?


A. *Sex chromosomes and hormone inhibin
B. Sex chromosomes
C. Inhibin
D. Somatic chromosomes
E. Testosterone

Large sized, rounded or polygonal shape cells with acidophilic cytoplasm containing numerous
lipid granules are found in a histological specimen of the testis between the seminiferous tubules.
What is produced by these cells?
A. Testosterone
B Inhibin
C Progesteron
D Estrogen
E-

Olygosperm (insufficient number of sperm) was revealed in an ejaculate of the liquidator of an


accident at the nuclear power plant during microscopy examination. What structures of the
reproductive system, in which spermatogenesis occurs, were damaged?
A *Primary spermatocytes
B Tubuli recti
C Rete testis
148
D Efferent ductules
E Ductus epididymis

Large, tetraploid cells which are located in recesses of the supporting cells are visible in a
histological specimen of the seminiferous tubule. They come into the prophase of the first meiotic
division. What cells are these?
A *Primary spermatocytes
B Secondary spermatocytes
C Spermatogonia type A
D Spermatogonia type B
E Spermatids

An increased level of testosterone was found in the blood during examination of the patient with
endocrine pathology. What cells in the male body are responsible for the production of this
hormone?
A *Glandular cells of testis
B Sustentacular cells of testis
C Spermatogenic cells
D Cells of the prostate gland
E Cells of the seminal vesicles

Groups of large irregular rounded shape cells with light nucleus and acidophilic cytoplasm are
determined in a histological specimen of the testis in loose connective tissue between the
seminiferous tubules. Determine the function of these cells.
A *Testosterone production
B Trophic function
C Estrogens production
D Barrier function
E Supporting function

Epithelial cells have axoneme in one of the part of the excurrent ducts of the mucosa. Specify the
part of the excurrent duct system.
A *Rete testis and efferent ductules
B Rete testis
C Efferent ductules and ductus epididymis
D Ductus epididymis and ductus deferens
E Ductus deferens and ejaculatory duct

Cells at different stages of spermatogenesis were determined in the biopsy of the testis in the wall of
seminiferous tubules. What is a name of these cells after the growth phase?
A *Primary spermatocyte
B Spermatogonia type A
C Sperm
D Secondary spermatocyte
E Spermatozoon

Cystic formation of the pelvis has been removed after surgery. Histological examination determined
that it originates from the right seminal vesicles. What kind of epithelium covers the cavity of
seminal vesicles?
A *Simple columnar
B Simple squamous
C Simple cuboidal
149
D Stratified cuboidal
E Stratified squamous

Cells with large nuclei, increased glycogen content and high activity of alkaline phosphatase are
found in the wall of yolk sac. These cells migrate through the vessels into sex cushions. What cells
of the male reproductive system will be differentiate from them?
A *Spermatogenic cells
B Leydig cells
C Fibroblasts
D Blood cells
E Supporting cells

ЧОЛОВІЧА СТАТЕВА СИСТЕМА. ДОДАТКОВІ СТАТЕВІ ЗАЛОЗИ.


СІМ᾿ЯВОНОСНІ ШЛЯХИ.

Тестові завдання з бази даних «КРОК-1»

2015
(тестові завдання з даної теми відсутні)

2014

The cells of male sexual glands begin to produce male sex hormone testosterone which induces
development of secondary sexual signs during pubescence. What cells of male sexual glands
produce this hormone?
A Leydig cells
B Sustentacular cells
C Sertoli cells
D Supporting cells
E Spermatozoa

2013 р.
(тестові завдання з даної теми відсутні)

2012 р.

One can see tubules in cross section of the histological preparation. The wall of these tubules
consists of mucosa, tunica muscularis and tunica adventitia. The epithelium of the mucosa is
represented by high columnar cells with stereocilia and low stem cells. What organ of the male
reproductive system was studied?
A *Ductus epididymis
B Testis
C Prostate gland
D Ejaculatory duct
E Urethra

2005-2011 рр.

Muscular-glandular organ, which has a lobular structure, is represented in a histological specimen.


Separate glands located in the lobules. The excretory ducts of these glands open into the canal,
located in the center of this organ. The wall of this canal is lined by transitional epithelium. The
stroma of this organ consists of connective tissue and muscle tissue. Smooth muscle cells are
150
located around glands forming longitudinal and circular layers. Reduction of these cells facilitates
the removal of the glands secrete. Name the organ.
A. *Prostate gland
B. Testis
C. Seminal vesicles
D. Epididymis
E. Mammary gland

The mucosal epithelium is pseudostratified in a histological specimen of a part of excurrent duct


system. Columnar cells with stereocilia on the apical surface interspersed with intercalated cells.
There are tunica muscularis and tunica adventitia. What part of the excurrent duct system is
represented?
A *Ductus epididymis
B Tubuli recti
C Seminiferous tubules
D Ductus deferens
E Ejaculatory duct

FEMALE REPRODUCTIVE SYSTEM. OVARY.

Tests of the “KROK-1” database

2015
(questions are absent)

2014
(questions are absent)

2013

The structures with a large cavity were defined at the histological slide of a female ovary. Primary
oocyte is surrounded by a zona pellucida, corona radiata and situated at the cumulus oophorus. The
wall is formed by a layer of follicular cells and theca. What structure of the ovary has the same
morphological features?
A * Mature (tertiary) follicle
B Primordial follicle
C Primary follicle
D Yellow body
E Atretic body

The round-shaped structure that consists of large glandular cells containing pigment lutein was
found at the histological slide of a female ovary. The small connective tissue scar is in the center of
this structure. What structure is this?
A * Yellow body
B Matured follicle
C Atretic body
D Secondary follicle
E White body

Normal embryo implantation is possible only in case of corresponding changes of the endometrium
of the uterus. Amount of what endometrial cells will increase in this case?
151
A * Decidual cells
B Fibroblasts
C Neurons
D Macrophages
E Myocytes

Oocyte surrounded with 1-2 layers of cuboidal follicular cells and scarlet covering around it have
been detected in the ovary specimen colored with hematoxylin-eosin. Name this follicle:
A *Primary
B Primordial
C Secondary
D Mature
E Atretic

2012

The endocrine function of ovary follicular cells was disordered as result of inflammation. Synthesis
of what hormone will be suppressed?
A * Estrogen
B Progesterone
C Lutropin
D Follicle stimulating hormone
E Folistatin

The polygonal cells are rich in lipids and glycogen was revealed in the mucosa lamina propria in the
endometrial biopsy of a healthy woman, taken in the secretory phase of the menstrual cycle. What
are these cells?
A * Decidual cells
B Smooth myocytes
C Endothelial cells of the damaged vessels
D Myofibroblasts
E Fibroblasts

The corpse of the unknown woman was delivered for the forensic examination. The rounded
structure with a diameter of 5 cm, containing a yellow pigment was revealed on the ovary section.
Pathological changes in the ovary were not founded. What cells this formation is composed from?
A * Lutein
B Follicular
C Interstitial
D Muscle
E Fibroblasts

A cyst of the ovary was found at 50-year-old woman. From what structure was it developed?
A * From follicle
B From cortex stroma
C From atretic body
D From white body
E From interstitial cells

The bright pink irregularly shaped structures (hematoxylin-eosin staining) are occurring at the
section of a normal ovary. What is the reason of these structures formation?
A * Atresia of the follicle
152
B Formation of the corpus luteum
C Ovulation
D Formation of the white body
E Necrosis of the follicle

An increased estrogens amount was found in the blood of a woman. What ovary cells take place at
these hormones formation?
A * Interstitial and follicular cells of secondary follicles
B Oocytes
C Follicular cells of primary follicles
D Follicular cells of primordial follicles
E Follicular cells and oocytes

2005-2011

Changes in the structure of the endometrium were found during microscopic examination of biopsy
material of women suffering from infertility as a result of the action of hormone progesterone.
Where is this hormone produced?
A. *Yellow body of the ovary
B. Ovarian follicles
C. Anterior pituitary
D. Posterior pituitary
E. Hypothalamus

Round shaped bodies containing luteal cells are revealed in the female ovary after ovulation. What
is produced by these cells?
A *Progesterone
B Estrogen
C Lutropin
D Follitropin
E Testosterone

Primordial and primary follicles were found in the histological preparation of ovarian cortex. At
what stage of oogenesis are they formed?
A *Small growth stage
B Large growth stage
C Maturation stage
D Reproductive stage
E Formation stage

ЖІНОЧА СТАТЕВА СИСТЕМА. ОВАРІО-МЕНСТРУАЛЬНИЙ ЦИКЛ.

Тестові завдання з бази даних «КРОК-1»

2015 р.

Atretic bodies and developed yellow body can be observed along with follicles of various orders in
an ovary specimen. What stage of ovarian and menstrual cycle is characterized by the described
ovary condition?
A. Premenstrual
153
B. Menstrual
C. Postmenstrual
D. Regeneration
E. Follicle growth

2014

Histological picture of the endometrium has the following features: thickening, edema, uterus
glands are corkscrew shaped with enlarged lumen and produce large amount of mucus, mitosis in
the cells are not observed, decidual cells are present in the stroma. What is the stage of the
menstrual cycle?
A * Secretory (premenstrual)
B Menstrual
C Regenerative
D Proliferative
E Relative rest

There is prolongation of the large follicle growth phase at the patient with pituitary adenoma (the
tumor of adenohypophysis). What is the length of the oocyte large growth phase in norm?
A * 12-14 days
B Several decades (from 10-13 to 40-50) after the birth
C After the birth until the puberty
D From 3 months of prenatal development to the birth
E 28 days

The hyperemia of the ovary, increased blood-follicular barrier permeability with next edema
development, infiltration of the follicle wall by the segmented leukocytes is observed at the female.
The follicle volume is large. The wall of it is very thin. What is the stage of the menstrual cycle?
A * Preovulatory stage
B Ovulation
C Menstrual stage
D Postmenstrual stage
E Relative rest stage

A diagnostic biopsy of the endometrium was made to the patient with infertility at the gynecologcal
department. At the microscopic examination t was revealed that the mucosa is edematous, uterine
glands are corkscrew shaped and filled with a thick secret. What hormone excess amount will lead
to such changes in the endometrium?
A * Progesterone
B Estrogen
C Testosterone
D Somatotropin
E ACTH

2013
(questions are absent)

2012

A low concentration of estrogen and high level of progesterone was revealed in the blood of not
pregnant 26-year-old women. At what stage of menstrual cycle the analysis was made?
A * Premenstrual phase (secretory)
154
B Menstrual phase
C Postmenstrual phase (proliferative)
D Phase of desquamation
E Proliferation phase of endometrium

The bleeding was stopped after the childbirth as a result of oxytocin action on the uterus wall. What
component of the uterine wall takes the main part in it?
A * Middle layer of the myometrium
B Endometrium
C Inner layer of the myometrium
D Superficial layer of the myometrium
E Perimetrium

The bleeding was stopped after the childbirth as a result of oxytocin action on the uterus wall. What
component of the uterine wall is reacting on the oxytocin action?
A * Myometrium
B Endometrium
C Perimetrium
D Parametrium
E Submucosa

2005-2011

It was found that the level of progesterone is approaching the lower limit of normal and the level of
estrogens is approaching the upper limit of normal during analyzing the blood of women. At what
stage of menstrual cycle was taken the blood test?
A. *In the phase of desquamation
B. In the early phase of proliferation
C. During ovulation
D. In the mid-phase of secretion
E. In the late phase of secretion

The patient was diagnosed with a collapse of the ovary during examination. Specify in which of the
following phases of the menstrual cycle occurs the ovulation.
A. *Relative rest phase
B. Menstrual phase
C. Postmensrtual phase
D. Premenstrual phase
E. During the entire menstrual cycle

There are atrophy of the endometrium and myometrium of the uterus, atrophy of the follicles in the
ovary, sclerotic changes of the blood vessels of these organs in women in the menopause. What
factors cause these changes?
A *Insufficiency of lutropin
B Excess of follitropin
C Excess of lutropin
D Excess of lactotropin
E Excess of estrogens

It was found that the level of hormones progesterone and estrogen is approaching the lower limit of
normal during analysis of the woman blood. At what stage of the cycle was taken a blood test?
A *Menstrual
155
B Postmenstrual phase (regenerative)
C Postmenstrual phase (proliferative)
D Relative rest phase
E Premenstrual

ЖІНОЧА СТАТЕВА СИСТЕМА. СТАТЕВІ ШЛЯХИ.

Тестові завдання з бази даних «КРОК-1»

2015 р.
(тестові завдання з даної теми відсутні)

2014 р.
(тестові завдання з даної теми відсутні)

2013 р.
(тестові завдання з даної теми відсутні)

2012 р.

Human embryo consisting from two blastomeres was found. Name the place of its localization
under the condition of its normal development.
A *Uterine tube
B Uterus
C Abdominal cavity
D Endometrium
E Ovary

The cessation of bleeding after birth relates with the influence of hormones on the structures of
uterus. What structural component of the uterine wall is involved in this process?
A *The middle layer of the myometrium
B Endometrium
C The inner layer of the myometrium
D The superficial layer of the myometrium
E Perimetrium

The cessation of bleeding after birth relates with the action of oxytocin on the uterine wall. What
layer of the uterus responds to the action of this hormone?
A *Myometrium
B Endometrium
C Perimetrium
D Parametrium
E Submucosa

2005-2011 рр.
Cyclic changes of epithelium were found in histological examination of smears from the vagina of
37-year-old women during the menstrual cycle. What kind of epithelium lines the vaginal wall?
A *Stratified squamous nonkeratinized
B Simple columnar glandular
C Pseudostratified ciliated epithelium
D Transitional epithelium
E Stratified cuboidal
156
157

You might also like